Sie sind auf Seite 1von 336

----------------------- Page 1----------------------CHEMISTRY (Mock Test-1) 41. emistry) (a) NH OH 4 (c) NH3 42.

The ratio of between the first to that between atom in H-atom is (a) 4/9 (b) (b) CH F 3 Which of the following the least tendency bonds between molecules ? compounds has to form hydrogen Mock Test1 (Ch

(d) HF the difference in energy and second Bohr orbit second and third Bohr : 1/3 (c) 27/5 (d) 1/2

43.

How many molecules are present in one gram of hydrogen ? 23 (a) 301 10 (b) 15 10 23 (c) 25 10 (d) 602 10 The shape of covalent molecule AX (a) T-shape (b) triangular (c) pyramidal (d) Any of the above upon the number A is :

23 23

44.

three depending of lone pairs

on

45.

Which of the following molecular orbital has the lowest energy ? (a) 2pz

(b) 2py (c) 2px (d) 2px 46. The enthalpy of H O(l)at 2 be 52, The -394, enthalpy -286 of formation o C H (g), CO (g) and 2 4 2 1 atm kJ mol 1 combustion 25 C and for

pressure respectively.

of C H (g) will be : 2 4 1 (a) 1412 kJ mol (b) 1412 kJ mol

1 (c) + 1412 kJ mol 47. When solutions equal are (d) + 1412 kJ mol volumes mixed, 10 ) 10 10 10 10

1 of the following precipitation will 10 occur only

of

AgCl (Ksp = 18 10 with : 10 + (a) 10 M (Ag ) and 4 + (b) 10 M (Ag ) and 5 + (c) 10 M (Ag ) and 5 + (d) 10 M (Ag ) and

M (Cl ) 4 M (Cl ) 5 ) M (Cl 6 ) M (Cl 2

----------------------- Page 2----------------------Mock 48. The heats Test1 of (Chemistry) of C H , C H 2 and H2 are 14005 kJ, 15583 kJ 2856 kJ. The heat of hydrogenation of ethene is : (a) 2736 kJ (b) 1368 kJ (c) 1368 kJ (d) 1368 kJ 49. In which compound, least ? (a) CsF (c) LiI BF3 is a : (a) Bronsted acid (c) Lewis acid of the 4 2 and 6

combustion

the following binary ratio rcation ranion is (b) LiF (d) CsI (b) Bronsted base (d) Lewis base has

50.

51.

What is the order of reaction which a rate expression 3 2 1 Rate = K [A] [B] ? 1 (a) (b) 3/2 2 (c) Zero (d) None

of

these

52.

For a reversible process at equilibrium, the entropy change can be expressed as : H (a) S = (b) S = T H T qrev (c) S = (d) = Tqrev T

53.

The (a) (b) (c) (d)

rate of a reaction : depends upon temperature is not equal to its molecularity is equal to its molecularity can not be predicted

54.

A compound formed by element A and B crystallises in cubic structure in which A atoms are at the corners of the while B atoms are in the centre of cube. Formula of the compound is (a) A B (b) AB 2 3 2 (c) AB3 (d) AB 109 M HCl is : (b) < 7 (d) 8

cube :

55.

pH of (a) >7 (c) 7

56.

The emf of the cell involving the reaction : + 2Ag (aq) + H (g) 2Ag(s) 2H (aq) 2 is 080V. The potential of silver electrode (a) 0.20 V (c) 0.80 V standard is (b) 0.40 V (d) 0.80 V +

oxidation : 3

----------------------- Page 3----------------------Mock 57. The one Test1 (Chemistry)

PVC in to have 4 osmotic pressure of 60 10 300K. The molecular mass of polymer is : 4 5 (a) 56 10 (b) 16 10 2 2 (c) 30 10 (d) 64 10 Calculate the molality of prepared by dissolving 18 glucose (molecular mass= 180) in 500g of water : (a) 0.2 M (b) 0.1 M (c) 0.4 M (d) 1.2 M What would happen when quantity of H O is added to solution of 2 2 FeSO4 ? a

solution containing 40 g of litre of dioxane was found

atm

at

58.

solution g of

59.

small

2+ (a) An electron is added to Fe 2+ (b) An electron is lost by Fe (c) Colour disappear (d) H is evolved 2 60. Which of following method employed for the purification of colloids? (a) Ultracentrifugation (b) Peptisation (c) Dialysis (d) Electrodialysis 9 If the mass defect of 4X is 009 then the binding energy per nucleon is : (a) 8.38 MeV (b) 838 MeV (c) 9315 MeV (d) 9315 MeV From the following enthalpies combustion, deduce which of the quoted expressions gives the heat of formation of CH at a fixed temperature. 4 C + O (g) CO (g) ; H = c 2 2 H (g) + 1 2 O (g) H O(l) ; H = h 2 2 2 CH (g) + 2O (g) CO (g) + 2H O(l) ; 4 2 2 H = m : (a) c + 2h m (c) c + h m 63. (b) 1 2ch + 1 2m (d) c h + m 2 is not

61.

a.m.u,

62.

of

Copper can be extracted from : (a) kupfer nickel (b) dolomite (c) malachite (d) galena 4

----------------------- Page 4----------------------Mock 64. Test1 (Chemistry)

The phenomenon of syneresis is : (a) migration of colloid in an electric field (b) process of converting gel into a true solution (c) separation of the dispersed from the gel (d) formation of a sol from a gel Which of following members of nitrogen

phase

65.

family does not show negative oxidation state ? (a) Bi (b) Sb (c) As (d) P 66. The freezing o (a) 382 C o (c) 382 C 67. Plaster of Paris is : (a) CaSO H O 4 2 (c) CaSO 2H O 4 68. 2 poing of heavy water is : o (b) 4 C o (d) 0 C (b) 2CaSO 2H O 4

2 2

(d) CaSO 1 2H O 4

Composition of Koppers gas is : (a) CO ( 14 16%) + H2 (3542%) + CO2 (30%) + CH4 ( 10 15%) (b) CO (51 1%) + CO2 ( 126%) + H2 (34%) + N2 ( 19%) + O2 (0 1%) + CH4 (c) CO (46%) + H (36.6%) + CO 2 N 2 (d) none of the above combines ( 14%) + 2 ( 1.1%) + CH (2.4%) 4

69.

Which one of the following with Fe(II) ions to form brown complex ? (a) N O (b) N O 2 3 2 5 (c) N O 2 (d) NO

70.

H S O is : 2 2 8 (a) Marshalls acid (b) a peroxy compound (c) an intermediate in the manufacture of H SO 2 4 (d) (a) and (b) both (b) dark blue (d) colourless 5

71.

Liquid oxygen is : (a) pale yellow (c) pale blue

----------------------- Page 5----------------------Mock 72. Decomposition reaction. Test1 of (Chemistry) H O 2 A 16 volume solution 2 of H O 2 having half life 15 minutes is initially. When will the solution become 1 volume : (a) after 120 minutes (b) after 150 minutes (c) after 60 minutes (d) after 90 minutes 73. Lucas reagent is : (a) Na + H O 2 (b) H SO 2 (c) MnO 2 (d) 74. 4 + H O 2 + HCl present 2 is a first order

ZnCl2 + HCl (conc.) (b) alcohol (d) water

Potassium is kept in : (a) ammonia (c) kerosene The correct formula of Borax is : (a) Na B O 8H O 2 4 7 2 (b) Na [B O 2 (OH) 4 5

75.

] 10H O 4

(c) Na B O 4H O 2 4 7 2 (d) Na [B O (OH) ]8H O 2 4 5 4 76.

Which of the following metal is obtained by the electro-metalluragy ? (a) Ag (b) Na (c) Fe (d) Pb Which of the following ionisation energy ? (a) Cu (b) Au (c) Ag (d) All have equal ionisation energies Chemical reduction is not suitable for : (a) conversion of bauxite into aluminium (b) conversion of cuprite into copper has highest

77.

78.

(c) conversion of haematite into iron (d) conversion of zinc oxide to zinc 79. Toluene on chloride produces (a) benzaldehyde (c) chlorobenzene oxidation : with (b) benzoic acid (d) none of these chromyl

80.

Which of the following is an important constituent of transistors ? (a) Osmium (b) Radium (c) Germanium (d) Silver 6

----------------------- Page 6----------------------CHEMISTRY (Mock Test-1) 41. emistry) (a) NH OH 4 (c) NH3 42. The ratio of between the first to that between atom in H-atom is (a) 4/9 (b) (b) CH F 3 Which of the following the least tendency bonds between molecules ? compounds has to form hydrogen Mock Test1 (Ch

(d) HF the difference in energy and second Bohr orbit second and third Bohr : 1/3 (c) 27/5 (d) 1/2

43.

How many molecules are present in one gram of hydrogen ? 23 (a) 301 10 (b) 15 10 23 (c) 25 10 (d) 602 10 The shape of covalent molecule AX (a) T-shape (b) triangular (c) pyramidal (d) Any of the above upon the number A is :

23 23

44.

three depending of lone pairs

on

45.

Which of the following molecular orbital has the lowest energy ? (a) 2pz

(b) 2py (c) 2px

(d) 2px 46. The enthalpy of H O(l)at 2 be 52, The -394, enthalpy -286 of formation o C H (g), CO (g) and 2 4 2 1 atm kJ mol1 combustion 25 C and for

pressure respectively.

of C H (g) will be : 2 4 1 (a) 1412 kJ mol (b) 1412 kJ mol 1 (c) + 1412 kJ mol (d) + 1412 kJ mol 47. When solutions equal are volumes mixed, 10 ) 10 10 10 10

1 1

of the following precipitation will 10 occur

of

AgCl (Ksp = 18 10 with : 10 + (a) 10 M (Ag ) and 4 + (b) 10 M (Ag ) and 5 + (c) 10 M (Ag ) and 5 + (d) 10 M (Ag ) and

only

M (Cl ) 4 M (Cl ) 5 ) M (Cl 6 ) M (Cl 7

----------------------- Page 7----------------------Mock 48. The heats Test1 of (Chemistry) of C H , C H 2 4 2 and 6

combustion

and H2 are14005 kJ, 15583 kJ 2856 kJ. The heat of hydrogenation of ethene is : (a) 2736 kJ (b) 1368 kJ (c) 1368 kJ (d) 1368 kJ 49. In which compound, least ? (a) CsF (c) LiI BF3 is a : (a) Bronsted acid (c) Lewis acid of the

the following binary ratio rcation ranion is (b) LiF (d) CsI (b) Bronsted base (d) Lewis base which has

50.

51.

What is the order of reaction a rate expression 3 2 1

Rate = K [A] 1 (a) 2 (c) Zero 52.

[B]

? (b) 3/2 of these

(d) None

For a reversible process at equilibrium, the entropy change can be expressed as : H (a) S = (b) S = T H T qrev (c) S = (d) = Tqrev T The (a) (b) (c) (d) rate of a reaction : depends upon temperature is not equal to its molecularity is equal to its molecularity can not be predicted

53.

54.

A compound formed by element A and B crystallises in cubic structure in which A atoms are at the corners of the while B atoms are in the centre of cube. Formula of the compound is (a) A B (b) AB 2 3 2 (c) AB3 (d) AB 109 M HCl is : (b) < 7 (d) 8

cube :

55.

pH of (a) >7 (c) 7

56.

The emf of the cell involving the reaction : + 2Ag (aq) + H (g) 2Ag(s) 2H (aq) 2 is 080V. The potential of silver electrode (a) 0.20 V (c) 0.80 V standard is (b) 0.40 V (d) 0.80 V +

oxidation : 8

----------------------- Page 8----------------------Mock 57. The one Test1 (Chemistry)

PVC in to have 4 osmotic pressure of 60 10 300K. The molecular mass of polymer is : 4 5 (a) 56 10 (b) 16 10 2 2

solution containing 40 g of litre of dioxane was found

atm

at

(c) 30 10 58.

(d) 64 10 solution g of

Calculate the molality of prepared by dissolving 18 glucose (molecular mass= 180) in 500g of water : (a) 0.2 M (b) 0.1 M (c) 0.4 M (d) 1.2 M What would happen when quantity of H O is added to solution of 2 2 FeSO4 (a) An ? 2+ electron is added to Fe 2+ (b) An electron is lost by Fe (c) Colour disappear (d) H is evolved 2 a

59.

small

60.

Which of following method employed for the purification of colloids? (a) Ultracentrifugation (b) Peptisation (c) Dialysis (d) Electrodialysis 9 If the mass defect of 4X is 009 then the binding energy per nucleon is : (a) 8.38 MeV (b) 838 MeV (c) 9315 MeV (d) 9315 MeV From the following enthalpies combustion, deduce which of the quoted expressions gives the heat of formation of CH at a fixed temperature. 4 C + O (g) CO (g) ; H = c 2 2 H (g) + 1 2 O (g) H O(l) ; H = h 2 2 2 CH (g) + 2O (g) CO (g) + 2H O(l) ; 4 2 2 H = m : (a) c + 2h m (c) c + h m 2

is

not

61.

a.m.u,

62.

of

(b) 1 2ch + 1 2m (d) c h + m

63.

Copper can be extracted from : (a) kupfer nickel (b) dolomite (c) malachite (d) galena

9 ----------------------- Page 9----------------------Mock 64. Test1 (Chemistry)

The phenomenon of syneresis is : (a) migration of colloid in an electric field (b) process of converting gel into a true solution (c) separation of the dispersed from the gel (d) formation of a sol from a gel Which of following members of nitrogen family does not show negative oxidation state ? (a) Bi (b) Sb (c) As (d) P The freezing o (a) 382 C o (c) 382 C poing of heavy water is : o (b) 4 C o (d) 0 C (b) 2CaSO 2H O 4

phase

65.

66.

67.

Plaster of Paris is : (a) CaSO H O 4 2 (c) CaSO 2H O 4 2

2 2

(d) CaSO 1 2H O 4

68.

Composition of Koppers gas is : (a) CO (14 16%) + H2 (3542%) + CO2 (30%) + CH4 (10 15%) (b) CO (511%) + CO2 (126%) + H2 (34%) + N2 (19%) + O2 (01%) + CH4 (c) CO (46%) + H (36.6%) + CO 2 N 2 (d) none of the above combines (14%) + 2 (1.1%) + CH (2.4%) 4

69.

Which one of the following with Fe(II) ions to form brown complex ? (a) N O (b) N O 2 3 2 5 (c) N O 2 (d) NO

70.

H S O is : 2 2 8

(a) Marshalls acid (b) a peroxy compound (c) an intermediate in the manufacture of H SO 2 4 (d) 71. (a) and (b) both (b) dark blue (d) colourless 10 ----------------------- Page 10----------------------Mock 72. Decomposition reaction. Test1 of (Chemistry) H O 2 A 16 volume solution 2 of H O 2 having half life 15 minutes is initially. When will the solution become 1 volume : (a) after 120 minutes (b) after 150 minutes (c) after 60 minutes (d) after 90 minutes 73. Lucas reagent is : (a) Na + H O 2 (b) H SO 2 (c) MnO 2 (d) 74. 4 + H O 2 + HCl present 2 is a first order

Liquid oxygen is : (a) pale yellow (c) pale blue

ZnCl2 + HCl (conc.) (b) alcohol (d) water

Potassium is kept in : (a) ammonia (c) kerosene The correct formula of Borax is : (a) Na B O 8H O 2 4 7 2 (b) Na [B O 2 (OH) 4 5

75.

]10H O 4

(c) Na B O 4H O 2 4 7 2 (d) Na [B O (OH) ]8H O 2 4 5 4

76.

Which of the following metal is obtained by the electro-metalluragy ? (a) Ag (b) Na (c) Fe (d) Pb Which of the following ionisation energy ? (a) Cu (b) Au (c) Ag (d) All have equal ionisation energies Chemical reduction is not suitable for : (a) conversion of bauxite into aluminium (b) conversion of cuprite into copper (c) conversion of haematite into iron (d) conversion of zinc oxide to zinc Toluene on chloride produces (a) benzaldehyde (c) chlorobenzene oxidation : with (b) benzoic acid (d) none of these chromyl has highest

77.

78.

79.

80.

Which of the following is an important constituent of transistors ? (a) Osmium (b) Radium (c) Germanium (d) Silver 11

----------------------- Page 11----------------------Answer Sheet (Mock Test-1) (CHEMISTRY) 41. 46. 51. 56. 61. 66. 71. 76. (b) (b) (a) (c) (d) (c) (c) (b) Hints 41. 42. 47. 52. 57. 62. 67. 72. 77. (c) (b) (c) (b) (a) (d) (c) (b) 43. 48. 53. 58. 63. 68. 73. 78. (a) (c) (a) (a) (c) (c) (d) (a) 44. 49. 54. 59. 64. 69. 74. 79. (d) (c) (d) (b) (c) (d) (c) (a) 45. 50. 55. 60. 65. 70. 75. 80. (c) (c) (b) (b) (a) (d) (b) (c)

& Solutions if H is

A hydrogen bond is formed attached to highly electronegative atoms like F, O, N etc. ns (Chemistry) 42. For H-atom 13.6 13.6

Mock Test1 Solutio

E3 = (3) = 1.5 eV 13.6 E2 =

2 =

2 =

13.6 4 13.6 1

(2) = 3.4 eV 13.6 E1 = 2 = (1) = 13.6 eV Now E2 E1 = (3.4) (13.6)

= 13.6 3.4 = 10.2 eV E3 E2 = (1.5) (3.4) = 3.4 1.5 = 1.9 eV E2 E1 E3 E2 5.4 = 5 43. Molecules in 1 gram H2 = of 1 6.02 10 2 23 a 23 = 1.9 27 10.2 = 5.36

= 3.01 10 44.

Shape of the molecule depends upon total number of electron pairs around the central atom.

12 ----------------------- Page 12----------------------Mock 45. Test1 Solutions (Chemistry) is less so 2x + 2H O 2 2 1360 52 than has lowest

Energy of bonding orbitals antibonding orbital energy than other given. C H 2 + 3O 4 2 2CO

46.

= 2 (394) + 2 (286) (52) = 1412 kJ + 47. Ionic product = [Ag ][Cl ] 4 = 10

10

4 = 10

more than its given ionic 10 (1.8 10 ) So precipitation of AgCl 4 + only with 10 M(Ag ) and 10 48. Hhydrogenation = Hcombustion (C H )

It

is

product will occur 4 M(Cl ) 2 4 2 6

+ Hcombustion (H ) Hcombustion (C H ) 2 = (1409.5) + (285.6) (1558.3) = 1695.1 + 1558.3 = 136.8 kJ 49. 50. LiI has the least ratio of rcation ranion According to Lewis, the acids are species which can accept electrons. BF has incomplete octet, hence 3 act as Lewis acid. 51. The order of reaction is the sum of the powers or exponents to which concentration terms are raised in the rate law expression so the order of reaction is 3 = 2 52. + (1) = 2 3 1 = 1/2 those lone pair

of

the this

Entropy of a system is a state function i.e., it depends upon the initial and final states of the system. When the state of a system changes, the entropy also changes qrev The change in entropy (S) = T where qrev = heat supplied reversibly T = temperature on absolute scale. Rate of temperature. a reaction depends upon 13

53.

----------------------- Page 13----------------------Mock 54. Test1 Solutions (Chemistry)

Number of atoms of A per unit cell = 8 (at the corners) 1 = 1

8 Number of atoms of B in one unit cell = 1 (present at the body centre of cube) So, number of atoms of elements and B per unit cell is one each. Hence formula of compound is AB. 55. + [H ] = 10 9 + 10 = 1.01 10 7 (obtained from water) 7 M A

+ pH = log [H ] = log (1.01 10 107) = 7 log 1.01 = 7 0.004 = 6.996 So, its pH is less than 7. 56. E cell or, = E + = E + 2 Ag Ag H H 0.80 = E + E Ag Ag H H + 2 E + Ag Ag = 0.80 V

or, hence, 57.

Eoxid = 0.80 V gram, V = 1000 ml = 1 litre = 6.0 10 T = 300 K w = mV w.RT m = V = 1.6 10 w = 5 1000 V(ml) = 0.2 M . RT 4 0.0821 300 6 104 1 4 atm,

Given, w = 4

58.

Molality = mol. mass 18 1000 = 180 500

59. +2

H O 2

H O + [O] 2 +3

2FeSO 4

+ H SO 2

+ [O] Fe (SO ) 4 is lost by Fe2+. 2 4 3

+ H O 2

An electron 60.

Peptisation is the phenomenon of converting freshly prepared precipitate into colloidal solution by the addition of suitable electrolyte. 14

----------------------- Page 14----------------------Mock 61. Test1 Solutions (Chemistry) Mass defect in amu 931.48 MeV = 0.09 931.48 = 83.83 MeV energy per nucleon binding energy of the nucleus

Binding energy

of nucleus =

Binding =

number of nucleon (mass number) 83.83 = = 9.315 MeV 9 62. H heat of combustion of CH4 = Hcombustion of CO2 + H(combustion of H O) H(CH ) 2 = c + 2h m 63. 64. Malachite is an ore of copper, so copper can be extracted from malachite. Gels on standing shrink and the liquid enclosed is released, the process is called weeping of a gel or syneresis. Syneresis is the process of separation dispersed phase from gel. Bi (bismuth) does not oxidation state because electronegativity than hydrogen. D O 2 (heavy water) is mobile and its show of a 4

of

the

65.

negative very less colourless,

66.

odourless, tasteless freezing point is 3.82C. point is 101.42C. 67. CaSO .1/2H O 4 2

liquid. boiling as plaster

Its

is known

of

Paris. 68. CO(46%) + H (36%) + CO (14%) 2 2

+ N (1.1%) + CH (2.9%) 2 4 is Koppers gas. 2+ 69. Fe (aq ) + NO (g) + 5H O (l) 2 nitric oxide 2+ [Fe(H O) .NO] 2 5 Brown 70. H S O 2 2 is 8 called Marshalls complex acid or

peroxydi-sulphuric acid (due to peroxide linkage). 15 ----------------------- Page 15----------------------Mock 71. Test1 Solutions (Chemistry)

Liquid oxygen is pale blue liquid. It can also be solidified to a bluish white solid at 218C. The 1/ 16 volume in 4 strength of H O 2 half lives as follows. N = N 1n t 0 2 1 = 16 1n 2 1 1n = 16 2 4 n 1 1 = 2 2 Thus Time taken n = 4 = 15 minutes 4 2 becomes

72.

= 60 minutes 73. Lucas reagent is a mixture of and anhydrous ZnCl , which is used for 2 distinction alcohols. 74. between conc HCl

three

types

of

Potassium is kept in kerosene oil because potassium reacts with water, and alcohol. Borax (sodium tetraborate decahydrate) Na B O .10H O. Borax occurs naturally 2 4 7 2 as tincal. Na [B O (OH) ].8H O = Na B O .10H O 2 4 5 4 2 2

ammonia

75.

2 for

76.

Electro-metallurgy chemically metals like Na.

is active and

done electropositive

77.

The ionisation potential of Cu, Ag and Au are 7.7 eV, 7.5 eV and 9.2 eV. So Au (gold) has highest ionisation potential. Bauxite is not chemically reduced to Al, as aluminium is fairly electropositive and reactive metal. It may react with reducing agent. CH Toluene + CrO Cl 3 2 2 chromyl chloride CHO Benzaldehyde Etards reaction

78.

79.

80.

Germanium (semiconductor) important constituent of transistors.

is

an 16

----------------------- Page 16----------------------ENGLISH (Mock Test-1) Directions (for Q. 136 to Q. 138) : Read the passage and answer the following questions. If I had been asked in my early youth whether I preferred to have dealings only with men or only with books, my answer would certainly have been in favour of books. In later years, this has become less and less the case. Not that I have had so much better experiences with men than

with books, on the contrary, delightful book even now come my way more often than purely delightful men. But the many bad experiences with men have nourished the meadow of my life as the noblest book could not do. glish) 136. In his early youth, the author : (a) preferred to have dealing only with books (b) liked to have dealings more with men than with books (c) liked to have more dealings with men than with books (d) liked to have more dealings books than with men The author says thay in later years his love of books diminished because : (a) he had given up the habit of reading books (b) he did not got many delightful books to read (c) he had better experience with than with books (d) even the bad experiences he had with men were more valuable than what the noblest book gave Which of the following statements best reflect the main argument of passage ? (a) neither men nor books give worth-while experience (b) books are always better than men (c) there are purely delightful men than purely delightful books (d) it is the experience with other human beings that nourishes ones life and not the books. Directions (for Q. 139 to Q. 143) : Choose the incorrect word in the given sentences. 139. Shree brought three (a) oranges (c) of dozens of oranges : (b) dozens (d) brought

purely

Mock Test1 (En

with

137.

men

138.

any

17

----------------------- Page 17----------------------Mock 140. Can your aphabets ? (a) Alphabets (c) Recognise Test1 (English) child recognise (b) Your (d) Can English

141.

Our college has good furnitures : (a) good (b) has (c) our (d) furnitures

142.

You must carry your luggages yourself: (a) your (b) carry (c) luggages (d) yourself The magistrate issued orders release of the prisoners : (a) issued (b) orders (c) release (d) prisoners Directions (for Q. 144 to Q. 147) : Choose synonym from the given words from each set. for the

143.

144.

ABSOLUTION : (a) amnesty (b) charge (c) conviction (d) sensual GROUSE : (a) fired (b) brave (c) complain (d) lazy ACUMEN : (a) bluntness (c) drive (b) dullness (d) cleverness

145.

146.

147.

ADHERE : (a) renounce (b) believe (c) disbelieve (d) withdraw Directions (for Q. 148 to Q. 150) : Choose antonym for the given words from each set. SEQUENCE : (a) progression (c) break BENIGN : (a) greedy (c) complimentary ANNESTY : (a) pardon (c) remission (b) chain (d) succession (b) cruel (d) perfect (b) mercy (d) punishment 18

148.

149.

150.

----------------------- Page 18----------------------Answer Sheet (Mock Test-1) (ENGLISH) 136. 141. 146. (d) (d) (d) 137. 142. 147. (d) (c) (b) 138. 143. 148. (d) (b) (c) 139. 144. 149. (b) (a) (b) 140. 145. 150. (a) (c) (d)

Hints 136.

& Solutions his early youth more dealings with Mock Test1 Soluti

According to author in he liked to have books than with men. ons (English) 137. According to that in diminished experiences valuable than

the passage the author says later years his love of because even the he had with men were more what the noblest book gave.

books bad

138.

In view of the given passage, the sentence It is the experience with other human beings that nourishes ones life and not the books is the statement which reflects the main argument of the passage. Dozen is never used Therefore, in place of dozens dozen will come. Option (b) is correct. Alphabet is never used Therefore, in place of alphabets alphabet will come. Option (a) is correct. Furniture is always used Therefore, in place of furniture will come. Option (d) is correct. Luggage is used in singular. Therefore, in place of Luggages luggage will come. Option (c) is correct. Order is used in singular. Therefore, in place of orders order will come. Option (b) is correct. Absolution means pardon and amnesty means general pardon to political offender. Therefore, option (a) is correct. Grouse means complain and complain means to object any one or displeasure. Therefore, option correct synonym. express (c) is 19 in plural.

139.

140.

in

plural.

141.

in singular. furnitures

142.

143.

144.

145.

----------------------- Page 19----------------------Mock 146. Test1 Solutions (English) or mental

Acumen means sharpness acuteness and cleverness also have same meaning. Therefore, option (d) is correct synonym. Adhere means to hold

147.

opinion

and

believe also have same Therefore, option (b) is correct synonym. 148. Sequence means continuity while break means Therefore, option (c) is correct antonym. Benign means kind while cruel means pitiless. Therefore, option (b) antonym. Amnesty means punishment is the Option (d) is correct. pardon. antoym of

meaning. or series dicontinuity.

149.

is

correct

150.

Therefore, amnesty. 20

----------------------- Page 20----------------------MATHEMATICS (Mock Test-1) 1 81. If a 1 + a 1 1 tics) (a) abc (c) 0 82. (b) abc (d) none of these + b 1 + c 1 1 + b 1 1 = 0, then 1 1 1 + c Mock Test1 (Mathema is equal to :

The smallest positive integer n for which 1 + i n = 1 is : 1 i (a) 4 (c) 2 3 6 , then A is : 4 (b) non-singular (d) none of these (b) 3 (d) 1

83.

If A = 2 (a) singular (c) 1 1

84.

1 x dx is equal to : 0 1 + x (a) + 1 2 (c) + 1

(b) 2 (d) 2 1

85.

(x + 1) ex 2 x dx is equal cos (xe ) 1 x (xe ) x

to : x tan (xe )

(a) tan

(b)

(c) 86. The

tan (xe ) a matrix h g h b f

(d) none of these g f is : c

(a) skew symmetric (b) symmetric (c) diagonal matrix (d) scalar matrix 1 87. (cos x), 0 x < 1, is equal to : (a) x (b) x 2 2 (c) x 88. (d) x lines sin

The distance between the 3x + 4y = 9 and 6x + 8y = 15 is : (a) 3 10 (b) 3 2 (c) 6 (d) none of these

21 ----------------------- Page 21----------------------Mock 89. Test1 (Mathematics) then the 4 (b) x + 2 + 2x (d) none of these 1 x) = 2 , where : 1 x value of 2 fof is

2 If f (x) = x + 1, equal to : 4 2 (a) x 2 + 2x 4 2 (c) x + x + 1 d dx (cos (a) (b) (c) (d) 1

90.

1 x 1 1 < x 1 1 < x < 1 1 x < 1 sin [x] for x > 0 [x] + 1

cos 2 [x] 91. If f (x) = [x] k x = 0 for x < 0

where [x] denotes the greatest integer less than or equal to x, then in order that f be continuous at x = 0, the value of k is : (a) equal to 1 (b) equal to 0 (c) equal to 1 (d) indeterminate [x] 92. Lim x 0 (a) 0 (c) 1 93. If f (3 x) = f (x), then 1 to : 1 2 (a) 2 1 f (x) dx 3 2 (c) 2 1 f (x) dx 94. 3 2 (b) 2 1 f (2 x) dx (d) none of these is equal x (b) 1 (d) does not exist 2 xf (x) dx is equal to :

2 The co-efficient of x in the expansion of 2 1 2 (1 + 4x + x ) is equal to : (a) 2 (b) 3 (c) 2 (d) none of these 2 If A = sin + cos values of : 13 (a) A 1 16 (c) 1 A 2 4 , (b) 4 3 (d) 4 22 A 1 then 3 A 16 for 13 all real

95.

----------------------- Page 22----------------------Mock 96. Test1 (Mathematics)

Let A and B be subsets of X. Then : (a) A B = Ac B (b) A B = A Bc

(c) A B = A B (d) A B = A B n 97. lim n (a) 5 (c) e 98. The are : 1 (a) 1, 5 (c) 2, 1 (d) none of these x 2 3 1 0 y 5 9 0 0 z 0 0 0 0 (b) 0, 1, 5 1 solution of the (4 n 1 n + 5 ) is equal to : (b) 4 (d) none of these equation 3 + x 1 = 2

99.

The

value

of

determinant

is equal (a) xyz (c) 0 100.

to

: (b) x + y + z + (d) none of these then

If z is a complex number, |3z 1 | = 3 |z 2 | represents : (a) a circle (b) y-axis (c) x-axis (d) a line parallel to y-axis The an probability announced 1 meeting is . If a student is 5 then the probability that the student miss at least one test is : 9 (a) (b) 25 2 (c) (d) 5 absent twice, will 7 75 4 5 that a teacher will test during give any

101.

class

102.

Two cards are drawn successively with replacement from a pack of 52 cards. The probability of drawing two aces is : 1 4 1 1 (a) (b) 12 51 52 51

1 (c) 13

1 (d) 13

1 13

1 17 23

----------------------- Page 23----------------------Mock 103. Test1 (Mathematics)

3 3 3 3 11 + 12 + 13 + + 20 is : (a) an odd integer but not a multiple of 5 (b) multiple of 10 (c) an odd integer divisible by 5 (d) an even integer There are n different books and m copies of each in a college library. The number of ways in which a selection of one more books is : (a) mnCn nCn (b) (m + 1)n mn ! (c) (m !)m (d) (m + 1) n 1 of 5

104.

or

105.

A G.P. consists of an even number terms. If the sum of all the terms is times the sum of the terms occupying odd places, then the common ratio will be : (a) 2 (b) 3 (c) 4 (d) 5 The arithmetical fraction that exceeds square by the the greatest quantity 1 (a) (b) 2 3 (c) (d) 4 The number of its is : 1 4 none of these

106.

107.

circles 2 2 x + y 4x 10y + 19 = 0 (a) 1 (c) 3 108. The points 60i + 3j , 40i 8j collinear if : (a) a = 40 (c) a = 40 with

common tangents to the 2 2 x + y + 2x + 8y 25 = 0 are : (b) 2 (d) 4 position and ai 52j (b) a = 40 (d) none of these

and

vectors are

109.

The number of ways in which a mixed doubles tennis game be arranged between 10 players consisting of 6 men and 4 women : (a) 90 (b) 48 (c) 12 (d) 180 24

----------------------- Page 24----------------------Mock 110. Test1 (Mathematics) This

Given that (a + b ) is perpendicular b and a is perpendicular to 2b + a . implies : (a) 2a = b (b) a = b (c) a = 2b (d) a = 2b The 6 (a) (c) number of ways in which 6 boys and girls sit alternatively is : 1036800 (b) 508400 518400 (d) none of these

111.

112.

The position vectors of point A and B are ^ ^ ^ ^ ^ ^ i j + 3 k and 3 i + 3 j + 3 k respectively. The equation of a plane is ^ ^ ^ r . (5 i + 2 j 7 k ) + 9 = 0. The points A and B : (a) lie on the plane (b) lie on the same side of the plane (c) lie on the opposite side of the plane (d) none of these Equation and of which the curve passing through satisfied the differential 1 2 is : x 3 (a) 6xy = 3x 2 (b) 6xy = 3x 6x + 29 (c) 6xy = 3x3 29x + 6 (d) none of these equation of the normal point + 29x 6

113.

dy equation dx = x +

114.

The

at the 2 t to the curve x = at , y = 2at is : 3 (a) tx + y = 2at + at (c) tx + y = at3

(b) tx + y = 2at (d) none of these

115.

The equation of the circle which touches

the axes of the x + y = 1 3 4 first-quadrant x2 + y2 2cx (a) 1, 6 (c) 3, 4 116.

co-ordinates and the line and whose centre is 2cy + c2 = 0, where c is : (b) 4, 5 (d) 2, 3

lies

in

the

If a vector r satisfies the r (i + 2j + k ) = i k , to : (a) i + (t + 3) j + k (c) 2i + 7j + 3k

equation then r is equal (b) j + t (i (d) 2i + 3j + 2j + k ) + k 25

----------------------- Page 25----------------------Mock 117. Test1 (Mathematics) 1 where f (x) = x sin (a) g x differentiable , x 0, x = 0, at x = 0 : f g is is not not is while continuous (b) g is differentiable but continuous (c) g is differentiable but g is continuous (d) both f and g are differentiable 118. The magnitude of a radian is : (a) 180 (c) 57 17 44.8 (b) 58 59 (d) 60

g (x) = xf (x)

119.

The vector c , directed along the internal bisector of the angle between the vectors a = 7i 4j 4k and b = 2i j + 2k with |c | = 5 6, is : 5 (a) (5i + 5j + 2k ) 3 5 (b) (i 7j + 2k ) 3 5 (c) (5i + 5j + 2k ) 23 5 (d) (i + 7j + 2k ) 3

120.

If a, b are odd integers, then the roots of the equation 2ax2 + (2a + b) x + b = 0, a 0 is : (a) non-real (c) irrational (b) equal (d) rational are : (b) G.P. (d) none of these floting 0.8642E02 0.2562E02 point in G.P., then

121.

If a > 0, b > 0, c > 0 loga x, logb x, logc x are in (a) A.P. (c) H.P.

122.

In normalized representation gives : (a) 3.3731E04 (b) 0.3373 (c) 3.373E0 (d) none of these

123.

The value of k f (x) = sin x cos x kx + b all real values is given by : (a) k < 2 (c) k 1

in

order decreases

that for

(b) k 2 (d) k < 1 26

----------------------- Page 26----------------------Mock 124. Test1 (Mathematics)

The value of the determinant 1 cos ( ) cos ( ) cos ( ) 1 cos ( ) cos ( ) cos ( ) 1 is :

(a) 2 cos cos cos (b) 4 sin sin sin (c) 4 cos cos cos (d) none of these 125. A problem in statistics is given to students whose chances of solving it are 1 1 1 , and . The probability 2 3 4 problem is solved is : 1 1 3 (a) (b) (c) 3 2 4 three that the

(d)

27 ----------------------- Page 27----------------------Answer Sheet (Mock Test-1) (MATHEMATICS) 81. 86. 91. 96. 101. 106. 111. 116. 121. (b) (b) (b) (b) (a) (a) (a) (b) (c) Hints 82. 87. 92. 97. 102. 107. 112. 117. 122. (c) (b) (d) (a) (c) (b) (c) (c) (c) 83. 88. 93. 98. 103. 108. 113. 118. 123. (a) (a) (c) (a) (c) (b) (a) (c) (b) 84. 89. 94. 99. 104. 109. 114. 119. 124. (d) (b) (d) (a) (d) (d) (a) (b) (d) 85. 90. 95. 100. 105. 110. 115. 120. 125. (b) (c) (d) (d) (c) (d) (a) (d) (c)

& Solutions Mock Test1 Solutio

81. The given determinant ns (Mathematics) 1 + a 1 1 = 1 1 + b 1 1 1 1 + c 1 a + 1 1 a 1 = (abc) 1 b 1 b + 1 1 1 c 1 c 1 c [Taking a, b, c from R , R and 1 = (abc) a + b 1 + c 1 + 1 1 b c 1 b + 1 1 c

a b + 1 R

1 2 respectively]

1 1 1 [Performing R1 R1 taking (1 a + 1 b + 1 c +

1 1 b 1 c + 1 and 0 1 1 1 b 1 c + 1

+ R2 + R3 1) common]

0 1 1 1 = (abc) a + b + c + 1 0 1

[C 1 1 = (abc) a +

(C 1 1 + b c 1

C ) and C 3 + 1 [Expanding by . (1)

(C 2 0 1 1st row] 1 2 1

C )] 3

1 = (abc) a = abc (0 + 1) = abc +

1 + b

1 + 1 c

. 1 28

. . 1 1 1 . + + = 0 is given a b c

----------------------- Page 28----------------------Mock 82. Test1 Solutions (Mathematics)

1 + i n = 1 1 i 1 + i . 1 + i n = 1 1 i 1 + i n 1 + i + i2 + i 2 = 1 1 i n 1 + 2i + i2 2 = 1 1 i 1 + 2i 1 1 (1) 2i n = 1 2 = 1 . . 2 [ . i = 1] in = 1

3 83. We have A = 2 6 4

n = 2

2 . . [ . i = 1]

|A |= (3 4) (6 2)

= 12 12 = 0 Since |A | = 0, then A is singular. 1 84. 1 x dx = 0 1 + x 1 (1 x) = 0 1 x 1 dx = 0 1 x 1 = sin1 x 0 . . . sin 2 Putting 1 x 2x dx = dt So, we get = sin 1 x 1 + 1 1 0 2 dt 0 t x dx = 2 t = 1 0 = 1 = t for x = 1 t = 1 1 = 0 dt for x = 0 0 1 x 2 dx 1 x 1 x = 1 2 1 x 2 dx x 2 dx 1 x . dx 0 1 + x 1 1 x 1 x

2 1 x

0 = sin 1 1 sin1 0 + 1 t 2 1 2 1 1 2 t1 2 + 1 . . . t = 1 2 + 1 0 = ( 2 0) + [t ] 1 = ( 2 0) + (0 1) = ( 2 1). ----------------------- Page 29----------------------Mock 85. Test1 Solutions (Mathematics) 29

(x + 1) ex 2 x dx cos (xe )

Putting xex = t, we get 2 = sec t dt = tan t x = tan (xe ). a 86. Let A = h g a Now A = g h h b f h b f g f c g f c the matrix A is [Calculating d . . . dx 2 tan x = sec x x x (xe + e ) dx = dt (x + 1) ex dx = dt (x + 1) ex 2 x dx = cos (xe )

dt 2 cos t

Transpose A of A by interchanging rows and columns of A] Since A = A , therefore symmetric. 1 87. sin (cos x) = sin1 sin x 2 = x 2

88.

Lines are 3x + 4y = 0 4 Slope and . . . Slope m1 = 3 6x + 8y = 15 8 m2 = 4

...(1)

...(2)

= 6 3 since m = m , the given lines are parallel. 1 2 Let y = 0 in first equation, we get

3x = 9, x = 3 So, let point are first is (3, 0) Required distance = ax + by c a2 + b2 [Length of perpendicular from (3, 0) on second line] 6 (3) + 8 (0) 15 = 36 + 64 18 15 = 10 = 10 30 ----------------------- Page 30----------------------Mock 89. Test1 Solutions (Mathematics) 3

(f 0 f) (x) = f (f (x)) 2 = f (x 2 = (x 4 = x 4 = x + 2x + 2x 2 + 2. + 1) 2 + 1 + 1 + 1) 2 + 1 . . [ . f (x) = x 2 + 1]

90.

Derivative will exist if 2 1 x i.e. i.e. i.e. i.e. > 0 2 1 > x 2 x < 1 |x | < 1 1 < x < 1. lim h 0 sin [x] = lim h 0 = [x] + 1 lim h 0 f (0 + h) = lim h 0 sin [0 + h] [0 + h] + 1 quantity] f (h )

91.

lim f (x) = x 0+

[0 + a = 0 small 0 = 0 + 1 = 0

Lim f (x) = Lim h 0 x 0 = lim h 0 = lim h 0 = lim

f (0 h)

f (h) cos 2 [0 h] [0 h] cos 2 1 1

h 0 = 0 [0 a = 0 small lim f (x) = k = 0 x 0 Since f is continuous at x = 0, lim x 0 92. We have Lim

quantity = 1]

f (x) = lim f (x) = lim f (x) x 0+ x 0 k = 0. [x]

+ x x 0 = Lim 0 + h = Lim |h | Lim h x 0 (0 + h) h 0 h h 0 h 0 h h now, Lim = Lim = 1 x 0 0 h h 0 h . . . L.H.L. R.H.L. [x] Lim does not exist. x x 0

= 1

31 ----------------------- Page 31----------------------Mock 93. Test1 Solutions 2 Let I = 1 Now f (3 x) = f (x) 2 I = 1 2 I = 1 3f (x) dx 1 2 x f (x) dx (3 x) f (x) dx x f (x) dx ...(1) (Mathematics)

2 I = 1 2 94. Using (a + b) (1 + 4x + x ) Binomial 2 n = a + n (b) + 2 ! 2 1 2 = 1 + 2 1 2 (4x + x ) + 1 2 (1 2 1) 2 ! 2 2 (4x + x ) 2 x = 1 + 2x + 2 Co-efficient of x 2 95. A = sin + cos 2 = sin 2 = sin + 1 + sin 2 = 1 sin + sin 2 = 1 sin 2 = 1 sin cos (1 sin 2 2 2 1 sin 4 2 2 ...(1) + (1 sin 4 2 sin 4 2 ) 4 2 ) 2 2 8 2 = 2 1 2 = 2 3 1 (16x 2 + x 4 3 + 8x ) + + n (n 1) b + 2I = 3 1 3 2 I = 2 1 f (x) dx. f (x) dx 3f (x) dx I

Now

= 1 (sin cos ) 1 = 1 (2 sin cos ) 4 1 2 = 1 (sin 2) = 1 4 2 0 sin 2 1 1 sin 1 1 2 2 0 2

4 1 1 4 3

4 1

sin 1 sin 4 1

2 0 2 2 1 2 sin 2 1

1 4 3 4

A 1 [from (1)] 32

----------------------- Page 32----------------------Mock 96. Test1 Solutions (Mathematics) element of A B.

Let p be any Then p (A B) Again, let q c A B . Then, Hence from n (i)

arbitray

p A and p B p A and p Bc p A Bc A B A Bc be any arbitrar element of ...(i)

q A Bc q A and q Bc q A and q B q A B A Bc (A B) and (ii), we A B = A Bc have

...(ii)

97.

lim n

(4

n 1 n + 5 ) n 1 n n 1 n 4 (5 ) 1 + 5

= lim n

n 1 n = lim 5 1 + 4 5 n 1 n

= lim 5 (1 + 0) n 0 = 5 1 = 5 98. We have 3 + x 1 3 + x 1 So, Also 1 x 1 x = 5 i.e. x = 1, 1 5 33 ----------------------- Page 33----------------------Mock Test1 Solutions x 2 99. We have = 3 1 expanding along R we get 1 y 5 0 z 0 0 5 9 z 0 0 (Mathematics) 0 y 0 0 0 0 ...(2) = 2 3 = 5 3 + x 3 + x 1 = 2 3 = 1 x x = 1 1 = 2 ...(1) = +2 = 2 1 = 2

= x

9 z

0 = x . y 0 + 0 0 [expanding along R ] 1 = x . y [z 0] = xyz 100. |3z 1 | = 3 |z 2 | |3 (x + iy) 1 | = 3 |(x + iy) 2 | . . [ . z = x + iy] |(3x 1) + 3iy | = 3 |(x 2) + iy | 2 2 2 (3x 1) + 9i y = 9 [(x 2) 2 2 2 + i y ]

[Squaring both sides] 2 9x 2 9x + 1 6x 9y 2 = 9 [x 2 + 4 4x y ] 2 2

2 2 + 1 6x 9y = 9x + 36 36x 9y 1 6x = 36 36x 30x = 35 7 x = 6 i.e. a line parallel to y-axis. 101. The probability that one test is held 1 4 8 = 2 = 5 5 25 Probability that test is held on both days 1 1 1 = = 5 5 25 Thus, probability that the student misses 8 1 9 at least one test = + = . 25 25 25 102. Let S denote the event of getting an ace i th in the i draw. Probability of getting aces in both the draws = P (S S ) = P (S ) P (S ) 1 2 1 [Multiplication

2 theorem]

4 = 52 1 = 13

4 52 1 13 34

----------------------- Page 34----------------------Mock 3 103. 11 Test1 Solutions (Mathematics) 3 + 20 3 (1 3 + 2 3 + + 10 ) 2 3

3 3 + 12 + 13 + 3 3 = (1 + 2 + + 20 ) 2 20 (20 + 1) = 2 . . 3 . 1 + 2 . . . n 2 = [10 (21)] = (10 21) = 44100 3025 2

10 (10 + 1) 2 3 3 n (n + 1) 2 + + n = 2 3 = ( n) 2 3 n (n + 1) 2 = 2

[5 (11) ] (5 11) 2

= 41075 = an odd integer divisible by 104. In 1, (m

5.

the case of each book we may take 0, 2, 3, ... m copies. We may deal with each book in + 1) ways and therefore with all the n books in (m + 1) ways. But this includes the case where all books are rejected and no selection is made. So, the number of ways in which selection can be made = (m + 1)n 1 Let 2n be the total where number of terms

105.

S2n = 5 [T1 + T3 + + T2n 1] terms 2n 2 ]

T , T T are 1 3 2n 1 occupying odd places a [1 r2n] 2 = 5 [a + ar + + ar

1 r [Sum of G.P., where a is first term and r is common ratio] 2n a [1 r ] 2 2n 2 = 5a [1 + r + + r ] 1 r a [1 r2n] 5a [1 r2n] = 1 r 1 r2 5 106. Let x Let 1 = 1 + r r = 4 be fraction f (x) = x x2 f (x) = 1 2x and minima, put f (x) = 0 1 1 2x = 0 x = 2 f (x) = 2 < 0 2 So, f (x) = x x is maximum at x = 1 . 2 35 ----------------------- Page 35----------------------Mock 107. Test1 Solutions (Mathematics) ...(1) 1 + r = 5

For maxima

The equation of circle is x2 + y2 + 2gx + 2fy + c = 0 comparing the equation 2 2 x + y + 2x + 8y 25 = 0 with (1), we g = 1, f = 4 Centre of first circle C1 ( 1, 4) also radius = g2 + f 2 c

get

= 1 + 16 + 25 = 42 comparing the equation 2 2 x + y 4x 10y + 19 = 0 with (1), we get g = 2, f = 5 Centre of second circle C2 (2, 5) also radius = 4 + 25 19 = 10 2 2 now C C = (2 + 1) + (5 + 4) 1 2 = 32 + 92 = 9 + 81 = 310 Also r1 + r2 = 42 + 10 ...(2)

r2 > 40 + 10 r2 > 210 + 10 r2 > 310 r > C C 2 1 2 C C < r + r 1 2 1 2 Circles do not touch each other. No. of common tangents = 2. 108. Let A, B, C be the three given points. The points are collinear if AB + kAC for some k where A = 60i + 3j B = 40i 8j C = ai 52j (40i 8j ) (60i + 3j ) = k [(ai 52 j ) (60i + 3 j )] 20i 11j = k [(a 60) i 55j ] and 20 = k (a 60) 11 = 55k 1 k = 5 1 (a 60) 5 100 = a 60 a = 60 100 = 40 20 = [Putting value of k]

r1 r1 r1 r 1

+ + + +

36 ----------------------- Page 36----------------------Mock 109. Test1 Solutions (Mathematics) 2 men and 2

Mixed doubles includes women. Since 2 men are selected of 6 men number of ways = 6C2

Also 2 women are selected of 4 women number of ways = 4C2 But they ways. Total also can be interchanged 6 no. of ways = C2 in 4 2 C2 2

6 5 1 2

4 3 1 2

= 180 110. Since (a + b ) is perpendicular to b (a + b ) . b = 0 a . b + b . b = 0 a . b = . b ...(1)

Also (2b + a ) is perpendicular to a 111. a . (2b + a ) = 0 2a b + a . a = 0 2 ( . b ) + a . a = 0 2 a 2 a 2b 2 = 2b a = 2 b. 2 = 0

[From (1)]

The number of ways of selecting boys = 6 ! The number of ways of selecting 6 girls = 6 ! Since girls and boys sit alternatively. Required number of ways = 2 6 ! 6 ! = 2 720 720 = 1036800

37 ----------------------- Page 37----------------------Mock 112. Test1 Solutions (Mathematics)

The position vectors of two given points are ^ ^ ^ ^ ^ a = i j + 3 k and b = 3 i + 3 j + 3 k and the equation of the given plane is

^ ^ ^ r . (5 i + 2 j 7 k ) + 9 = 0 or We have ^ ^ ^ ^ ^ ^ a . n + d = ( i j + 3 k ) . (5 i + 2 j 7 k ) + 9 = 5 2 21 + 9 < 0 ^ ^ and b . n + d = (3 i + 3 j + 3 k ) ^ r . n + d = 0

^ ^ ^ . (5 i + 2 j 7 k ) + 9 = 15 + 6 21 + 9 > 0 So, the points a and b are on the opposite sides of the plane. dy 113. dx Integrating = x + 2 x both sides, we 2 x y = 2 This equation x 1 + c ...(1) get 1

passes through (3, 9) 2 3 1 = 9 + c 2 3 9 1 9 = + c 2 3 9 1 c = 9 + 2 3 29 c = 6 of c in (1), we 2 x 1 29 y = + 2 x 6 3 3x 6 + 29x get

Putting value

y = 6x 3 6xy = 3x 6 + 29x. 38 ----------------------- Page 38----------------------Mock 114. Test1 Solutions (Mathematics)

2 x = at , y = 2at dx dy = 2at, = 2a xt dt dy dy dt = dx dt dx = 2a 1 = t 1 Slope of tangent = t Also slope of normal 1 = slope of tangent = t equation of normal is y y = slope of normal (x x ) 1 2 y 2at = t (x at ) 3 tx + y = 2at + at . 1 1 2at

115.

The equation of co-ordinate axes is 2 x + y 2

the

circle

touching 2

the

2cx 2cy + c 2 = c 2

= 0 ...(1) ...(2)

2 i.e. (x c) + (y c) This touches the line x + y = 1 3 4 4x + 3y = 1 12

i.e.

i.e. 4x + 3y 12 = 0 Perpendicular distance (c, c) and line is

between

...(3) circle

ax + by c x2 + x2 1 2 i.e.

= c = c = c 5

4c + 3c 12 42 + 32 |7c 12 |

Hence 7c 12 = 5c or 5c 7c 12 = 5c or 5c c = 6 or c = 1 c = (1, 6).

39 ----------------------- Page 39----------------------Mock 116. Test1 Solutions (Mathematics)

Let r = (xi + yj + zk )

r (i + 2j + k ) = i k

(xi + yj + zk ) (i + 2j + k ) = i k

2xk xj yk + yi + 2j 2zi = i k

[Doing cross product] (y 2z) i + (z x) j + (2x y) k = i k Comparing the both sides co-efficients y 2z = 1 z x = 0 2x y = 1 x = z = t, where t is a scalar y 2z = 1 y = 1 + 2t r = xi + yj + zk of i, j, k on ...(i) ...(ii) ...(iii)

= t i + (1 + 2t) j + t k

= j + t (i + 2j + k ). 1 117. g (x) = xf (x), where f (x) = x sin x at x = 0 g (0) = 0 f (0) = 0 (0) = 0 g (x) g (0) g (0) = Lim x x 0 xf (x) 0 = Lim x 0 = Lim x 0 x f (x) = 0

also g (x) = xf (x)+ f (x) Lim g (x) = Lim [xf (x) + f (x)] x 0 x 0 = 0 + f (0) = 0 + 0 = g (0) Lim g (x) = g (0) x 0 Hence g is continuous, and differentiable. 180 118. 1 radian = degree = = 57 17 44.8 3.14 180

g (x)

is

40 ----------------------- Page 40----------------------Mock 119. Test1 Solutions (Mathematics) a b |b |

The required vector c = a |a | 1 = b |b | (7i 4j 4k ) +

|a | Now =

7i 4j 4k 49 + 16 + 16

9 2i j + 2k 1 = 4 + 1 + 4 = 3 (2i j + 2k )

1 c = 9 1 c = 9 2 |c | i 9 7 (7i 4j 4k ) + 3

1 (2i j + 2k

2 k 9 2 4 54 = 81 81 ...(1)

j +

2 1 49 = + 81 81 (5 6)2 = 54 2 81 54 2 25 6 = 81 2 25 6 81 = 54 2 = 225

= 15 15 c = (i 7j + 2k ) 9 5 = (i 7j + 2k ). 3

[From (1)]

120.

Equation is 2ax2 + (2a + b ) x + b = 0 Discriminant = (2a + b)2 4 . 2a . b 2 Roots are = (2a + b) > 0 rational. in G.P. b2 = ac logx b2 = logx ac 2 logx b = logx a + logx c 2 logb x = loga x 1 + logc x H.P. 1 8ab

121.

Since a, b, c are

loga x, logb x, logc x are in 122. 0.8642 E02 0.2562 E02 = 0.8642 10 = 86.42 25.62 = 3.3731 E0. 2

0.2562 10

41

----------------------- Page 41----------------------Mock 123. Test1 Solutions (Mathematics)

f (x) = sin x cos x kx + b f (x) = cos x + sin x k 1 1 = 2 2 cos x + 2 sin x k [multiplying = 2 sin cos x + cos 4 and dividing by 2] sin x k 4

1 . . . sin 4 = cos 4 = 2 = 2 sin + x k 4 If f (x) decreases for all x i.e. f (x) is negative k max of 2 sin + x 4

i.e. 124.

k 2. 1 cos ( ) 1 cos ( ) cos ( ) 1

cos ( )

cos ( ) cos ( ) cos2 + sin2 = cos cos + sin sin cos cos + sin sin cos cos + sin sin cos2 + sin2 cos cos + sin sin

cos cos + sin sin cos sin + sin sin 2 2 cos + sin

cos = cos cos = 0 0 = 0.

sin sin sin

0 0 0

cos cos cos

sin sin sin

0 0 0 42

----------------------- Page 42----------------------Mock 125. Test1 Solutions 3 (Mathematics) be the respective events of

Let E , E , E 1 2

___ ___ ___ solving the problem and E , E , E be the 1 2 3 respective problem. Then 1 P (E ) = 1 2 ___ P (E ) = 1 1 ___ P (E ) = 1 2 ___ P (E ) = 1 3 , P (E ) = 2 3 1 = 2 1 = 3 1 = 3 3 2 2 1 events of 1 , P (E ) = 3 4 not solving 1 the

4 4 P (none solves the problem) = P [(not E ) and (not 1 ___ ___ = P (E E E 1 2 ___ ___ = P (E ) . P (E ) . P 1 2 ___ . . [ . E , 1 1 = E ) and (not E )] 2 ___ ) 3 ___ (E ) 3 ___ ___ E , E are independent] 2 3 3

2 3 4 4 Hence P (the problem will be = 1 P

3 =

1 solved)

(none solves the problem)

1 = 1 4 =

3 4 43

----------------------- Page 43----------------------PHYSICS (Mock Test-1) 1. In Youngs experiment, light of wavelength 6000 is used to produce fringes of width 0.8 mm at a distance of 2.5m. If the whole apparatus is dipped in a liquid of refractive index 1.6, the fringe width will be : Mock Test1 (P (a) 0.2 mm (c) 0.6 mm 2. (b) 0.4 mm (d) 0.5 mm

hysics)

In the given circuit, if point C is connected to the earth and a potential of + 2000 V is given to the point A, the potential at B is : (a) 400V (b) 500V (c) 800V (d) 1000V A double convex thin lens made of glass (refractive index = 1.5) has both radii of curvature of magnitude 20cm. Incident light rays parallel to the axis of lens will converge at a distance d cm such that : (a) d = 20 (b) d = 40 20 (c) d = 10 (d) d = 3 Radius of curved road on highway is R and width of the road is b. The outer edge of the road is raised the height of h with respect to inner edge so that a car with velocity v can pass safe over it. The value of h is : 2 2 v R v b (a) (b) g R 2 v v b (c) (d) Rgb Rg The magnetic flux 2 closed loop is = 6t + 7t + 1, where is in milliweber and t in seconds. What will be the value of induced emf after 2 second ? (a) 29 mV (b) 60 mV associated national by

3.

4.

5.

with

(c) 31 mV (d) 22 mV 44 ----------------------- Page 44----------------------Mock 6. Test1 (Physics)

The speed of an electromagnetic wave in vacuum is : 1 (a) c = 0 0 0 (c) c = 0 (d) c = 0 through it in (b) c = 0 0 0

7.

A 60 watt bulb carries a current 0.5 amp. The total charge passing 1 hour is : (a) 1800 coulomb (b) 2400 coulomb (c) 3000 coulomb (d) 3600 coulomb The direction of current in an iron-copper thermocouple is : (a) from copper to iron at cold junction (b) from iron to copper at hot junction (c) from copper to iron at hot junction (d) no current will flow

8.

9.

Two identical cells sends the same current in 2 resistance, whether connected in series or in parallel. The resistance of the cell should be : 1 (a) 2.5 (b) 2 (c) 2 (d) 1 A wire of resistance R is divided equal parts. These parts are connected in parallel, the equivalent resistance of such connection will be : (a) 0.01R (b) 0.1R (c) 100R (d) 10R in 10

internal

10.

11.

The work done in carrying a charge of 50C from a point A to a point B in an electric field is 10 mJ. The potential difference (VB VA) is : (a) 200V (b) + 200V (c) 2kV (d) + 2kV The capacity of the capacitors are shown in adjoining figure. The equivalent

12.

capacitance B and the will be :

charge

between on the

the 6 F

points capacitor

and

(a) 15 F, 90 C (c) 15 F, 270 C

(b) 6 F, 180 C (d) 27 F, 540 C

45

----------------------- Page 45----------------------Mock 13. Test1 (Physics)

Four capacitors are connected as shown in the adjoining fig. The potential difference between A and B is 1500 volt. The energy stored in 2 F capacitor will be : (a) 50.6 J (c) 0.506 J (b) 0.81 (d) none J

14.

An electron and a proton are at a distance of 1 . The moment of this dipole in C-m will be : (a) 3.2 1029 (b) 3.2 1019 (c) 1.6 1029 (d) 1.6 10 19 electrons

15.

The force between two separated by a distance r varies as : (a) r (b) r2 (c) r2 (d) r1

16.

At room temperature, the forbidden energy gap of germanium crystal about : (a) 0.072 eV (b) 0.75 eV (c) 0.105 eV (d) 1.05 eV The activity of a certain element drops to 1/64 of its initial value in 30 second. Its half life is : (a) 6 sec (b) 5 sec (c) 4 sec (d) 2 sec The wavelength of the K characteristic Xry spectra varies with atomic number Z as approximately : 1 1 (a) (b) Z Z2 (c) Z (d) Z radioactive

is

17.

18.

line

of

19.

What wavelength is corresponding to a beam of electrons whose kinetic energy is 100 eV ? 34 (h = 6.6 10 Js, 1 eV = 1.6 10 3 m = 9.1 10 kg)

19 J,

(a) 2.4 (c) 3.6

(b) 1.2 (d) 4.8 46

----------------------- Page 46----------------------Mock 20. Test1 (Physics) for emission sodium is

Threshold wavelength 7 5 10 m. Photoelectric for light of : (a) any wavelength (b) wavelength above (c) wavelength (d) all below 6 10 5 10

occurs 7 m 7 m 14 Hz for a focal

frequency below 5 10

21.

If the distance of the far point myopia patient is doubled , the length of the lens required to cure it will become : (a) double (b) half (c) three times (d) the same Focal length maximum for : (a) red light (c) green light If yellow light in the experiment is replaced the fringe width will (a) increase (b) decrease (c) remain unaffected (d) first increase At a transverse maximum appearance of a convex lens

22.

will

be

(b) yellow light (d) blue light Youngs double slit by red light, then :

23.

and then

decrease

24.

certain wave of

(a) sinusoidal (b) sinusoidal (c) sinusoidal (d) straight 25.

instant, a stationary is found to have kinetic energy. The string at that instant is : A shape with amplitude 3 A shape with amplitude 2 shape with amplitude A line

In Youngs double slit experiment, if the square amplitudes of two superposing waves are in the ratio 1 : 9, then the ratio of the intensity at minima to that maxima will be : (a) 1 : 3 (b) 1 : 4

at

(c) 1 : 9 26. The equation of string can be written as

(d) 1 : 1 wave travelling in a

y = 3 cos ( 100t x) Its wavelength (a) 2 cm (c) 100 cm is : (b) 5 cm (d) none of these 47 ----------------------- Page 47----------------------Mock 27. Test1 (Physics)

Intensity at any point due to interference of two waves will be maximum, when path difference at that point is : (a) (2n + 1) (b) 2n 2 (c) n (d) 2 The displacement y executing periodical motion is given by 2 y = 4 cos (t 2) sin (1000t) This expression may be considered to be a result of the superposition of how many independent harmonic motions : (a) 5 (b) 4 (c) 3 (d) 2 of a particle

28.

29.

One end of a copper rod of length 1.0 m and area of cross-section

103 m2

is

immersed in boiling water and the other end in ice. If the coefficient of thermal conductivity of copper is 92 cal/m-s C and the latent heat of ice 4 8 10 cal kg, then the amount of which will melt in one minute, is : (a) 8 103k (b) 9.2 103k (c) 5.4 103k 30. (d) 6.9 103k

is ice

Two smples A and B of a gas initially at the same temperature are compressed from volume V to V 2. (A isothermally and B adiabatically). The final pressure of A is : (a) twice the final pressure of B (b) less than the final pressure of B (c) equal to the final pressure of B (d) greater than the final pressure of B

31.

Alcohal is more volatile because : (a) its boiling point is lower than water (b) it is an organic compound (c) its freezing point is lower than water (d) its vapour pressure is 2.5 times greater than water

than

water

32.

The respective speeds of the molecules are 1, 2, 3, 4 and 5 km/sec. The ratio of their rms velocity and the velocity will be : (a) 1 : 3 (b) 3 : 4 (c) 3 : 11 (d) 11 : 3 If the double number than of that hydrogen molecules of O , 2 and that of then of

average

33.

H2 ratio

are of

kinetic energy of oxygen at 300K, is : (a) 1 : 16 (c) 1 : 1

(b) 2 : 1 (d) 1 : 2

48

----------------------- Page 48----------------------Mock 34. Test1 (Physics)

If the surface tension of a liquid is T, the gain in surface energy for an increase in liquid surface by A, is : 2 2 2 (a) A T (b) A T 1 (c) AT (d) AT The reason regular body (a) metallic (c) shearing for the is : strain strain change in shape of a strain

35.

(b) longitudinal (d) volume stress

36.

The dimensions of four wires of the same material are given below. In which wire, the increase in length will be maximum : (a) length 50 cm, diameter 0.5 mm (b) length 300 cm, diameter 3 mm (c) length 200 cm, diameter 2 mm (d) length 100 cm, diameter 1 mm What is the weight of a body at a distance 2r from the centre of the earth, gravitational potential energy of the body at a distance r from the centre of the earth is U ? U U (a) (b) 2r 3r U (c) (d) Ur if the

37.

4r 38. Two particles of masses m and m initially 1 2 The

at rest start moving towards each other under mutual force of attraction. speed of the centre of mass at any time t when they are at a distance r apart, is : m m 1 1 2 (a) G r2 m1 + m2 t m m 1 1 2 (b) G r2 m2 t m m 1 1 2 (c) G r2 m1 t (d) Zero 39. Moment of inertia angular acceleration equal to : (a) work done (b) torque (c) angular momentum (d) force If the K.E. of a body is increased by 300%, its momentum will increase by : (a) 175% (b) 300% (c) 150% (d) 100%

40.

49 ----------------------- Page 49----------------------Answer Sheet (Mock Test-1) (PHYSICS) 1. 6. 11. 16. 21. 26. 31. 36. (d) (b) (b) (b) (a) (a) (a) (a) Hints 2. 7. 12. 17. 22. 27. 32. 37. (c) (a) (b) (b) (a) (c) (d) (c) 3. 8. 13. 18. 23. 28. 33. 38. (a) (c) (b) (b) (a) (c) (c) (d) 4. 9. 14. 19. 24. 29. 34. 39. (d) (c) (c) (b) (d) (d) (c) (b) 5. 10. 15. 20. 25. 30. 35. 40. (c) (a) (c) (c) (b) (b) (c) (d)

& SolutionsMock Test1 Solutions (Physics)

1.

The fringe width in

air is given by D = (i)

2d where D = distance of screen from slits = 2.5 m = wavelength of light used = 6000 2d = separation between slits fringe width = 0.8 mm When the whole apparatus is dipped in water, then = = refractive index of = (ii) medium = 1.6 D 2d D or or = 2d = 0.8 Hence, 2. = 1.6 Equivalent capacitance of 2 F and 3 F capacitors which are connected in series = 1.2 F 2 + 3 5 capacitor, charge will be same q = CV = 1.2 10 = 2.4 10 q Now, VA VB = C1 = 1.2 10 But VA = 2000 V VB = 2000 1200 = 800 V 50 = 6 3 C 2.4 10 2 10 3 = 1200 V 3 6 C = 2 3 6 = = 0.5 mm [from (i)] [from (ii)]

So, that

In

each

2000

----------------------- Page 50----------------------Mock 3. Test1 Solutions (Physics) 1 20

From lens makers formula 1 1 1 f = ( 1) R1 R2 Here : = 1.5, R1 = 20 cm, R2 = 20 cm 1 1 So, = (1.5 1) + f 20 1 2 or = 0.5 f 20 1 1 or = f 20 f = 20 cm The incident light rays parallel to the axis of lens will converge at the focus. Hence, d = f = 20 or d = 20 cm Let angle of inclination is . h tan = b For turning of a car without skidding 2 v tan = Rg From equations (i) and (ii), we get 2 h = b Rg 2 v b Hence, h = Rg v

4.

(i)

(ii)

5.

2 Given: = 6t + 7t + 1 where is in millimeter The induced emf is given by d e = dt d 2 or e = (6t + 7t + 1) dt d 2 d

or

e = dt = = t = e t = =

(6t ) + dt 12t + 7 + 0 12t + 7 2 sec 2 = 12 2 + 7 24 + 7 = 31 mV

(7t) + dt

(1)

Putting

6.

In vacuum, the speed of electromagnetic wave is 1 c = 0 0 51

----------------------- Page 51----------------------Mock 7. Test1 Solutions (Physics)

The total charge passing through bulb is q = it Here, i = 0.5 A, t = 1 hour t = 1 60 60 sec t = 3600 sec from eq. (i) q = 0.5 3600 = 1800 coulomb ...(1)

8.

In an iron-copper thermocouple, current flows from copper to iron at the hot junction. Suppose the two identical internal resistance equal to r. When they are connected in series, current 2E 2E I1 = = R + 2r 2 + 2r cells the (i) . . [ . R = 2 ] Again when they are connected parallel, the current E 2E 2E I2 = = = R + r 2R + r 4 + r 2 According to the question I1 = I2 2E 2 + 2r = 4 + r 2 + 2r = 4 + r 2E

the

9.

have

in (ii)

10. When a in 10

2r r = 4 2 r = 2 will

wire of resistance R is divided equal parts, then each wire R have a resistance of . 10 In parallel combination, their equivalent resistance is : 1 10 10 10 = + + + 10 times R R R R 1 R R = 100 = R R R = 0.01 R charge of 100

11.

The work done in carrying a 50 C from A to B is given by W = charge potential difference W = q (VB VA) Here : W = 10 mJ = 10 10 q = 50 C = 50 10 W (VB VA) = q = 2 10 50 10 6

3 J = 10 6 C

2 J

4 = 10 50 = 200 V 52

----------------------- Page 52----------------------Mock 12. Test1 Solutions (Physics) of C2

The given circuit can The equivalent and C3 is

be modified as capacitance

C = C2 + C3 = 6 + 12 = 18 F The equivalent capacitance of C and C 1 i.e. CAB = C1 C C1 + C Since, C1 CAB = 9 18

9 + 18 CAB = 6 F and C are in series, therefore,

charge on C1 and C will be same i.e. q = CAB V

= 6 90 = 540 C Now the potential on 6 F and 12 F are same, q 540 So, V = = = 30 V C 18 Hence, charge on 6 F capacitor is q6 = 30 6 = 180 C 13. Since, 2 F and 3 F capacitors parallel. So, their effective capacitance is C = 2 + 3 = 5 F Now 12 F, 20 F and C are in series. Therefore, their effective capacitance is 1 1 1 1 = + + C 12 20 5 5 + 3 + 12 20 = = 60 60 60 Hence, C = = 3 F 20 q Now VAB = C The charge at C, 20 F and capacitors is equal q 1500 = 3 or q = 4500 C Hence, potential difference across capacitor q 4500 V2 = = = 900 V C 5 So, energy stored in 2 F capacitor will be 1 2 1 6 U = CV = 2 10 900 900 2 2 2 = 81 10 = 0.81 J ----------------------- Page 53----------------------Mock 14. Test1 Solutions (Physics) are in

12 F

2 F

53

Since, the charge on an electron and a proton is equal and opposite, therefore, they will constitutes a electric dipole. dipole moment is given by p = 2ql

Here :

p = q 2l 19 q = 1.6 10 C 2l = 1 = 1 1010 m p = 1.6 10 = 1.6 10 19 29

...(1)

from eq. (1)

1 10

10

15.

According to coulombs law, between two electrons is given by 1 q q 1 2 F = 40 r2 1 or F 2 r 2 or F r eV

the

force

16.

The forbidden energy gap of germanium crystal is about 0.75 tempearature. Using the relation N = N0 N Here So, : = N0 1

at

room

17.

1t T1 2 2

and t = 30 sec 64 1 130 T1 2 = 64 2 16 = 2 6 = T1 2 T1 2 = 5 sec 130 T 1 2 2 30

or

or Hence, 18.

According to Mosleys law = a(Z b )2 2 or and Therefore, we get 1 Z Z 1 from equations 2 or 1 (i) and (ii) (ii), ...(i)

Z2 54

----------------------- Page 54----------------------Mock 19. Test1 Solutions for (Physics) momentum of an (i) by h = mv h So, = 2mE where h = Plancks constant = 6.6 10 E = kinetic energy of electrons = 100 eV = 100 1.6 10 34 J.s 19 V 31 kg 34 19 (ii) p

Using the electron. de-Broglie

relation

p = 2mE wavelength is given h =

m = mass of an electron = 9.1 10 Therefore, from eq. (ii) 6.6 10

31 2 9.1 10 100 1.6 10 = 1.22 10 = 1.22 1.2 20. 10 m

For photoelectric emission, it is necessary that the wavelength of the incident light is less than threshold value. i.e., Hence, the < 0 photo emission occurs 7 m. lens of for

light of wavelength below 5 10 21. For curing of myopia, concave focal length equal to the distance of eyes own far point is used. i.e. f = d Hence, if distance d is doubled, then focal length will also doubled.

22.

From lens makers formula 1 1 f = ( 1) R1 R2 1 f ( 1) 1 f 1 and

(i)

(ii)

So, f But wavelength of red light is more as compared to other given light. focal length of convex lens is maximum for red light.

Hence,

55 ----------------------- Page 55----------------------Mock 23. Test1 Solutions (Physics)

The fringe width is given by D = But 2d rd > yo

Hence, fringe width increases. 24. In a transverse stationary string, when the particle mean position, their maximum. Since, all particles pass mean positions simultaneously, the string becomes straight when the kinetic energy is maximum. Using the relation I a2 2 I max So, Imin 2 : a : a 1 2 = 1 : 3 2 = (a a )2 1 2 (a + a ) 1 2 ...(i) wave passes velocity their on a their is

25.

Given

= 1 : 9 a : a 2 1 Let a1 = x, a2 = 3x Therefore, from equation (i), we get Imax (x + 3x)2

= Imin = 4 Imin Hence, Imax 26. = 4 1 (x 3x)2 16 = 1 4

Comparing the given equation y = 3 cos (100t x) with standard equation y = a cos (t kx) We have, 2 k = or But k = 2 = = 2 cm 56

----------------------- Page 56----------------------Mock 27. We know Test1 the I (a 1 Solutions relation 2 + a 2 2 + 2a a cos ) 1 2 (Physics)

For intentisity to be maximum cos = + 1 or where Path difference = 2 or = 2 or = 2 Hence, = n 28. = 2n n = 0, 1, 2, ........

phase difference 2n

2 t Given : y = 4 cos 2 sin (1000t)

t = 2 cos = 2 cos t 2cos

t sin (1000t)

2 t 2 (i)

2 2 sin 1000t + t 2

sin 1000t

. . [ . cos A sin B = sin (A + B) sin (B A)] From equation (i), it is clear that it is superposition of three independent harmonic motions. 29. Heat taken by ice per sec m 8 10 60 Heat conducted through KA ( Q2 = l 92 10 3 (100 0) 1 According to the law of calorimetry heat taken = heat given m 8 10 60 92 10 3 4 = 92 10 3 (100 0) 1 ) 1 2 rod per sec 4

Q1 = mL =

m =

100 60
4

8 10 = 6.9 103 kg

57 ----------------------- Page 57----------------------Mock 30. Test1 Solutions is have P V 1 (Physics) isothermally,

Since, A therefore, we

compressed = P V 2

V P V = P 1 2 2 P2 = 2P1 Again since, B adiabatically, therefore, P V = P V 1 1 2 Since, So Hence, final pressure final pressure of B. 31. is 2 compressed

P V = P V 1 2 2 P = 2 P 2 1 > 1 P > P 2 P 2

< P 2 of A is less than

At boiling point, all the substances are converted into vapour. Because point of alcohal is lower than point of water, so it will vaporise earlier as compared to water, i.e., alcohal is more volatile than water. The rms velocity 2 1 vrms = 5 55 = = 11 km sec 5 Now the average velocity of molecules is 1 + 2 + 3 + 4 + 5 v average 15 = 5 Hence v rms : v average that the E T kinetic energy = 11 : 3 = 3 km sec = 5 + 2 of the molecules is 2 + 3 2 + 4 2 + 5 2

boiling boiling

32.

33.

We know Therefore,

does

not

depend upon the molecules of the gas. Hence, the kinetic energies of both gases at a given temperature are same. or EH2 : EO2 = 1 : 1

the

58 ----------------------- Page 58----------------------Mock 34. Test1 Solutions (Physics) done in

From the relation, the work increasing the surface area of a liquid by A is given by Here : So, gain W = TA A = A (at a certain temperature) in surface energy = work done W = TA = AT

35. 36.

A body undergoes without change in volume. From the law of Youngs modulus FL Y = Al

change

in

shape

where F = force acting on the wire L = initial length of the wire l = extention in the length of wire A = area of the cross hyphen section wire FL l = YA L l A

of

(i) F Y = constant

From the above relation (i) we extension is more for option (a). 37. Using Weight the relation GM w = mg = m R GMm and U = r Ur 2

sec

that

(i)

or So,

GM = m Ur w = m m Ur = m m Ur = m m U 2 1

(ii)

1 R2

2r
1 4r2

Hence, 38.

w = 4r

The centre of mass will not move, because there is no external force. There is a force of mutual attraction under which centre of mass will not shift. Hence, speed of centre of mass is zero. 59

----------------------- Page 59----------------------Mock 39. Test1 Solutions (Physics)

Using the relation = I Torque = moment of inertia

angular acceleration
40. Let the initial kinetic energy is 100%. Final kinetic energy = (300 + 100)% = 400% (K.E.)final 400 4 So, = = (K.E.)initial 100 1 From the relation, we have 2 p E = 2m 2 or or pfinal pinitial = 4 1 = (K.E.)initial p E p E (K.E.)final

So, % increase in increase in

2 pfinal = 1 pinitial = 2p momentum = 2p p = p momentum p = 100% = 100% p 60

----------------------- Page 60----------------------REASONING (Mock Test-1) Directions (for Q. 126 to Q. 128) : Read the given information and answer the following questions. A, B, C, D, E, F, G and H are the family members. B is the sister of G and G is the brother of C. E is the wife of A, whose father is H. D is the husband of B and F is the son of G. A is the father of B. Mock Test1 (Re asoning) 126. How F is related with E ? (a) Son (c) Grand-son How D is related to C ? (a) Uncle (c) Brother-in-law How H is related to (a) Grand father (c) Uncle C ? (b) Son (d) Father (b) Mother (d) Nephew (b) Son (d) Brother

127.

128.

Directions (for Q. 129 to Q. 132) : Choose the correct relation. 129. If MALT is written as ZEBRA will be written as : (a) YFCSB (b) AFSCB (c) ACFSB (d) AFCSB If WATER is written as XBUFS, then SALT will be written as : (a) TBMU (b) UCNV (c) WZKS (d) WZSK If SING is written will be written as : (a) VCMG (c) WDNH If PAGES TIGHT will be (a) GHITT (c) HITTG as UKPL, then (b) VCGM (d) WDHN as (b) HGTTI (d) GHTTI GESPA, then TAKE NBMU, then

130.

131.

132.

is written written as :

Directions (for Q. 133 to Q. 135) : Insert the missing number in the following series. 133. 2, 3, 4, 32, 5 : (a) 6 (c) 128 30, 24, 19, 15, 12 : (a) 11 (c) 10 (b) 27 (d) 156 (b) 14 (d) 9 61

134.

135.

1, 3, 6, 10, 15, , 28, 36, 45 : (a) 17 (b) 19 (c) 21 (d) 22

----------------------- Page 61----------------------Answer Sheet (Mock Test-1) (REASONING) 126. 131. (c) (a) Hints 126. 127. 132. (c) (d) 128. 133. (a) (c) 129. 134. (d) (c) 130. 135. (a) (c)

& Solutions and E are

According to question, A husband wife, while B, G and C are their children and F is the son of G. Therefore, F is the grand son of E. ns (Reasoning) 127.

Mock Test1 Solutio B sisters.

D is the husband of B. Where as C, and G are the brothers and Therefore, D is the brother-in-law of c. As per the question, A & E are husband & wife, while B, C and children. H is the father of E. Therefore, H is the grand father of C. Each alphabet of alphabet of MLTA. will be written as AFCSB. G

128.

are

their

129.

NBMU is Therefore, is the

the ZEBRA the same

next

130.

Each alphabet of XBUFS alphabet of WATER. In SALT will be written as TBMU. The alphabet of UKPI are the next to next alphabets of SING. Therefore, TAKE will be written as VCMG. The last three alphabets of PAGES are the first three alphabets of GESPA. Similarly, last two alphabets of GESPA are the first two alphabets of PAGES. TIGHT will be written as GHTTI. The series shows that the thrid and fifth

next way

131.

132.

Therefore,

133.

numbers are the multiple of 4. Therefore, next number in the series will be 128, i.e., 32 4 = 128. 134. Therefore, next number in the series will be 10. 135. The differences of each successive and previous numbers has the sequential increase of one as : 3 1 = 2, 6 3 = 3, 10 6 = 4, 15 10 = 5, 21 6 = 15, 28 7 = 21, 36 8 = 28, 45 9 = 36, (i.e. 2, 3, 4, 5, 6, 7, 8 and 9). 62 Therefore, the missing number is 21.

----------------------- Page 62----------------------CHEMISTRY (Mock Test-2) 41. emistry) (a) negligible (c) the same 42. Calcium cynamide reacts form ammonia and : (a) CaCO 3 (c) CaO 43. SO2 oxidises : (a) K Cr O 2 2 7 (c) Mg 44. with (b) (b) smaller (d) greater steam to 3 2 (d) Ca(OH)2 (b) KMnO 4 (d) all of these many Compared to the first ionisation potential of an atom, the second potential is : ionisation Mock Test2 (Ch

Ca(HCO )

In froth floatation process chemicals (frother, collector, activator and depressant) are used. Which is called a frother ? (a) Pine oil (b) CuSO4 (c) Potassium xanthate (d) NaCN + alkali NaCl is heated : with conc. H SO 2 4

45.

and

solid K Cr O to give 2 2 7 (a) chromic sulphate (b) chromous chloride (c) chromyl chloride (d) chromic chloride

46.

When ammonical solution of salt is saturated with carbon dioxide we get : (a) MgCO3 (b) NaHCO3 (c) (NH ) CO 4 2 3 of carbonic (d) NH HCO 4 acid H CO

common

3 2 3

47.

The

anhydride

is : (a) Na CO 2 (c) CO 2 48. The best absorbent for CO (a) NaOH (c) H SO 2 49. is : 3

(b)

CO

(d) C O 2 2 (b) NaO (d) water 4 of : (b) silver (d) lead

Cinnabar is an ore (a) mercury (c) zinc

63 ----------------------- Page 63----------------------Mock 50. Test2 (Chemistry) HNO 3 anhydride of sulphurous of

The correct statement is : (a) NO is the anhydride of 2 (b) H S

is the 2 hydrosulphuric acid (c) SO2 is the anhydride acid (d) Cl is the anhydride of HCl 2 51. Which of the following carbon atom ? H H | | (a) HCCCl | | H H H | (c) HCCCH3 | H | has

asymmetric Cl | HCCH | H H | Br | | H Cl | |

(b)

(d) HCCD | |

H 52.

OH

Which molecular formula belongs to the acetylene series ? (a) C H (b) C H 5 8 4 8 (c) C H (d) C H 6 12 3 8 Which of the following not react with PCl ? 5 (a) Acetaldehyde (c) Methyl alcohol (b) Ethane (d) Acetic acid R R CO + HCN RCOH R is an example of : (a) nucleophilic addition (b) electrophilic addition (c) electrophilic substitution (d) nucleophilic substitution | CN | substances will

53.

54.

Reaction,

55.

A mixture of camphor and benzoic acid can be separated by : (a) chemical method (b) fractional crystallisation (c) extraction with a solvent (d) sublimation What is the weight of oxygen required for the complete combustion of 2.8 kg of ethylene ? (a) 9.6 kg (b) 96.0 kg (c) 6.4 kg (d) 2.8 kg 64

56.

----------------------- Page 64----------------------Mock 57. Test2 (Chemistry)

Boiling points of carboxylic acids are : (a) higher than corresponding alcohols (b) lower than corresponding alcohols (c) equal to that of corresponding alcohols (d) none of the above The reagent with which acetaldehyde and acetone react easily, is : (a) Fehlings solution (b) Grignards reagent (c) Shiffs reagent (d) Tollens reagent both

58.

59.

The density of neon (a) S.T.P. (c) 273C 1 atm PF molecule is : 3

will

be highest at : (b) 0C, 2 atm (d) 273C, 2 atm

60.

(a) trigonal bipyramidal (b) tetrahedral (c) trigonal pyramidal (d) square planar 61. Which will give positive iodoform test ? (a) CH CCl 3 O (b) CH CNH 3 O (c) CH .CHOH.CH 3 (d) CH COC H 3 O 62. The radio-activity due to C14 isotope 2 2

3 5

(t1/2 = 6000 years) of a sample of wood from an ancient tomb was found to be nearly half to that of fresh wood; tomb is, therefore, about : (a) 12,000 years old (b) 9,000 years old (c) 6,000 years old (d) 3,000 years old 63. Oxidation number of + 1 for phosphorus is found in : (a) H P O (b) H PO 4 2 7 3 (c) H PO 3 64. 4 (d) H PO 3 and alc.

the

2 3 KOH on

Aniline, chloroform heating give : (a) phenol (c) phenylcyanide

(b) chorobenzene (d) phenyl isocyanide 65

----------------------- Page 65----------------------Mock Test2 (Chemistry)

65.

The triad of nuclei that contain isotopes : 14 14 17 (a) 6C , 7N , 9F (b) 6C14, 6C13, 6C12 (c) 6C126C139F19 14 (d) 6C , 7N 14 , 9F 19

66.

Commercial detergents contain mainly : (a) ROSO Na 3 (b) RONa (c) RCOONa (d) ROCH CHORCH OR 2

2 in

67.

510 mg Victor 67.2 cm3

of a liquid on vaporisation Meyers apparatus displaces of air at STP. The molecular (b) 170 (d) 130

weight of the liquid is : (a) 1700 (c) 17 68.

For the reaction, 4A + B 2C + 2D, the statement which is not correct, is : (a) the rate of formation of D is half the rate of consumption of A (b) the rates of formation C and equal (c) the rate of disappearance of B fourth the rate of disappearance of A (d) the rate of disappearance of C is half the rate of disappearance of B Which has the minimum freezing point? (a) 1 molal CaCl2 solution (b) 1 molal urea solution (c) 1 molal NaCl solution (d) 1 molal KCl solution In aqueous solution, hydrogen (H ) will

D is one

are

69.

70.

2 not reduce (a) Ag+ (c) Cu2+ 71. In the reaction, VO + Fe O 2 3 FeO + V O . 2 5 : (b) Zn2+ (d) none of these

The equivalent weight of VO is equal to its : (a) mol wt./8 (b) mol

wt./3

(c) mol 72.

wt./6

(d) none of these

Colloidal solutions are not purified by : (a) ultrafilteration (b) electrophoresis (c) electrodialysis (d) dialysis 66

----------------------- Page 66----------------------Mock 1 73. H (g) + 2 O 2 (g) H O(l) 2 2 H298 K = 68.32 kcal Heat of vaporisation of water at 1 atm and 25C is 10.52 kcal. The standard heat of formation (in kcal) of 1 mole of water vapour at 25C is : (a) 57.80 kcal (b) +57.80 kcal (c) 78.84 kcal (d) 78.84 kcal 74. Central metal ions in a complex acts as : (a) Lewis base (b) Bronsted acid (c) Lewis acid (d) Arrhenius acid Which has the maximum undergo E1 elimination ? (a) N-butyl bromide (b) Isobutyl bromide (c) Tertiary butyl bromide (d) Secondary butyl bromide tendency to Test2 (Chemistry)

75.

76.

The order of reaction with respect to a particular component, y is zero. implies that : (a) the number of molecules forming the activated complex is zero (b) the rate of reaction is independent of the concentration of y (c) the rate of reaction is proportional to the concentration of y (d) the rate of reaction does not vary with temperature For the process, CO (s) CO (g) 2 (a) both H and S (b) H is +ve and (c) H is ve and (d) both H and are +ve S is ve S is +ve S are ve 2

This

77.

78.

Salts of A (atomic weight 7), B (atomic weight 27) and C (atomic weight 48) were electrolysed under identical conditions using the same quantity of electricity. It was found that when 2.1 g of A was

deposisted, the weight of B deposited were 2.7 g and 9.6 valencies of A, B and C are respectively : (a) 2, 3 and 2 (b) 3, 1 and 3 (c) 3, 1 and 2 (d) 1, 3 and 2 79. Nitric acid converts iodine into : (a) iodic acid (b) hydroiodic acid (c) iodine pentoxide (d) iodine nitrate The positron of : (a) a proton (c) a neutron has a charge equal to that

and g.

C The

80.

(b) an rtic (d) an electron 67

----------------------- Page 67----------------------Answers Sheet (Mock Test-2) (CHEMISTRY) 41. 46. 51. 56. 61. 66. 71. 76. (d) (b) (d) (a) (c) (a) (b) (b) Hints 41. 42. 42. 47. 52. 57. 62. 67. 72. 77. (a) (c) (a) (a) (c) (b) (b) (a) 43. 48. 53. 58. 63. 68. 73. 78. (c) (a) (b) (b) (b) (d) (a) (c) 44. 49. 54. 59. 64. 69. 74. 79. (a) (a) (a) (b) (d) (a) (c) (a) 45. 50. 55. 60. 65. 70. 75. 80. (c) (c) (a) (c) (b) (b) (c) (a)

& Solutions is to + 2NH 3 3 Mock Test2 Solutio always

Second ionisation potential greater than the first ionisation potential. Calcium cynamide reacts with steam form ammonia and calcium carbonate CaCN 2 + 3H O CaCO 2

43.

Mg is reductant so it is oxidised by SO . 2

ns (Chemistry) 44. Froth is a colloidal system of a gas liquid. It has very good adsorption capacity, frothers generate a stable froth which rises to the top of the tank. The best example of frother is pine oil. in

45.

NaCl solid

is

heated to 7

with give

conc. chromyl

H SO 2 chloride

and 4

K Cr O 2 2

(orange red) vapours. 4NaCl(s) + K Cr O (s) + 6H SO 2 2 7 4NaHSO 4 + 2KHSO 4 + 2CrO Cl 2 chromyl chloride (orange 46. When ammonical solution salt is saturated with carbon di oxide we get sodium bicarbonate (baking soda). NH + H O + CO NH HCO 3 2 2 NaCl(aq) + NH HCO 4 3 NH Cl + NaHCO 4 sodium bicarbonate (baking soda) 47. CO is an acidic gas. It turns moist blue 2 litmus slightly reddish. It dissolves in water forming unstable, dibasic, carbonic acid, hence, it is also called carbonic anhydride. +4 CO +4 + H O H CO 2 2 (aq) 2 3 carbonic acid ----------------------- Page 68----------------------Mock 48. 49. 50. Test2 Solutions (Chemistry) CO2 is 68 3 of red) common 2 2 4 + 3H O 2

CO2 is best absorbed by NaOH. absorbed by alkali to give salts. Cinnabar is an ore of mercury, it is HgS. Sulphur di oxide (SO ) is 2 fairly

soluble

in water. It produces hence, it is also known anhydride. +4 SO + H O H SO 2 2

sulphurous acid as sulphurous +4 2 3

Sulphurous acid 51. 52. An asymmetric carbon is one which has all the four groups attached on it are different. Alkynes C H , n 2n2 acetylene series. 53. PCl usually used to replace OH group 5 or oxygen CH OH + PCl 3 CH CHO + PCl 3 of carbonyl group. CH Cl + POCl 5 3 CH3 5 Acetaldehyde H Ethylidene dichloride CH COOH + PCl 3 Acetic acid 54. 55. 5 CH COCl + POCl 3 Acetyl chloride + HCl 3 C Cl Cl + POCl 3 Phosphrous oxy trichloride + HCl 3 possess so C H 5 8 is general the formula member of

Carbonyl compounds show nucleophilic addition. Aqueous solution of NaHCO3 can be used to separate benzoic acid from its mixture with camphor. Benzoic acid forms water soluble sodium benzoate with NaHCO . 3 C H + 3O 2CO + 2H O 2 4 2 2 2 1 mole ethylene need 3 mole of oxygen. i.e., 28 kg C H require 3 32 kg of oxygen 2 4 Hence, 2.8kg ethylene require 3 32 2.8 = = 9.6 kg 28 The boiling points of carboxylic acids are

56.

57.

higher than alcohols of molecular masses due to stronger H-bond formation in carboxylic acid molecules. 58.

comparable

Grignards reagent (RMgX) is a polar molecule. It adds on polar >C==O group which is present in both aldehydes as well as ketones. Fehling solution, Schiffs reagent and tollens reagent react69 with aldehydes only.

----------------------- Page 69----------------------Mock 59. Test2 Solutions (Chemistry)

Density of a gas, d = PM/RT i.e., it is directly propotional to pressure and inversely proportional to temperature. Thus under the conditions of highest pressure and lowest temperature, density of neon is maximum. 3 P-atom has sp hybridisation position occupied by lone pair of electrons. CH CHCH gives positive iodoform 3 3 | OH test. Boil CH CHCH 3 | OH 3 I NaOH 2 Iodoform (pale yellow) crystalline p.pt (pale yellow) r0 since r n and r = 2 0.693 k = t1/2 t = t = 2.303 6000 0.693 2.303 6000 = 6000 N0 log N0 2 . log 2 0.693 2 CHI 3 with one

60. 61.

N0 62. N = t

0.693 = 6000 years old 63. Oxidation found in state of H PO . +1 for phosphorus is

3(+1) + (x) + (2) = 0 3 + x 4 = 0 x = + 1 64. Aniline, chloroform and alcoholic KOH on heating gives phenyl isocyanide. This reaction is known as carbyl amine reaction. C H NH 6 5 + CHCl 2 3 (alc) C H N C 6 5 Phenyl isocyanide (foul smelling) 65. Isotopes have same atomic number but differ in the number of neutrons in nucleus so the triad of nuclei, that contain 14 13 12 isotopes are 6C , 6C , 6C . Commercial detergents sodium alkyl sulphate ROSO Na. w 67. PV = nRT wRT m = PV molecular weight = 170 ----------------------- Page 70----------------------Mock 68. Test2 Solutions (Chemistry) = 510 10 PV = m 3 RT contain 3 the + 3KCl + 3H O 2 + KOH

66.

mainly

0.0821 273
70

1 67.2/ 1000

For the reaction 4A + B 2A + 2D, the rate of reaction at a given instant can be represented as 1 dA = dB = + 1 dC = + 2 dt 1 dD

4 dt dt 2 dt the statement (d) is not correct. 69. 1 molal CaCl solution has the minimum 2 freezing point.

70.

Zn is present above electro chemical reduced by H . 2

to

the series

hydrogen in so it cannot

be

71. 72.

2+ 5+ V V2 equivalent weight = molecular weight/3 The migration of colloidal particles under the influence of electric field towards an electrode is known as electrophoresis so electrophoresis is a characteristic colloidal particles. 1 H 2 2 H = 68.32 kcal H O (l) H 2 H H + 1/2O 2 2 H O (g) 2 = 10.52 kcal H O (g) 2 = 68.32 + 10.52 57.80 kcal + O 2 H O (l) 2

of

73.

74.

Metal cation possesses the tendency to accept electron pair from ligands so the central metal ion in complex acts as Lewis acid. E1 elimination takes place formation of a stable carbonium ion which is possible from a tertiary alkyl halide only. The rate of reaction is independent of the concentration of y . Sublimation H = +ve Equivalent equivalents of C 2.1 = 7/n1 n : n : n 1 2 3 27/n2 of requires A = 2.7 = 48/n3 Equivalents 9.6 energy of i.e., B = via the

75.

76. 77. 78.

= 0.3 : 0.1 : 0.2 = 3 : 1 : 2

79. +5 10HNO

Nitric acid converts iodine into iodic acid. + 3 Nitric acid 0 I 2 iodine +5 2HIO 3 Iodic Nitrogen dioxide + 10NO 2 +4 + 4H O 2

80.

Positron is anti-electron. It possesses the same magnitude of charge as proton. 71

----------------------- Page 71----------------------ENGLISH (Mock Test-2) Directions (for Q. 136 to Q. 139) : Read the passage and answer the following questions. I was under the canvas with guns. They smelled clearly of oil and grease. I lay and listened to the rain on the canvas and the clicking of the car over the ralls. There was a little light come through and I lay and looked at the guns. They had their canvas jackets on. I thought they must have been sent ahead from the third army. The bump on my forehead was swollen, and I stopped the bleeding by lying still and letting it coagulate, then picked away the dried blood except over the cut. Mock Test2 (E nglish) 136. The (a) (b) (c) (d) author lay still because : he was injured and bleeding he was hiding he hasnt enough space to sit in he secretly wanted to guns

know

about

137.

Coagulate means : (a) to be dried up (b) to stop flowing (c) to collect (d) become thick and set The was (a) (c) vehicle in which the author travelled : a motor car (b) a railway carriage a motor trolley (d) a military truck their canvas

138.

139.

In the sentence they had jackets on the word they stands for : (a) guns (b) soldiers (c) railway carriages (d) third army Direction (for Q. 140 to Q. 142) : Choose the antonyms of the given words.

140.

ADDUCE : (a) decay (c) withdraw REPOSE :

(b) to allure (d) bring forward

141.

(a) in motion (c) do not care

(b) rest (d) tranquility 72

----------------------- Page 72----------------------Mock 142. ACCURACY : (a) perfect (c) right Test2 (English) (b) faulty (d) convect

Direction (for Q. 143 to Q. 146) : Each of the following words is followed by four likely synonyms. One of them is not synonym. Pick up that. 143. Difficult : (a) calm (c) tough Sorrow : (a) misery (c) beautiful Uniformity : (a) evenness (c) similarity Courtesy : (a) respect (c) politness (b) inconvenient (d) hard (b) unhappiness (d) sadness (b) originality (d) consistency (b) decency (d) crude

the

correct

144.

145.

146.

Directions (for Q. 147 to Q. 150) : Select the words which are related to each other in the same way as the capitalised words related to each other. 147. BORROWING : RENTAL : : ? (a) Give : Take (b) Dog : Animal (c) Leading : Landing (d) Stealing : Theft CHOP : MINCE : : ? (a) Fry : Bake (b) Pass : Fail (c) Cut : Cake (d) Speak : Listen HYGIENE : CLEANLINESS : : ? (a) Soothe : Unnerve (b) Deaf : Dumb (c) Laugh : Joke (d) Awarness : Consciousness PAYMENT : SETTLEMENT : : ? (a) Expenditure : Spending (b) Costs : Cheap

are

148.

149.

150.

(c) Worth : Account (d) Discount : Free 73 ----------------------- Page 73----------------------Answers Sheet (Mock Test-2) (ENGLISH) 136. 141. 146. (a) (c) (d) Hints 136. 137. 137. 142. 147. (d) (b) (d) 138. 143. 148. (b) (a) (a) 139. 144. 149. (a) (c) (c) 140. 145. 150. (c) (b) (a)

& Solutions

According to the passage the author lay still because he was injured and bleeding. of Coagulate is become Mock Test2 Soluti

The meaning thick and set. ons (English) 138. 139.

According to the passage the author was travelled by a railway carriage. According to the author, sentencethey had their canvas jackets on the word they is being used for guns. Adduce means to give instance but withdraw means take away. Therefore, option (c) is correct antonym. Repose means to place confidence or in the care of another but do not care having opposite meaning of repose. option (c) is correct antonym. Accuracy means conformity with truth but faulty means incorrect Therefore, option (b) is correct antonym. as in the

140.

proof

or

141.

Therefore,

142.

or

errorful.

143.

Difficults synonyms are inconvenient, tough & hard while calm has different meaning as peaceful or quite. Therefore, option (a) is not a correct synonym. Sorrows unhappiness beautiful Therefore, synonym. Uniformitys similarity originality Therefore, synonyms are and sadness has different option (c) is not synonyms are and consistency have different option (b) is not misery, while meaning. a correct evenness, while meaning. a correct

144.

145.

synonyms. 146. Courtesys synonyms are respect, decency and politness while crude have opposite meaning of courtesy. Therefore, option (d) is not a correct synonym. As per question borrowing and rental

74

147.

----------------------- Page 74----------------------Mock Test2 Solutions (English)

are synonyms, in the same way stealing and theft are also synonyms. Therefore, option( d) is correct. 148. As per question chop and mince both have same meaning, in the same way fry and bake also have Therefore, option (a) is correct. As per question hygiene and cleanliness are synonyms in the same way laugh and joke are also synonyms. option (c) is correct. As per question payment settlement are synonyms in the same way expenditure and spending synonyms. Therefore, option (a) is correct.

same

sense.

149.

Therefore, and are 75

150.

----------------------- Page 75----------------------MATHEMATICS (Mock Test-2) 81. hematics) (a) 1 t (c) 2at 82. (b) t (d) 2a the equation 2 If x = at , y = 2at, then dy is equal to : dx Mock Test2 (Mat

If , are the roots of 3x2 6x + 5 = 0, then the equation whose 2 roots are + and (a) x2 3x 2 = 0 2 (c) x + 3x + 2 = 0 (d) x

is : + (b) x2 3x + 2 = 0 2 + 3x 1 = 0

83.

The A.M. of two number is 34 and G.M. is 16, the numbers are : (a) 64 and 3 (b) 64 and 4 (c) 2 and 64 (d) none of these

84.

The value of the determinant 1 x y + z 1 y z + x is equal to : 1 z x + y (a) 1 + x + y + z (b) (x y) (y z) (z x) (c) x + y + z (d) 0 If A and B are skew symmetric of order n, then : (a) A + B is a zero matrix (b) A + B is a diagonal matrix (c) A + B is symmetric (d) A + B is skew symmetric matrices

85.

86.

Let A and B be two sets n (A) = 70, n (B) = 60 and n (A B) = 110. Then n (A B) is equal to : (a) 120 (b) 100 (c) 20 (d) 240 In ABC, a = 4, b = 12 and B = 60, then the value of sin A is : 2 3 (a) 3 (b) 2 1 (c) 2 3 1 (d) 3 2

such

that

87.

88.

If sin (120 ) = sin (120 ) 0 < , < , then all value of , given by : (a) + = (b) = or + = 3 (c) = (d) + = 0 The number of 4 digits can be formed out of the digits 3, 4, 5, 6, 7, 8, 0, digit is repeated, then their number equal to : (a) 270 (b) 720 (c) 6C4 (d) 7P2

and are 3

89.

if is

no

76 ----------------------- Page 76----------------------Mock 90. Test2 (Mathematics)

Maximum value of f (x) = sin x + cos x is : (a) 2 (b) 1 (c) 2 (d) 1 2 1 0 , B = 2 0 1 12 0 0 , then : Let A =

91.

(a) (b) (c) (d) 92. If (a) (b) (c) (d)

AB AB AB AB

BA 0 = 0 BA 0 BA = 0 |a | = |b |, then (a + b ) . (a b ) is : positive negative zero none of these

= =

0, BA 0, 0,

93.

The value of the integral dx (x ) ( x) is equal (a) 2 (c) 0 (b)

to

(d) none of these

94.

If Q.D. = 16, the most likely value of S.D. will be : (a) 42 (b) 24 (c) 10 (d) none of these The vectors 2i + j (a) = 1 (c) = 0 i + j + 2k , i + k are coplanar if : (b) = 1 (d) = 2 j k and

95.

96.

Area of parallelogram whose diagonals are a and b is : (a) a + b (b) a . b 1 (c) 2 1 |a b | 1 + 2 ! (a) 2e (c) e 1 3 ! + 4 ! (b) e (d) none of these difference unit vector, of two unit vectors then angle between (b) 60 (d) 30 77 is 1 + is : (d) |a b |

97.

1 +

98.

If the again a them is : (a) 90 (c) 45

----------------------- Page 77----------------------Mock Test2 (Mathematics)

99.

2 The curves y = x and 6y = 7 x intersect at the point (1, 1) at an angle : (a) (b) 6 3 (c) (d) 2 4 + i sin n 2 is : n 2 If xn = cos of x , x , x 1 2 3 (a) i (c) i 3 cos cos 3 + cos to : (a) 0 (c) 3 sin (b) 5 (d) 1 sin 3 + sin 3

100.

, then the vaue

(b) 1 (d) 1

101.

is

equal

102.

x 3 lim equals : x 3 x 2 4 x (a) 1 (b) 0 (c) 2 (d) none of these ex ( f (x) + f (x)) dx is equal (a) ex f (x) (c) ex ( f (x) f (x)) to :

103.

(b) ex f (x) (d) none of these is : (c) 3 e G.P., then c c (b) b d (d) c that R) may bigger fully sphere contain (centre a C , 1 (d) 2

104.

Maximum value (a) 7

of

5 + 4x 4x2

(b) 6

105.

If a, b, c, d, e are in d (a) b b (c) a

equals :

106.

In radius

order

smaller

sphere (centre C , radius r), then correct 2 relationship is : 1 (a) C C 1 (c) C C 1 107. Given 2 d f (x) = f (x) dx ff (x) dx = f (x) + e. The function f (x) could be : (a) f (x) = ex (b) f (x) = log x 1 (c) f (x) = x 78 ----------------------- Page 78----------------------Mock 108. Test2 (Mathematics) x 1 and (d) f (x) = x implies 2 < r + R < 2 (R r) (b) C C 1 < 2 (R + r) 2 (d) C C < R r 1 2

If f (x) = x + 2 when f (x) = 4x 1 when x > 1, then : (a) f (x) is discontinuous at x = 0 (b) f (x) is continuous at x = 1 (c) Lim f (x) = 4 x 1 (d) none of these and B are two det A = det B, then : = B = B 3 3

109.

If A that (a) A (c) A

matrices

such

(b) A = B (d) none of these

110.

Area bounded by lines y = 2 + x, y = 2 x and x = 2 is : (a) 16 (b) 8 (c) 3 (d) 4 All letters of the word AGAIN are permuted in all possible ways and the words so formed (with or without meaning) are written as in dictionary th then the 50 word is : (a) INAGA (b) IAANG (c) NAAGI (d) NAAIG The complex number z 1 = z + 1 3 z is : satisfying the

111.

112.

condition arg

(a) a parabola (c) a straight line 113. The projections x, y, z axes are 12, the direction cosines of the are : 12 14 3 (a) 11, < , , 11 11 11 12 4 3 (b) 19, < , , 19 19 19 12 4 3 (c) 13, < , , 13 13 13 12 4 3 (d) 15, < , , 15 15 15 100 114. If 0 equal to : (a) 3 (c) 22 115. If n is a positive divisible by : (a) 2 (c) 5

(b) a circle (d) none of these of a line segment 4, 3. The length and line segment > > > > on

1 cos 2x dx = 200k,

then

is

(b) 2 (d) integer, then n3 + 2n is (b) 3 (d) 6 79

----------------------- Page 79----------------------Mock 1 116. 1 . 2 2 . 3 (a) 4 log 2 1 (c) 2 log 2 1 117. Test2 1 + 3 . 4 (Mathematics) 1 1 + is equal 4 . 5 (b) 3 log 2 (d) none of these to :

If f (x) = 1 + x, 0 is the itself, then the value of is : (a) 2 (b) 1 (c) 2 (d) 0 The values equations in : (a) (1, 2) (c) (1, 2)

inverse

of

118.

of x which satisfies both the x2 1 0 and x2 x 2 0 (b) (1) (d) (1, 1) curve

lie

119.

The normal to the x = a (cos + sin ), y = a (sin cos )

at any point is such that : (a) it is at a constant distance origin (b) it makes a constant angle with x-axis (c) it passes through the origin (d) none of these 120.

from

the

Given the four lines with equations x + 2y 3 = 0, 3x + 4y 7 = 0, 2x + 3y 4 = 0, 4x + 5y 6 = 0. Then : (a) they are all parallel (b) they are all concurrent (c) they are the sides of a quadrilateral (d) none of these The range of the function f (x) = [x] x denotes the greatest integer x (a) [0, 1) (b) (1, 0] (c) (1, 0) (d) none of these 6 + 8i + 6 8i is equal to : (a) 3 2 i (b) 2 2 i (c) 42 i (d) none of these If y = 4x 5 is tangent y2 = px3 + q at (2, 3), then : (a) p = 2, q = 7 (c) p = 2, q = 7 to the curve

121.

122.

123.

(b) p = 2, q = 7 (d) p = 2, q = 7

124.

If

the

expansion (r + 3)th (a) n + r (c) n r 125.

co-efficient of (r + 1)th term in the 2n of (1 + x) be euqal to that of term, then : + 1 = 0 (b) n r 1 = 0 + 1 = 0 (d) none of these

The centres of the circles 2 2 2 2 x + y = 1, x + y + 6x 2y = 1 and x2 + y2 12x + 4y = 1 lie on : (a) a straight line (c) x2 = 9y (b) a circle (d) none of these 80

----------------------- Page 80----------------------Answers Sheet (Mock Test-2) (MATHEMATICS) 81. 86. 91. 96. (a) (c) (c) (c) 82. 87. 92. 97. (b) (c) (c) (b) 83. 88. 93. 98. (b) (b) (b) (b) 84. 89. 94. 99. (d) (b) (b) (c) 85. 90. 95. 100. (d) (c) (d) (b)

101. 106. 111. 116. 121.

(c) (d) (d) (c) (b) Hints

102. 107. 112. 117. 122.

(a) (a) (b) (b) (b)

103. 108. 113. 118. 123.

(a) (b) (c) (b) (d)

104. 109. 114. 119. 124.

(b) (d) (b) (a) (b)

105. 110. 115. 120. 125.

(a) (d) (b) (d) (a)

& Solutions 2 x = at dx = 2at dt y = 2at dy Mock Test2 Solutio ...(1)

81. ns (Mathematics)

dt From (1) and (2) dy

= 2a

...(2)

dy = dt = 2a

dx dy dx 82. , 2 3x 6x + 5 = 0 are

dt dx 1 2at of 6 dx the dy = t equation 1 .

roots

Sum of roots + Now ( + ) and + sum of roots = + 2 = 2 + 2 Product of roots = ( + ) . equation 2 has

= 3 whose

= 2 roots are

2 + + = 3 2 = 2 ( + ) 2 Equation is x 3x + 2 = 0. 2 . . [ . x ( + ) x + = 0] 81

----------------------- Page 81----------------------Mock 83. Test2 Solutions (Mathematics)

Let numbers be x, y The A.M. of x, y = x + y = 34 2 x + y = 68 The G.M. of x, y = xy = 16 Putting value of x as y we get 256 + y = 68 y 2 2 2 y 64y 4y + 256 = 0 y (y 64) 4 (y 64) = 0 (y 64), (y 4) = 0 256 for y = 64, x = = 4 64 256 for y = 4, x = = 64 4 So, numbers are 64 and 4 1 x y + z We have 1 y z + x 1 z x + y 1 x x + y + z = 1 y x + y + z 1 z x + y + z 1 x 1 = (x + y + z) 1 y 1 1 z 1 [Taking (x + y + z common = (x + y + z) 0 [C 3 from C ] 3 [C and C 1 are identical] 3 C 3 + C ] 2 y 68y + 256 = 0 256 + y = 68y xy = 256 256 from (2) in (1), ...(1) ...(2)

84.

= 0. 85. A and B are two matrices of same order, i.e. n Then A = A B = B (A + B) = A + B skew symmetric

= (A) + (B) = (A + B) (A + B) is skew-symmetric. 86. n (A B) = n (A) + n (B) n (A B) = 70 + 60 110 = 20 82 ----------------------- Page 82----------------------Mock 87. Test2 Solutions (Mathematics)

Using sine rule a = sin A we get, 4 = sin A sin 60 4 12 sin A = 3 2 3 2 sin A = sin A = sin A = 12 3 4 2 12 3 = 1 . 2 3 ...(1) 12 sin B b

3 3 2

88.

sin (120 ) = sin (120 ) 120 = 120 = 120 = 180 (120 ) . . [ . sin ( ) = sin ] 120 + 120 180 = + + = 60 = 3 =

from (1)

So,

or 89. + = 3 The total number of arrangements of 7 digits 3, 4, 5, 6, 7, 8, 0 taking 4 at a time is 7P . But these arrangements 4 include those numbers which have 0 at thousands place. Such numbers are not four digit numbers. When 0 is fixed at thousands place, we have to arrange remaining 6 digits taking 3 at a time. The number of such 6 arrangements is P . 3 Hence, total number of 4 digit numbers 7 = P4 6 P3 .

also

by

= 7 6 5 4 6 5 4 = 840 120 = 720 83 ----------------------- Page 83----------------------Mock 90. Test2 Solutions (Mathematics)

f (x) = sin x + cos x For maxima f (x) = cos x sin x or minima f (x) = 0 cos x sin x = 0 cos x = sin x tan x = 1 x = 4 f (x) = sin x cos x < 0 x = 4 f (x) = sin 4 1 = 1 = 2 2 2 Maximum value of sin x + cos x is sin + cos 4 4 1 1 cos 4 2 = 2 in f (x), we get

. . . Putting

= 2 + 2 2 = 2 2 Maximum value of f (x) = 2. 91. 0 0 0 1 0 0 = 0 0 = 0 0 0 1 BA = 1 12 2 0 0 = 25 0 AB = 2 92. 1 0 12 =

0 0

AB = 0, BA 0

(a + b ) . (a b ) 2 = a 2 = |a | = 0. 2 b |b | 2 . . [ . |a | = |b |] 84

----------------------- Page 84----------------------Mock 93. Test2 Solutions 2 Let x = cos So x = cos = (1 sin 2 = ( x = = = = ( from (1), ) sin 2 cos 2 (1 sin ) cos 2 2 cos cos 2 ) cos ...(3) sin 2 2 ...(2) + 2 + 2 ) + sin sin sin (Mathematics) 2 ...(1) 2 2

dx = 2 cos sin + 2 d dx d dx = 2 ( When x = , we have ) sin cos d 2 sin 2 2 = 0 = 0 = 0 , we have 2 = cos 2 (1 sin 2 cos ( So integral 2 2 ( 0 = = = 2 = 2 = . 2 94. Q.D = 3 Now Q . D = 16 2 16 = S.D. ( is S.D) cos 2 ) cos = 2 dx (x ) ( x) is ) sin cos d = 0 2 = 0 ) = cos + sin 2 2 = 0 ...(4) = 2 ( ) sin cos sin cos

2 = cos + 2 (1 cos ) sin 2 sin sin ( ) sin2 When x =

2 2 ( ) sin ( ) cos 2 2 ( ) sin cos d 0 ( ) sin cos 2 2 d 0 2 [] 0 0 2

S.D. =

3 16 3 2

S.D. = 24. 85

----------------------- Page 85----------------------Mock 95. Test2 Solutions (Mathematics)

The vectors are coplanar iff [ i + j + 2k , i + j k , 2i j + k ] = 0 1 2 = 1 1 = 0 2 1 [2 1] 1 [ + 2] + 2 [ 1 2] = 0 [expanding = 2 or = along R ] 1 3 2 2 4 = 0 3 6 4 = 0 ( + 2) (2 2 2) = 0 2 4 + 8 2 = 1 3.

96.

Area of parallelogram 1 = 2 (where d and d are diagonals) 1 2 1 = 2 1 |a b | 1 + 2 ! 3 ! ^ ^ ^ i , j , k + 4 ! be unit vectors such that ^ ^ ^ i j = k 1 + |d 1

d | 2

97.

e = 1 +

98.

Let

^ 2 i ^ 2 i + j + j ^ 2 ^ 2

^ 2 2 ( i j ) = k ^ ^ ^ 2 2 i . j = k ^ 2 2 (ij cos ) = k . . [ . a . b = ab cos ]

1 + 1 2 (1) (1) cos = 1 ^ ^ ^ . . [ . i = j = k = 1] 2 cos = 1 1 cos = 2 = 60. 86 ----------------------- Page 86----------------------Mock 99. The Test2 Solutions 2 curve y = x dy = 2x dx At (1, 1) slope of curve m1 = 2 The curve 6y = 7 x3 dy dx At (1, 1) slope Now m2 = 2 m1 m2 tan = 1 + m m 1 2 2 + 1 2 tan = 1 + 2 (1 2) 5 2 tan = 0 tan = 6 dx dy = 2 1 ...(2) = 3x 1 x 2 2 ...(1) (Mathematics)

100. We = 2 have, xn = cos + i sin n n 2 2 x1 = cos 2 + i sin 2, x2 = cos 22 + i sin 22 , x3 = cos 2 x 1 + x 2 + x 3 + i sin 22 + i sin 3 2 + sin 2 + sin + 22 23 + + i sin + + + 23 2 22 23 22 + 3 2 3 + i sin 2 3 .

= cos + i sin + cos 2 2 + cos = cos 2 + + i sin = cos + + 2 22 = cos

cos 22 + cos 23 +

2 + i sin 2 1 1 2 1 1 2 [Sum of infinite G.P., where . . . a + ar + ar 2 + = 1 r a

a = 2, r = 1 2

= cos + i sin . . = 1 + i (0) [ . = 1. cos = 1, sin = 0] 87 ----------------------- Page 87----------------------Mock cos 101. cos Test2 3 cos 3 + sin sin Solutions (Mathematics) 3 + sin 3

3 cos = [4 cos

3 3 cos ] cos 3 sin + + [3 sin 4 sin sin for cos 3 and sin 3] 2 + 3 + sin 2 4 (sin 2 + cos + 3 4 sin 2 ) + 6 2 3 ]

[Using 2 = cos

formula 2

4 cos

2 = sin + cos = 1 4 + 6 = 3. 102.

x 3 lim x 2 4 x x 3 = lim x 3 (x 3) (x 2 + 4 x) = lim x 3 = lim x 3 = lim 2 x 3 3 2 + 4 3 = 2 1 + 1 = 2 = 1 (x 2) (4 x) (x 3) (x 2 + 4 x) 2 (x 3) x 2 + 4 x

x 2 4 x

(x 3)

x 2 + 4 x x 2 + 4 x

103.

ex (f (x) + (f (x)) dx x x = e . f (x) dx + e f (x) dx = f (x) . ex dx [f (x) . ex dx] dx + ex f (x) dx + c [evaluating the first integral by parts] = ex f (x) ex f (x) dx + ex f (x) dx + c = ex f (x) + c.

104.

Let f (x) = 5 + 4x 4x2

...(1)

f (x) = 4 8x For maxima or minima f (x) = 4 8x = 0 x = f (x) = So x = 1 2 is a point of Putting x = 1 2 in (1), we 0 1 2 8 < 0 local maxima. get 1 2 4 1 4

f (x) = 5 + 4 = 5 + 2 1 = 6.

88 ----------------------- Page 88----------------------Mock 105. Test2 Solutions (Mathematics)

Let a, b, c, d, e be in G.P., with common ratio r. e 2 e d e . . = r . = r and = r c d c d e c = b d

d = r c

d 2 . . . = r b

106.

The bigger sphere is drawn with centre C and radius R such that C A = R ...(1) 1 1 The smaller sphere is drawn fully within bigger sphere with centre C and radius 2 r such that C B = r 2 from figure C C 1 C C 1 C C 1 C C 1 = C A C A 1 2 = C A (C B + AB) 2 1 2 = C A C B AB 2 1 2 = (R r) AB 2 2 ...(2)

[from (1) and (2)] C C < R r. 1 2 x x

107. dx Also from

(e ) = e x f (e ) dx = e (1) and (2). we get x f (x) = e . x + 2, x + c

...(1)

...(2)

108.

Given f (x) = 4x 1 f (1) = 1 + 2 = 3 lim + x 1 f (x) = lim h 0

when x 1 when x > 1 ...(1) f (1 + h)

= lim [4 (1 + h) 1] h 0 lim x 1 = lim [(1 h) + 2] h 0 = 3 lim + f (x) = lim f (x) = f (1) ...(3) = 3 f (x) = lim h 0 ...(2) f (1 h)

x 1 x 1 Hence f (x) is continuous at x = 1. 89 ----------------------- Page 89----------------------Mock 109. Test2 Solutions two 0 3 2 0 2 1 3 2 (Mathematics) 3 3 1 1 and B = 1 matrices 1 3 1 0 2 2 2 1 A and B 1

Let us take such that 1 2 1 1 det A =

A =

1 1 = [3 2] + 1 [4 3] 1

= 1 + 1 = 2 1 3 1 0 2 2 1 2 = 1 [2 4] + 1 [6 2] 1

det B =

= 2 + 4 = 2 Here det A = det B 1 But A = 0 1 1 Also A = 0 1 2 3 1 2 3 1 1 2 B 1 1 1 2 B = 0 1 1 3 2 2 1 2 1

Also A B So, none of above follows. 110. The lines y = 2 + x and x = 2 insersect point C. co-ordinates of C are (2, 4). From the figure, clearly area bounded by the 3 curves = area of ABC Let AO be the perpendicular on BC AO = 2 [from figure] Now, area of ABC 1 = 2 1 = 2 at

BC AO 4 2

= 4 sq units. 90 ----------------------- Page 90----------------------Mock 111. Test2 Solutions (Mathematics) words. arranging

Starting with letter A, and arranging the other 4 letters, there are 4 ! = 24 There are the first 24 words. Then starting with G, and A, A, I and N in different ways, there are 4 ! 24

= 2 ! 1 ! 1 ! Next, are the 12 37 words 2 th

= 12 words word starts starting th with I. There with I. word. th word

This

accounts upto the 48 th The 49 word is NAAGI. The 50 is NAAIG. 112.

Let P (z), A (1 + 0i) and B (1 + 0i) be the given points. z 1 Then, arg = z + 1 3 BPA = 3 P moves in such a way that BPA = 3 P lies on a circle. Hence, complex number, z satisfying the z 1 equation arg = is a circle. z + 1 3 Projections of line segment on x, y, z axes are 12, 4, 3 length = 122 + 42 + 32 = 144 + 16 + 9 = 169 = 13 and direction cosines are 12 4 3 , , 13 13 13

113.

114.

We have 100 0 100 0 91 2 sin2 x dx = 200k 1 cos 2x dx = 200k

----------------------- Page 91-----------------------

Mock

Test2

Solutions

(Mathematics) 2 . . [ . 1 cos 2x = 2 sin x]

2 (100) 0 . . [ . |sin x | has period ] 2 (100) [ cos x] = 200k 0 2 [ cos + cos 0] = 2k 2 [ + 1 + 1] = 2k . . [ . cos = 1 and cos 0 = 1] 2 2 = 2k k = 2. 115. 3 3 For n = 1, n + 2n = 1 + 2 = 3 which is divisible by 3 n3 + 2n is divisibly by 3 for smallest positive 1 116. integer. 1 + 1 |sin x | dx = 200k

1 . 2 2 . 3 3 . 4 = 1 1 1 1 + 1 1 2 2 3 3 1 = 1 2 2 3 1 + 4 1

1 1 1 = 1 + 2 1 + 2 3 4 Now logarithmic series 2 x loge (1 + x) = x 2 Putting x = 1, we get 1 log 2 = 1 e Putting + 2 3 1 4 1 + + 3 x 4 3 x + 4

value of log 2 above, we get e = 1 + 2 log 2 = 2 log 2 1. e

92 ----------------------- Page 92----------------------Mock 117. Test2 Solutions (Mathematics)

f (x) = 1 + x Let 1 Now f = f 1 + x = f f (x) =

y = f (x) 1 x = f (y) y = 1 + x y 1 x = y 1 (y) = x 1 1 (Given) 1 f (x) = f x 1 (x)

2 + x = x 1 2 + 1 = x (1 ) 2 x + x = + 1 2 x + + (1 x) = 0 This is a quadratic equation in For roots to be real Discriminant > 0 1 4x (1 x) > 0 2 1 4x + 4x > 0 2 (2x 1) > 0 1 x > 2 Putting value of x in (1), we get 2 1 + + 1 1 = 0 2 2 2 1 + + = 0 2 2 2 + 2 + 1 = 0 2 ...(1)

( + 1) = 1. 118.

= 0

x2 1 0 and x2 x 2 0 x2 1 and (x 2) (x + 1) 0 or x 2 0, x + 1 0 or x 2 0, x + 1 0 1 x 1 and x 2, x 1 or x 2, x 1 1 x 1 and x 1 or x 2 = { 1} So (1) value of x satisfies equations x2 1 0 and x2 x 2 0 both the

93 ----------------------- Page 93----------------------Mock 119. Test2 Solutions (Mathematics)

x = a (cos + sin ) dx = a ( sin + cos + sin ) d dx = a cos d y = a (sin cos ) dy d dy = a sin d from (1) and (2) dy = dx Slope of the normal 1 = tan normal = y a (sin cos ) = cot [x a (cos + sin )] [equation of normal y y = m (x x )] 1 1 2 = cot d dy ...(2) = a (cos + sin cos ) ...(1)

d = dx

a sin = tan a cos

y sin a (sin = cos (x a cos a [multiplying both sides 2 y sin a sin = x cos x cos + x cos + Length upon the =

sin cos ) sin ) by sin ]

+ a sin cos 2 + a cos + a sin cos 2 2 y sin a (sin + cos ) = 0 y sin a = 0 of perpendicular from (0, 0) normal

2 2 = a = constant. cos + sin equation are x + 2y 3 = 0 3x + 4y 7 = 0 2x + 3y 4 = 0 ...(1) ...(2) ...(3) ...(4)

120.

The

Solving

equation

4x + 5y 6 = 0 (1) and (2), we get

3x + 6y 9 = 0 [Multiplying (1) by 3] 3x + 4y 7 = 0 2y 2 = 0 y = 1 Putting value of y in (1), we get x + 2 3 = 0 x = 1 1) lies on 3x + 4y 7 = 0 but on 2x + 3y 4 = 0 0. are neither concurrent, nor a quadrilateral nor parallel.

The point (1, not 4x + 5y 6 = Hence, they they can form

and

94 ----------------------- Page 94----------------------Mock 121. Test2 Solutions (Mathematics)

f (x) = [x] x = [x (x)] . . = {x} [ . [x] + {x} = x] Domain of {x} = [0, 1) Now putting 0 in {x}, we get f (x) = 0 Similarly, putting 1 in {x}, we get f (x) = 1 Range = ( 1, 0).

122.

Let 6 + 8i = a + ib 6 + 8i = (a + ib)2 [Squaring both sides]

2 6 + 8i = a

2 2 + i b + 2iab

2 2 6 + 8i = (a b ) + 2iab Comparing the co-efficients of both sides 2 b = 6 2ab =8 4 a = b Putting value of a in(1) 16 2 b = 6 2 b 4 2 16 b = 6b b4 + 6b2 16 = 0 b4 + 8b2 2b2 16 = 0 a 2 b b we get So, (b 2 2 + 8) 2 (b 2 2 = 0, b + 8 = 0 b = 2 4 a = 2 = 22 2 + 8) = 8 2 ...(1)

6 + 8i = 2 2 + 2 i 6 8i = 2 2 2 i 6 + 8i 6 8i = (2 2 + 2 i) (2 2 2 i) = 2 2 i. 95

----------------------- Page 95----------------------Mock 2 123. y At (2, 3), we = px Test2 3 + q dy 2y dx have dy 6 dx dy dx Tangent at (2, 3) is y 3 = 2p (x 2) = 2p (slope of tangent) = 3p (4) = 12p = 3px 2 Solutions (Mathematics)

i.e. Also Comparing

y = 2px + 3 4p y = 4x 5 is a tangent 4x 5 = 2px + 3 4p the co-efficients, we get 2p = 4 3 4p = 5 i.e. 4p = 8 p = 2 2 3 Also (2, 3) lies on y = px + q 2 3 . . . (3) = p (2) + q 9 = 8p + q q = 9 8p q = 9 16 q = 7 p = 2 Hence p = 2, q = 7. 2n 124. Tr + 1 = T r + 3 Cr x 2n = T (r + 2) + 1 = C r + 2 in x r + 2 r

Since the co-efficient of (r + 1)th term the expansion of (1 + x)2n be equal to that of (r + 3)th term, 2n Cr = Cr + 2 r + r + 2 = 2n n = r + 1 n r 1 = 0. 2n

96 ----------------------- Page 96----------------------Mock 125. The Test2 Solutions of (Mathematics) the circle x2 + y2 = 1 is

centre

O (0, 0) . . [ . circle is of 2 2 2 (x a) + (y b) = r ] The centre of the circle 2 x + y 2 + 6x 2y = 1 is A (3, 1) . . [ . The circle is of 2 2 x + y + 2fx + 2gy + c = 0 = (, )]

the

form

the and

form centre

The centre of 2 2 x + y 12x + 4y = 1 is B (6, 2) . . [ . The circle is of x2 + y2 + 2fx + 2gy + c = 0]

the

circle

the

form

2 2 OA = (3 0) + (1 0)

(distance between first two circles) = 9 + 1 = 10 2 2 AB = (6 + 3) + (2 1) = 81 + 9 = 310 2 2 OB = (6 0) + (2 0) = 36 + 4 = 40 = 210 Now, AO + OB = 10 + 210 = 310 = AB A, O, B lie on a straight line. 97 ----------------------- Page 97----------------------PHYSICS (Mock Test-2) 1. An ideal gas with pressure P, volume V and temperature T is expanded isothermally to a volume 2V and final pressure P . The same gas is expanded 1 adiabatically to a volume 2V and the final pressure P . In terms of the ratio 2 two specific heats for the gas, the ratio P P is : 1 2 hysics) (a) 2 (c) 21 2. A two radioactive hours and element 1 its 16 undisintegrated, element will be : (a) 4.0 hour (c) 0.5 hour 3. the half life of the (b) 2.0 hour (d) 1.0 hour 0 the masses of all the molecules are halved th part remain (b) 2 (d) 2 1 disintegrates for of the

Mock Test2 (P

The pressure of a gas in a vessel is P . If

and their speeds resulting pressure P will be (a) 2P0 (c) P 2 0 4.

are : (b) P0 (d) 4P 0

doubled,

the

The element used for carbon-dating for more than 5600 years, is : (a) Po94 (b) U238 (c) U234 (d) C 14 In a circuit, the current is measured by hot wire as 10 A. Its peak value will be (a) 7.07 A (c) 20 A value of ammeter : (b) 14.14 A (d) 10 A

radioactive

5.

alternating

6.

If R and L represent respectively resistance and self-inductance, which of the following combinations has dimensions of frequency ? (a) L R (b) R L 2 2 (c) R L (d) R L The energy gap between conduction and valence bands of silicon is : (a) 2 eV (b) 1.7 eV (c) 1.1 eV (d) 1 eV

the

7.

98 ----------------------- Page 98----------------------Mock 8. Test2 (Physics)

Diode is used as a/an : (a) modulator (b) oscillator (c) amplifier (d) rectifier A polyatomic gas ( = 4 3) at pressure P is compressed to (1/8)th of its initial volume adiabatically. The pressure will change to : (a) 32 P (b) 16 P (c) 8 P (d) 4 P In a satellite, revolution is T, then be proportional to : 1 (a) 3 T 1 (c) T if the time kinetic energy (b) T 1 (d) T2 period will of

9.

10.

2 3

11.

When the current in a 2A to 4A in 0.05 sec, induced in self-induction of the (a) 0.8 H (c) 0.2 H

coil changes from an emf of 8 volts is the coil, the coefficient coil is : (b) 0.4 H (d) 0.1 H valve

of

12.

The amplification factor of triode depends upon the relative : (a) change in grid voltage (b) change in plate voltage (c) distance between the plate from cathode (d) change in cathode voltage

grid

and

the

13.

The dimensional formula of gravitational potential is : 2 2 2 (a) [LT ] (b) [L T ] 2 2 3 2 (c) [L T ] (d) [L T ] A ball of mass 0.2kg moves velocity of 20m/sec and it 0.1 sec. The force acting on the ball is : (a) 10 N (b) 15 N (c) 20 N (d) 40 N When a transistor is used then : (a) emitter-base junction biased and the base-collector junction is reverse biased (b) emitter-base junction is reverse biased and the base-collector forward biased (c) both junctions are forward biased (d) both junctions are reverse biased in is with stops a in

14.

15.

circuit, forward

junction

is

99 ----------------------- Page 99----------------------Mock 16. Test2 the (Physics) following statement is

Which of wrong ? (a) Voltmeter has connected in (b) Ammeter connected in (c) Ammeter connected in (d) None of these

high resistance and is parallel has low resistance series has low resistance parallel

and and

is is

17.

The lengths of two open organ pipes are l and (l + l) respectively. Neglecting end correction, the frequency of beats will be : v l v

(a) 2 2l v (c) 4l 18. If 0.5 resistance henry and

(b) 2 2 l v (d) 2 l of 100 , capacitance of inductance 10 10 6F of

are connected in series through 50 hertz A.C. supply, the impedance will be : (a) 1891.7 (b) 190 (c) 18.7 (d) 1.87 19. For the structural analysis Xry are used bacause : (a) Xry have wavelength of the order of interatomic spacing (b) Xry are highly radiations (c) wavelength of Xry is of the order of nuclear size (d) Xry are coherent radiations of crystal,

penetrating

20.

A particle A has charge + q and a particle B has charge + 4q with each of them having the same mass m. When both particles are allowed to fall from rest through the same electro-potential vA difference, the ratio of their speeds vB will become : (a) 4 : 1 (b) 2 : 1 (c) 1 : 2 (d) 1 : 4 Generally, the visible spectrum is : (a) 100 to 1500 (b) 2000 to 3500 (c) 10000 to 14000 (d) 4000 to 7900 approximate limit of

21.

100 ----------------------- Page 100----------------------Mock 22. Test2 (Physics) is

An achromatic combination of lenses formed by joining : (a) 2 concave lenses (b) 1 convex and 1 plane mirror (c) 2 convex lenses (d) 1 convex and 1 concave lens In a face centered cubic cell, the number of atoms per unit cell is : (a) 4 (b) 3

23.

(c) 2 24.

(d) 1

The induced currents always produce expanding magnetic fields round their conductors in a direction that opposes the original magnetic field. This law is called : (a) Lenzs law (b) Flemings law (c) Ohms law (d) Kirchhoffs law A 3 F capacitor is charged to a potential of 300 V and a 2 F capacitor is charged to 200V. The capacitors connected in parallel with opposite polarity joined together. What amount of charge will flow plate are so connected ? (a) 1300 C (b) 700 C (c) 250 C (d) 600 C The difference in pressure of outer inner surface of a soap bubble of radius R and tension T, is : T (a) T (b) R T 4T (c) (d) 2R R The resultant circuit is : 4 (a) 3 10 (c) 3 (d) 3 101 (b) 3 20 resistance 7 in

25.

are then plates of when the

26.

and

27.

the

given

----------------------- Page 101----------------------Mock 28. Test2 (Physics) 1 superimposed, amplitude is : (a) A2 + A2 1 2 (b) A2 + A2 + 2A 1 2 (c) A1 + A2 (d) A1 A2 29. The equation of a simple harmonic then the common 2

If two waves of amplitude A and A are

A cos 1 2

progressive

wave is y = 0.2 sin (157t 1.57x), where t, x and y are in second, metre and metre respectively. The frequency wavelength of the wave are : (a) 25 sec1 and 2m (b) 30 sec1 and 6m (c) 25 sec 1 and 4m (d) 30. 36 sec1 and 8m

and

Work done in time t on a body of mass m which is accelerated from rest speed v in time t , as a function of time 1 t, is given by : t (a) 2 mv t1 v 2 (c) m t t1 1 2 1 mv 2 (b) 2 t1 t 1 v2 2 (d) m t 2 t2 1

to

31.

An air bubble in a glass of water shines brightly due to : (a) total internal reflection (b) diffraction (c) refraction (d) none of these The frequency of an open organ pipe f. If half part of the organ pipe is dipped into water, then its frequency will be : f (a) (b) 2f 2 3f (c) (d) f 4 The radioactive The reaction number (b) 6 (d) 10 102 of is is

32.

33.

92U238 rtic

to

234 90Th . emitted are : (a) 1 (c) 8

----------------------- Page 102----------------------Mock Test2 (Physics)

34.

The a (a) (b) (c) (d)

factor which determines the pitch of tuning fork, is : physical condition frequency wavelength none of these The in

35.

A convex glass lens (g = 1.5) has a focal length of 8 cm when placed in air. focal length of the lens when placed water (w = 4 3) is : (a) 8 cm (b) 4 cm (c) 16 cm (d) 32 cm The ratio of the intensities interfering waves is 4 : 1. The maximum to minimum intensity is : (a) 1 : 4 (b) 3 : 1 (c) 4 : 1 (d) 9 : 1 The instrument, temperature by measuring radiations, is called : (a) hydrometer (b) pyrometer (c) thermopile (d) thermometer The (a) (b) (c) (d) susceptibility is positive ferromagnetic substance diamagnetic substance paramagnetic substance non magnetic substance the which intensity

36.

of two ratio of

37.

measure

38.

for :

39.

A fly wheel is in the form of solid circular wheel of mass 72kg and radius of 0.5m. If it revolves at 70 rpm, then the energy of the ratation is : (a) 2400 J (b) 240 J (c) 24 J (d) 2.4 J A radioactive substance has a half-life of 60 minutes. After 3 hours, the fraction of atoms that have decayed, would be : (a) 87.5% (b) 25.1% (c) 8.5% (d) 12.5% 103

40.

----------------------- Page 103----------------------Answers Sheet (Mock Test-2) (PHYSICS) 1. 6. 11. 16. 21. (d) (b) (c) (c) (d) 2. 7. 12. 17. 22. (c) (c) (c) (a) (d) 3. 8. 13. 18. 23. (a) (d) (c) (b) (a) 4. 9. 14. 19. 24. (d) (b) (d) (a) (a) 5. 10. 15. 20. 25. (b) (b) (a) (c) (a)

26. 31. 36.

(d) (a) (d) Hints

27. 32. 37.

(d) (d) (b)

28. 33. 38.

(b) (a) (a)

29. 34. 39.

(c) (b) (b)

30. 35. 40.

(d) (d) (a)

& Solutions Mock Test2 Soluti

ons (Physics) 1. Since A is compressed isothermally, then P PV = P .2V 1 Again adiabatically, since gas or P1 = 2 = 2 1 P2 2. 2 decay 0 1 where 1 So, 16 14 2 N = N, t = 2 hour 0 16 12 T1 2 = 2 12 T1 2 = 2 2 4 = T1 2 2 Hence, 3. T1 2 = 4 = 2 1 = 0.5 hour 1t T1 2 2 law, the is P 2 2 So, or P 1 2 compressed P PV = P (2V ) 2 = =

From the radioactive amount left after time t is N = N

According to kinetic theory of gases, the pressure of an ideal gas is _2 1 mnv P = 3 V 1 P m 3 _2 P m v 1 1 1

or P2

= _2 m v 2 2 m _ , v 1 2 _ _ v2 = 2v P _ 2 0 mv = = P2 m _ 2 (2v) 2 _ = v

Here : P

= P , m 1 0

= m, m 1 2

1 2 104

----------------------- Page 104----------------------Mock Hence, 4. 5. Element carbon-dating Test2 Solutions P2 = 2P0 C14 is for more than used for 5600 years. radioactive by (Physics)

The rms value of current is given Irms = 10 A Peak value of current is I0 = 2 Irms = 2 10 = 10 2 = 10 1.414 = 14.14 A 2 3 A 2 The dimensions of L = [ML Therefore, dimensions of 2 R [ML T = L 2 [ML T 1 = [T] = dimensions T 3 A 2 A 2 ] 2 ] = [T 1 ] 2 A 2 ] 2 ]

6.

The

dimensions of R = [ML T

of frequency band of

7. 8.

Conduction band and valence silicon are separated by energy gap of 1.1 eV.

When plate of diode is connected to A.C. source, then during positive half cycle when plate is at positive potential, current will flow through diode and during negative half cycle, current will not flow through it. Thus diode can be used as a rectifier. The adiabatic equation of an ideal gas is

9.

given by PV = constant Here : P V = P V 1 1 2 2 1 = 4 3, P1 = P, V2 = 8 So, P2 = P1 V 1 V2 4 3 = P (2 ) in 3 4 3 V1

Hence, 10.

= P (8) P2 = 16P

For a satellite to revolve orbit, kinetic energy = potential energy GMe m K = 2R 1 or K R Now according to Keplers law, we have 2 3 T R or R T Therefore, kinetic energy K T 1 2 3 [from 2 3

a circular

(i)

(ii)

eqs. (i) and (ii)] 105

----------------------- Page 105----------------------Mock or 11. Test2 K T Solutions 2 3 The induced emf in the coil is given by i e = L Here : t e = 8 volt, i = 2 4 = 2 A, t = 0.05 sec t L = e i = 8 8 0.05 0.05 (2) (Physics)

= 2 = 0.2 H 12. Amplification factor is defined as the ratio of change in anode potential required to produce a certain change in the plate current to the change in the grid voltage required to produce the same change in the plate current. So, it depends upon the distance between the grid and plate from cathode. The gravitational potential of a body is work done V = mass W V = m dimensions of W Dimensions of V = dimensions of m 2 [MLT = [M ] 14. Using or the law of motion v = u + at 2 the body from ] [L] 2 = [L T 2 ]

13.

0 = 20 a 0.1 20 or a = = 200 m s 0.1 Hence, force acting on Newtons second law is given by F = ma = 0.2 200 = 40 N 15.

While using transistor in emitter-base junction should be forward biased and base-collector junction should be reverse biased.

circuit,

16.

A voltmeter is an instrument used to measure the potential difference between two points and is connected in parallel. Moreover, the resistance of an ideal voltmeter is high. On the other hand, ammeter measures current, so, it is connected in series. The resistance of an ideal ammeter is zero. 106

----------------------- Page 106-----------------------

Mock

Test2

Solutions

(Physics)

Hence, option (c) is wrong. 17. The frequency of organ pipes are given as v n1 = and n2 = beats v for two open

2l 2(l + l) Therefore, beats frequency n = n1 n2 v v = 2l 2(l + l) = v l + l l 2 l (l + l) v l 2l2 (approx.) an A.C. inductance 2 ~ X ) C L 2 2 = R + 1 2L circuit and

v l = 2 l (l + l) = 18. The impedance of containing resistance, capacitance is 2 Z = R + (X

Here : R = 100, L = 0.5 H C = 10 10 f = 50 Hz

2c

6 F 2

Z = (100)2 + 1 2 3.14 50 10 10

2 3.14 50 0.5

= 10000 + (318.5 157)2 2 10000 + (161.5) = 10000 + 26082.25 = 36082.25 = 189.95 190 = 19. The diffraction of X-rays can take place only when spacing between two adjacent planes is of the order of wavelength of X-rays. It happens so in case of crystals. So, X-rays are used for structural analysis of crystals.

20.

Charge on particle A = + q Charge on particle Velocity or vA Therefore, vB = qB of particle is v =

B = + 4q 2qV m

v q qA

107 ----------------------- Page 107----------------------Mock Test2 Solutions (Physics) 1 = 1 : 2 4

q = = 4q 21.

The spectrum for which the human eye is sensitive is called visible spectrum. The range of visible spectrum is from 4000 to 7900 . A combination of lenses by which images of different colours of an object formed at any point, is called achromatic combination of lenses. obtain an achromatic combination lenses, a convex lens of crown glass is placed in contact with a concave lens of flint glass and they are joined together by the transparent canada balsam layer. A face centered cube has 6 face centered and eight corner atoms. The six centered atoms at six faces of the cube are shared by two adjacent cells. Similarly each corner atom is shared by surrounding cells. Hence, effective number of atoms unit cell of face centered cube is = 8 1 + 6 1 = 1 + 3 = 4 8 2 are the To of

22.

23.

face eight per

24.

The induced currents always produce expanding magnetic field round their conductors in a direction that opposes the original magnetic field. This is explained by Lenzs law. The capacitance C1 = 3 F The capacitance C2 = 2 F of of first second capacitor capacitor

25.

Voltage across first capacitor V1 = 300 V Voltage across second capacitor V2 = 200 V In parallel combination, charge across first capacitor q = C V 1 1 1 = 3 300 = 900 C charge across second capacitor q = C V 2 2 2 = 2 200 = 400V C 108 ----------------------- Page 108----------------------Mock Test2 Solutions (Physics)

Therefore, total charge

in circuit

q = q1 + q2 = 900 + 400 = 1300 C 26. The soap bubble has two surfaces inner surface and outer surface. The force on soap bubble due to surface tension = T (2 2R) = 4RT In equilibrium, P1 R 2 = P2 R 2 + 4RT ; the

where P1 = pressure inside the bubble P2 = pressure outside the bubble T = surface tension (P P ) R 1 2 (P 2 = 4RT 4T So, P ) = 1 2 R 27. From the figure P = Q R and S P Therefore, bridge, so, resistance. = 12 R 6 8 = 3 4 = 3 2 2

= Q S it is balanced no current will flow

Wheat-stone in 10

Equivalent resistance of upper arm = 4 + 6 = 10 Equivalent resistance of lower arm = 8 + 12 = 20 Net resistance between A and C is 10 20 20 R = = 10 + 20 3 28. When two waves of amplitude A1 A2 are superimposed, amplitude is given by

and then

resultant

2 2 A = A + A + 2A A cos 1 2 1 2 where = phase difference between two waves 29. Given equation is y = 0.2 sin (157t 1.57x) Comparing equation (i) with equation y = a sin (t kx) we get, = 157 or 2 = 157 157 1 or f = = 25 sec 2 3.14 and k = 1.57 (i) standard (ii)

109 ----------------------- Page 109----------------------Mock or or = Test2 2 = 1.57 2 3.14 1.57 v 30. We know acceleraton = a v Force = m.a = m. t1 Distance covered in time t 1 2 1 v 2 s = 2at = 2 t1 t Work done = force distance mv = t1 1 v 2 t1 t 2 1 v = 2m t1 t = 4 m Solutions (Physics)

2 2

31.

An air bubble in a glass of water shines because of total internal reflection. A ray of light going from water bubble is totally reflected bubble shines. Frequency of an open organ pipe is given by v f = 2l where v = velocity of transverse wave in the pipe l = length of open organ pipe When half length of pipe is dipped in to water, it behaves as a closed organ pipe, then frequency v f1 = 4 l 2 2v f1 = 4l eqs (i) and (ii), we f1 = f get

into and

an the

air air

32.

(i)

From

(ii)

33.

In the reaction 92U238 90Th234 The decrease in mass number = 238 234 = 4 The decrease in atomic number = 92 90 = 2 Therefore, one (alpha) particle 2He4 emitted. is

34.

Pitch sound

is and

the depends

characteristic of upon its frequency.

musical 110

----------------------- Page 110----------------------Mock Test2 Solutions (Physics) of are low

The tones of higher frequency interpreted as shrill and that frequency are interpreted as grave.

35.

Using

the relation for convex lens 1 1 1 f = ( a g 1)R1 R2 fw (a g 1) = fa (w g 1) where fw = focal length of lens in water fa = focal length of lens in air = 8 cm a g = 1.5 1.5 w g = 4 3 = 1.125 fw 8 or Hence, fw = = (1.125 1) 8 0.5 0.125 fw = 32 cm I1 4 = I2 We know I1 So, I2 a1 or a2 I max Therefore, Imin = (a 1 (2a = 2 (2a a ) 2 2 2 a )2 2 2 + a ) 2 2 a1 = 2a2 2 (a + a ) 1 2 = 1 = a2 2 2 1 I a2 2 a1 = 1 4 (1.5 1)

36.

Given :

3a 2 = a2 9 I max Hence, Imin 37. Pyrometer is the device to measure the temperature by measuring the intensity of radiations received from the body. It is based on Stefans law. 111 ----------------------- Page 111----------------------Mock 38. 39. Test2 Solutions (Physics) = 9 : 1 = 1

Susceptibility of ferromagnetic substance is very high and positive. The angular velocity is given by = 2  t = 7.3 rad sec 1 Moment of inertia is I = 2 1 = 2 Therefore, given by 2 = 9 kinetic 2 I 2 1 = 2 2 energy is Mr 2 = 2 70 60

72 (0.5)

rotational 1

Erotational =

9 (7.3)

= 239.8 J 240 J 40. The amount t is of substance left after time

N = N

1t T1 2 0 2

Here : t = 3 hour = 3 60 minute = 180 minute T1 2 = half-life = 60 minute N0 = initial number of atoms N = N 1180 60 0 2 3 = N 1 = N0 0 2 8 Therefore, amount decayed after 3 hours N0 = = 8 = 0.875N0 = 87.5% N0 112 ----------------------- Page 112----------------------REASONING (Mock Test-2) Directions (for Q. 126 to Q. 130.) : Answer the following questions. soning) 126. B is the aunt of C and A is the husband of B. B is the sister of D. How is D related to A ? (a) Aunt (b) Nephew (c) Brother-in-law (d) Uncle A is the daughter of B, C is the sister of A. B is the daughter of D. How related to D ? (a) Aunt (b) Daughter (c) Grand mother (d) Grand daughter Rajeev is the brother of Mohan. Sonia is the sister of Sudhir. Mohan is the son of Sonia. How is Rajeev related to Sudhir ? (a) Father (b) Brother (c) Son (d) Nephew Arun is the father of Rohit. Rohit is the brother of Mala. Mala is the wife of Dilip. How is Dilip related to Rohit ? (a) Uncle (b) Brother-in-law (c) Father-in-law (d) Son is C Mock Test2 (Rea N0 8 7N0

127.

128.

129.

130.

Anand is the husband of Shama. Vibha is the sister of Anand. Sangeeta sister of Shama. How Sangeeta is related to Vibha ? (a) Uncle (b) Son (c) Brother-in-law (d) Sister-in-law Directions Complete the options.

is the

(for Q. 131 to Q. 133) series with the help of given

131.

125, 235, 345 : (a) 173 (c) 259 138, 161, 185, 210 : (a) 109 (c) 331 1 2 , 12 (a) 9 6 (c) 7 11 2 , 10 3 , :

(b) 358 (d) 455 (b) 236 (d) 402

132.

133.

5 (b) 7 4 (d) 9 113

----------------------- Page 113----------------------Mock Test2 (Reasoning) to on 135) : Answer the basis of the

Directions (for Q. 134 the given questions alphabate. 134.

cc cc, dd d, cc cc, dd dd : (a) d, c, d, c, d (b) d, c, c, d, d (c) d, c, d, c, d (d) d, c, d, d, c k mnkl klmn (a) nml (c) nlk : (b) lmn (d) rol 114

135.

----------------------- Page 114----------------------Answers Sheet (Mock Test-2) (REASONING) 126. 131. (c) (d) Hints 127. 132. (d) (b) 128. 133. (d) (d) 129. 134. (b) (d) 130. 135. (d) (b)

& Solutions

126.

According to question, A & B are husband and wife and B is Ds sister. D is the brother-in-law of A. ns (Reasoning) 127.

Therefore, Mock Test2 Solutio are is the the

According to question, A & C daughters of B, while D mother/father of B. Therefore, C is the grand daughter of D. According to question, Sonia mother of Rajeev & Mohan. While Sudhir is the brother of Sonia. Therefore, Rajeev is the nephew of Sudhir. As per the question, Rohit & Mala are brother & sister while Dilip husband of Mala. Therefore, Dilip is the brother-in-law of Rohit. According to question, Anand and Shama are the husband and wife while Vibha and Sangeeta are the sisters of Shama respectively. Therefore, Sangeeta is the sister-in-law of Vibha.

128.

is

the

129.

is

the

130.

Anand

and

131.

In the given series each successive no. is ovtained by adding 110 to the previous number. Therefore, the next no. will be 345 + 110 = 455

132.

In the given series each successive no. is obtained by adding 23, 24 and 25 to the previous no. respectively Therefore, the next no. will 120 + 26 = 236 115 be

----------------------- Page 115----------------------Mock 133. Test2 Solutions (Reasoning) by one successive 4 Therefore, the next number will 134. be 9 In the set of five alphabets, the middle one is different with remaining Therefore, the series will be as follows. ccdcc, ddcdd, ccdcc, ddcdd. four. .

In the given series the numerator of each successive number increases while denominator of each number decreases by one :

Option 135.

(d) is correct in

In the set of three alphabets, is repaining itself continously. Therefore, complete series will be as follows. klmnklmnklmn. Option (b) is correct.

116 ----------------------- Page 116----------------------CHEMISTRY (Mock Test-3) 41. Sapphire is a mineral of : mistry) (a) Cu (c) Hg 42. Mock Test3 (Che (b) Al (d) Zn

Which element is most metallic ? (a) Bismuth (b) Antimony (c) Arsenic (d) Phosphorus The element cited as an example to prove the validity of Mandeleefs periodic law is : (a) scandium (b) germanium (c) gallium (d) all of these Sodium thiosulphate is formed when (a) Na SO is boiled with Na S 2 4 (b) Na SO 2 3 is boiled with : 2 Na S and I 2 2

43.

44.

(c) Na S is boiled with S 2 (d) 45. NaOH is neutralised by H SO 2 Which form of percentage of carbon ? (a) Steel (b) Wrought iron (c) Cast iron (d) All have same percentage iron has the 4 lowest

46.

The molecular weight of heavy water is : (a) 20 (b) 16 (c) 12 (d) 10 The number of different substitution products possible when ethane is allowed to react with bromine in sunlight are : (a) 5 (b) 8 (c) 6 (d) 9 Which reaction metal loses with ozone ? its meniscus after

47.

48.

(a) Cu (c) Ag 49. Ammonium detection of : 3 (a) PO4 (c) Bi3+ molybdate

(b) Pb (d) Hg is 2+ (b) Cd (d) Cu3+ used for the

50.

Which is bad conductor of electricity ? (a) HCl (b) HBr (c) HI (d) H F 2 2 Glucose and fructose are : (a) optical isomers (c) position isomers (d) chain The rare gases are (a) polyatomic (c) diatomic : (b) triatomic (d) monoatomic 117 (b) functional isomers isomers

51.

52.

----------------------- Page 117----------------------Mock 53. Test3 (Chemistry) presence of

Stephans reaction is reduction of : (a) acyl halide in the Pb/BaSO4 (b) alkyl isocyanide with Na and alcohol (c) alkyl cyanide with SnCl2 and HCl (d) alkyl cyanide with LiAlH4 Which would not react sulphonyl chloride in an aqueous NaOH? (a) N-ethyl aniline (b) p-toluidine (c) N, N-dimethyl aniline (d) aniline

54.

with

benzene

55.

Which of the following alcohols is most likely to yield the carboxylic CH CHCOOH on oxidation ? 3 | CH3 (a) 2-methyl propan-1-ol (b) 2-methyl propan-2-ol (c) Propan-2-ol (d) Butan-1-ol Acetylene reacts cuprous chloride to form : (a) H CuCl 2 (c) Cu H 2 2 with (b) Cu C 2 2 2 ammonical

acid

56.

(d) CuH

57.

Which is the most stable alkene ? CH3 (a) C==CH CH3 CH3 (b) CH CH== CH 3 2 CH3 (c) (d) CH3 CH ==CH 2 halides of C== C CH3 2 on Ag O 2 treatment (moist in with a ether), CH3

58.

Alkyl suspension gives : (a) alkanal (c) alkanes

(b) alkanol (d) alkoxy alkane thionyl (b) chloride SO , Cl 3 2 2 to

59.

Ethanol reacts with give ethyl chloride and : (a) SO , HCl 3 (c) SO , HCl 2

(d) S, SO

60.

When acetamide is hydrolysed by boiling with acid, the product formed is : (a) acetamide (b) acetic acid (c) ethyl amine (d) C H OH 2 5 118

----------------------- Page 118----------------------Mock 61. Test3 (Chemistry)

The electrolytic bath used in gold plating of copper articles, contains : (a) KCN + AuCl3 (b) CuSO4 (c) AuCl3 (d) molten gold The pair likely hydrogen bonding : (a) CH COOH and CH COOCH 3 (b) SiH4 (c) H O (d) and SiCl4 and H O 2 2 2 3 to form 3 the strongest 3

62.

HCOOH and CH COOH

63.

If a 6.84% (wt/vol.) solution of cane sugar (mol. wt = 342) is isotonic

with

1.52%

(wt/vol.) solution of thiocarbamide, then the molecular weight of thiocarbamide is : (a) 180 (b) 60 (c) 76 (d) 152 64. An element forms an oxide, in which the oxygen is 20% of the oxide by weight, the equivalent weight of the element will be : (a) 128 (b) 60 (c) 40 (d) 32 How many gram of I2 solution which requires Na S O to react with it 2 2 3 2 S O + I S O 2 3 2 4 (a) 11.4 g (c) 0.588 g 66. The equilibrium system, can be studied by (a) phase rule (c) both (a) and (b) : (b) distribution law (d) none of these are present in a 40 ml of 0.11 N 2 + 2I 6 (b) 2.54 g (d) 12.7 g in a heterogeneous :

given

65.

67.

Friedel Crafts reaction using CH COCl/AlCl is not successful in case 3 3 of : (a) C H CH 6 5 (c) C H OH 6 5 (b) 3 C H NO 6 5 6 in the He+

(d) C H 6 transition

68.

The

optical

spectrum from n = 4 to n = 2 corresponds to which transition in the hydrogen spectrum ? (a) n = 2 to n = 1 (b) n = 3 to n = 1 (c) n = 3 to n = 2 (d) n = 4 to n = 2 69. The volume of ammonia obtained by the combination of 10 ml of N and 30 ml of 2 H2 is : (a) 10 ml (c) 40 ml (b) 30 ml (d) 20 ml 119 ----------------------- Page 119----------------------Mock Test3 (Chemistry)

70.

The half life period of a radio element is 140 days. After 420 days, 1 g of the element will be reduced to : (a) 1/4 g (b) 1/8 g (c) 1/16 g (d) 1/2 g The oxidation K Fe(CN) is : 4 6 (a) + 3 (c) + 2 number of Fe

active

71.

in

(b) + 6 (d) + 4 heat kcal will be : of combustion hence, the (b) 26.0 kcals (d) +94.0 kcals heat of of

72.

The standard carbon is 94.0 formation of CO2 (a) 46.0 kcals (c) 94.0 kcals

73.

The resistance of 0.01 N solution of an electrolyte was found to be 210 ohm at 298 K, using a conductivity cell of cell 1 constant 0.66 cm . The equivalent conductance of solution is : 2 2 1 (a) 3.14 mho cm eq 1 2 1 (b) 314.28 mho cm eq 2 1 (c) 31.4 mho cm eq 2 1 (d) 314.28 mho cm eq Isotopes have : (a) difference in neutrons (b) difference in protons (c) difference in electrons (d) no difference 75% of a first order reaction was found to complete in 32 minutes. When 50% of the same reaction complete ? (a) 4 minutes (b) 8 minutes (c) 16 minutes (d) 24 minutes n and and (a) (b) (c) (d) and l values of an orbital A are 3 2 and of another orbital B 0. The energy of : B is more than A A is more than B A and B are of some energy none of the above of methane (CH ) was weighed. 4 was then pushed again weighed out and the when filled with are 5 will

74.

75.

76.

77.

A flask Methane flask

oxygen at the same temperature pressure. The mass of oxygen would be : (a) the same as the methane (b) half of the methane (c) double of that of methane (d) negligible in comparison to methane

and

that

of 120

----------------------- Page 120----------------------Mock 78. Test3 (Chemistry) 2 The conjugate acid of HPO4 (a) H PO 3 3 (c) PO3 4 is : H PO 3 4 (d) H PO2 2 4 a reaction is (b)

79.

Equilibrium constant of related to : (a) temperature (b) standard free energy change G (c) free energy change (d) none of the above The velocity possessed gaseous molecule is : (a) most probable velocity (b) average velocity (c) rms velocity (d) none of the above by

80.

majority

of

121 ----------------------- Page 121----------------------Answer Sheet (Mock Test-3) (CHEMISTRY) 41. 46. 51. 56. 61. 66. 71. 76. (b) (a) (b) (b) (a) (c) (c) (a) Hints & 41. 42. 47. 52. 57. 52. 67. 72. 77. (a) (d) (d) (c) (d) (b) (c) (c) Solution of 2 colour being due to traces of cobalt and other metals. ns (Chemistry) 3 Mock Test3 Solutio 43. 48. 53. 58. 63. 68. 73. 78. (d) (d) (c) (d) (c) (a) (d) (d) 44. 49. 54. 59. 64. 69. 74. 79. (b) (a) (c) (c) (d) (d) (a) (b) 45. 50. 55. 60. 65. 70. 75. 80. (b) (d) (a) (b) (c) (b) (c) (a)

Sapphire is a natural crystalline form blue, transparent corundum (Al O ), the

42. 43.

Metallic character increases group so bismuth is most metallic.

down

the were and

The discovery of these elements made later on, however Mandeleef left blank spaces in periodic table predicted their propery. Na S + I 2 + Na SO 2 2 3 Na S O 2 2 3 Sodium thio sulphate of with thio Na S 2 (aq )

44.

A Na SO 2

mixture is 3

of

solution treated

+ 2NaI and

calculated

quantity of iodine to give sodium sulphate. This is used as an antichlor. 45.

Wrought or malleable iron is the purest form of iron. It contains 0.1 to 0.25 percent carbon. The molecular weight of D O is 20. 2 The products to react CH CH 3 number of different substitution possible when ethane is allowed with bromine in sunlight are 9. Cl, CH CHCl , 2 3 2 2 2 3 O3 to form HgO which for the CH CCl , 3 CH Cl.CCl , 2 CHCl CCl , 2

46. 47.

2 3 3 3

CH ClCH Cl, 2 CH ClCHCl , 2 CHCl CHCl , 2 CCl CCl 3 48. 49.

Hg reacts with sticks on walls. Ammonium detection of PO4

molybdate 3 and As

is used 3+ .

122 ----------------------- Page 122-----------------------

Mock 50. 51.

Test3

Solutions

(Chemistry)

H F being weak acid is slightly ionised. 2 2 Glucose isomers. Glucose fructose formula for both and fructose are has aldehyde group and has keto group. is C H O . 6 12 6 functional Molecular

52.

C /C = 1.66 for rare gases and thus rare P V gases are monoatomic in nature. reduction of

53.

Stephans alkyl cyanide

reaction is the with SnCl and HCl 2 SnCl2 + HCl

RCN Alkyl cyanide

RCHO + NH Cl 4 Alkyl aldehyde + SnCl4

54.

N, N-dimethyl aniline is a tertiary amine and has no replaceable H-atom. C H N 6 5 CH 3 CH3 [O]

55.

CH CHCH OH CH CH.COOH 3 2 | CH3 2-methyl propanol-1

3 | CH3 2-methyl propanoic acid

56.

HC CH + Cu Cl Acetylene 2 2 cuprous Chloride

+ 2NH OH 4

CuC CCu + 2NH Cl + 2H O 4 Cuprous acetylide (red ppt) CH3 57. CH3 C==C CH3 CH3 , is the most stable. 2

Most substituted alkenes are most stable. 58. RX + Ag O 2 Alkyl (ether) halide ROR + 2AgX di alkyl ether (Alkoxy alkane)

59.

Ethanol reacts with thionyl chloride to give ethyl chloride, sulphur dioxide, and hydrogenchloride gas. C H OH + SO Cl C H Cl + SO 2 5 2 2 2 5 2 Ethanol Thionyl Ethyl Sulphur chloride chloride dioxide + HCl Hydrogenchloride this is the best method for the preparation of alkyl chloride because SO2 gases are removed leaving behind pure R-Cl. and HCl 123

----------------------- Page 123----------------------Mock 60. Test3 Solutions (Chemistry)

When acetamide is hydrolysed by boiling with acid, the product formed is acetic acid. CH CONH 3 acetamide HOH CH COOH 2 H+ 3 acetic acid together 4 complex salt releases gold slowly giving co-herent gold film on the article gold plated. to be

61.

KCN and AuCl3 combine forming a complex salt K[Au(CN) ]. This

62. 63.

Dimerisation occurs in carboxylic which indicates strong Hbonding. For two isotonic C non-electrolytic = C . 1 2 = m = 1.52 1000

acids solution of

6.84 1000 342 100

m 100 1.52 342 100 1000 6.84 1000 100

= 76

64.

Equivalents of

metal = 80 = E E = 8 80 8 20 20

equivalents of oxygen

= 32 Na S O 2 2

65.

milli

eq. of I 2 w

milli

eq. of

254/2 weight of 66. 67.

1000 = 40 0.11
I2 = 0.588 g

Both phase rule and distribution law are applied to heterogenous systems. Friedel CH COCl/AlCl 3 case of Crafts is 3 C H NO 6 5 because 2 Crafts 124 NO 2 not reaction successful in using the

deactivate the ring for Friedel reaction (Electrophilic substitution).

----------------------- Page 124----------------------Mock 68. For transition in He (Z = 2) 1 2 n2 1 (1) 16 hydrogen (2) Test3 Solutions (Chemistry) + 1 = R Z2 1 2 n1 1 2 1 = R (2) 22 42 = 4R For transition in 3

1 2 1 1 = R 1 2 2 n1 n2 On equating (1) and (2) R 3 = R 1 1

4 3 = 4 2 n1 1

2 n1 1

2 n2

2 n2 So the transition is from n = 2 to n = 1 69. N and H 2 2 combine in 1 : 3 ratio forming 2 3 420 70. n = = 3 140 Here, N0 = 1 g We know

mole of NH .

1n 2 3 = 1 1 = 1 2 1 Nt = g 8
N = N t 0 of

1 8

71.

Let x be the oxidation number K Fe(CN) 4 6 (4 1) + x + 6(1) = 0 4 + x 6 = 0 x = +2 Given C + O 2 CO ; 2

Fe

in

72.

H = 94.0 kcals The equation combustion of carbon formation of CO . 73. = K V 1 = R 1 = 210 represents as well as heat of 2 heat of

l a

0.66 10,0000
2 1

= 314.28 mho cm

eq

125 ----------------------- Page 125----------------------Mock 74. Test3 Solutions difference 2.303 75. t = k 2.303 32 = 32 = k 2.303 k . log 25 . log 4 100 . log 50 log 2 0.04332 2.303 t = 0.04332 0.6932 = 0.04332 76. A is 3d and B is 5 and thus lower lower energy level. is 5s, (n + l) for both value of n decides = 16 minutes . log (a x) 100 (i) (Chemistry) in the a number

Isotopes have of neutrons.

Also,

k = 0.04332 2.303 t = k 2.303 t =

0.3010

77.

Under similar conditions of temperature and pressure Moles of methane = moles of oxygen x g = 16 w = w g 32 x 32 16 w = 2x thus the mass of oxygen double of the mass (x) of methane. (w) will be

78.

Conjugate acid = base + H+ 2 = HPO 4 + H + = H PO 2 2 4

79.

Standard free energy change (G) equilibrium constant (K) each other as follows. G = 2.303 RT log K

and are

related

to

80.

Most probable velocity is the velocity possessed by maximum number of molecules of the gas at a given temperature. It is denoted by . and is given by the expression 2RT = M 126

----------------------- Page 126----------------------ENGLISH (Mock Test-3) Directions (for Q. 136 to Q. 140) : Read the passage and answer the questions : Mock Test3 (English) The emotional appeal of imperialism never completely stilled the British conscience. Homever, liberal thinkers throughout the nineteenth century argued that democracy was incompatible with the maintenance of authoritarian rule over foreign people. To think imperially was to think in terms of restrictive and protective measures; in defiance of the revealed truths of classical economics. Thus, when the British government took over responsibility for India from the East India Company in 1858, many politicians were conscious of saddling Britain with a heavy burden. In the first seventy years of the nineteenth century, enlightened British liberals looked forward to the day when India would stand on its own feet. Even in the heyday of colonialism British radicals continued to protest that self-proclaimed imperialists, however honourable their motives, would place fait accomply before the country and commit blunders of incalculable consequency. 136. What was the attitude of liberals towards the British and colonial policy ? (a) one of repeated protests (b) one of total indifference (c) one of active co-operation (d) one of only verbal co-operation Which class of British society was a force the British imperialist

137.

behind the imperialist Britain ? (a) common class (c) labour class 138.

foreign

policy

of

(b) middle class (d) aristocracy

Which class of the British society had a vested interest in imperial expansion ? (a) middle class (b) officer class (c) labour class (d) aristocracy Give the name of the supreme tactician of the Indian liberation movement : (a) Mrs. Annie Besant (b) Lokmanya Balgangadhar Tilak (c) Mahatma Gandhi (d) enlightened British liberal themselves 127

139.

----------------------- Page 127----------------------Mock 140. Test3 (English)

What do you think were the revealed truths of classical economics ? (a) clamping of artificial restrictions on foreign trade (b) laissez faire and free trade (c) allowing only subsistence wages to the workers (d) wholesale nationalization of the means of production Directions (for Q. 141 to Q. 144) : Choose the correct synonym from the given from each set. words

141.

Dignity : (a) character (c) nobleness Repute : (a) ridiculous (c) discredit Let : (a) pupil (c) documents Feign : (a) gross (c) pretend

(b) level (d) class (b) esteem (d) humiliated (b) smooth (d) to permit (b) wicked (d) sympathy for each

142.

143.

144.

Directions (for Q. 145 to Q. 148) : Choose the antonym from the given words set. 145. Jolly : (a) cheerful (c) serious

(b) blissful (d) fun

146.

Blunt : (a) sharp (c) frank Embrace : (a) accept (c) ban Be-like : (a) disparage (c) impossible

(b) dull (d) rude (b) hug (d) adopt (b) underrade (d) dwarf

147.

148.

Directions (for Q. 149 to 150) : Choose the incorrect word in the given sentences. 149. She comes to me each (a) comes (c) day day : (b) she (d) each : (b) some (d) havent 128 ----------------------- Page 128----------------------Answer Sheet (Mock Test-3) (ENGLISH) 136. 141. 146. (a) (c) (a) 137. 142. 147. (d) (b) (c) Solution British Mock Test3 Soluti 138. 143 148. (b) (d) (c) 139. 144. 149. (c) (c) (d) 140. 145. 150 (b) (c) (b)

150.

I havent some spare pen (a) I (c) spare

Hints & 136.

According to the author attitude of the British liberals towards the imperialist and colonial policy was one of the repeated protests. ons (English) 137. According to the author (self-proclaimed imperialism) was a force behind the imperalist foreign Britain. According to the passage officer class of the British society has a vested in imperial expansion. The supreme liberation Gandhi. tactician movement of was

aristocracy policy of

138.

interest the Indian Mahatma

139

140.

According to passage truths of classical economics are laissez fair and free trade.

141.

Dignity means nobleness means inspires. Therefore, option synonym.

excellence a character (c) is correct

and which

142.

Repute means reputation and esteem means honour. Therefore, option correct synonym. Let means to allow and means give the permission. option (d) is correct synonym.

(b)

is

143.

to permit Therefore, sense

144.

Feign means bad or indifferent and pretend means excuse. Therefore, option (c) is correct synonym. Jolly means happy or Serious means sad or gloomy. Therefore, option (c) is correct. Blunt means pointless or not sharp but Sharp means edged Therefore, option (a) is correct. Embrace means clasps or in means mercy

145.

but

146.

pointed. arms debar. or 129

147.

receive egarly but ban Therefore, option (c) is correct.

----------------------- Page 129----------------------Mock 148. Test3 Solutions (English)

Be-like means possibly but impossible means not possible. Therefore, option (c) is correct. Instead of each every should be used.

149. 150.

Instead of some any should be used. 130

----------------------- Page 130----------------------MATHEMATICS (Mock Test-3) x If y = sin x + e , then sin x ex (a) x 2 (cos x + e ) sin x ex (c) x 2 (cos x + e ) x 1 (d) ( sin x + e ) (b) x 3 2 d x 2 dy hematics) sin x ex is equal to : Mock Test3 (Mat

81.

(cos x +e ) 82. If 2 ax + 1 , then 2 (a) 2b 2 (c) b 83. If 2ac ac 2 4b 2ac 2 (d) b 4ac (b) the value of (a + b + c) + bx + c = 0, are of the form the roots of the equation and 1 2 is :

sin + cosec = 2, then 2 2 sin + cosec is equal to : (a) 1 (b) 2 (c) 4 (d) none of these Newtons method for finding the root of the equation f (x) = 0 is : f (x ) n (a) xn + 1 = xn f (x ) n f (x ) n (b) xn + 1 = xn + f (x ) n f (x ) n (c) xn + 1 = xn f (x ) n (d) 1 none of these dx x to : (b) 2 (d) tan 2 and B = 0 equals : 0 (a) 0 0 0 0 (b) 0 6 4 3 0 0 1 1 e 1 + e 3 , then AB

84.

85.

x is equal 0 e + e (a) 4 1 e 1 (c) tan e + 1 0 2 Let A =

86.

0 (c) 0 87. If the x2 bx

4 0 roots 1

0 (d) 0 of such 1 (b) c a b (d) a + b the that

6 0 equation + = 0,

= are ax c + 1 then the value of is : a + b (a) a b (c) c

131

----------------------- Page 131----------------------Mock 88. Test3 (Mathematics)

If A is any matrix, then : (a) A matrix is a set of numbers (b) A matrix is a number (c) A matrix is an numbers in rectangular array (d) A matrix is a set One a b b c c d d a (a) a + b + (c) 4abc factor of c d d a is : a b b c c + d

arrangement

of

89.

the

determinant

(b) abcd (d) 4 (a + b + c + d) equation

90.

Solution of the differential dy mx + ay = e is : dx (a) (a + m) y = emx + cex (b) y = emx + cex (c) (a + m) y = emx + c (d) yeax = memx + c

91.

Let A = {1, 2, 3, 4}, B = {2, 3, 4, 5, 6}, A B is equal to : (a) {1} (b) {5, 6} (c) {1, 2, 3} (d) {2, 3, 4} When the length of the shadow of the

then

92.

pole is equal to then the elevation is : (a) 75 (c) 45 dx 93. 3 x 1 x 1 (a) 3 2 (b) 3 log log

the height of of the source (b) 60 (d) 30

the pole, of light

is equal to : 1 + c

3 1 x 1 + c 3 1 x

3 1 x 3 (c) 3 log 3 + c 1 x + 3 1 1 (d) 3 log 94. 1 x 3 + 1 (b) 2 < x < 2 (d) 8 < x < 2 132 ----------------------- Page 132----------------------Mock 95. Test3 (Mathematics) terms mean is 14 and 1 x3 1 + c

If |x + 5 | < 3, then : (a) 8 < x < 8 (c) 2 < x < 8

The mean of first three mean of next two terms is 18. The of all five terms is : (a) 15.0 (c) 15.2

(b) 15.6 (d) 14.5

96.

The probability that at least one of the events A and B occurs is 0.6. If A and B occur simultaneously with probability __ __ 0.2, then P (A) + P (B) is : (a) 1.4 (b) 1.2 (c) 0.8 (d) 0.4 A force is a : (a) localised vector (c) zero vector (b) free vector (d) unit vector

97.

1 98. The sum of the series 2 terms is : 1 (a) 2 3 (c) 2 99. The probability that 1 5 (b) 6 (d) 1 a +

1 + 3

1 + to 9 6

man

will

live

another 10 years is

and the probability 4 that his wife will live another 10 years is 1 . Then the probability that both will be 3 alive in 10 years is : 11 7 (a) (b) 12 12 5 1 (c) (d) 12 2 100. A and A , 1 2 G and G and two H.Ms H and H are 1 2 inserted between any two number, 1 1 H1 + H2 equals : A + A 1 2 (a) (b) G G A 1 2 1 1 (c) G1 + G2 (d) A1 Two A.M.s two 1 then G G 1 2 + A 1 1 + A2 2 1 G.M.s 2

101.

A man is known to speak truth 3 out of 4 times. He throws a die and reports that it is a six. The probability that it is actually a six is : 1 1 3 (a) (b) (c) (d) 2 3 8 1 1 + 3 definite sum if : (a) |x | > 1 (c) |x | = 1 3x + 3x2 (b) 1 + 3x3 |x | < 1 1 (d) |x | = 3 1 + The series

3 7 has a

102.

133

----------------------- Page 133----------------------Mock 103. Test3 (Mathematics)

The equation of smallest degree with real co-efficients having 2 + 3i as one of the roots is : (a) x2 4x + 13 = 0 (b) x2 + 5x + 6 = 0 2 (c) x 1 2x + 1 = 0 1 + 6 Cr (a) 4 (c) 5 Cr = 4 Cr (b) 2 (d) none of these not a multiple of xn in of the 3, then expansion the of 1 , then r equals : (d) x 2 + 2x + 1 = 0

104.

If 5

105.

If n is co-efficient

2 loge (1 + x + x ) is : 2 (a) n 2 (c) n 106. If |x | < 1, then the

1 (b) n 1 (d) n co-efficient 2 + x (b) n + 2 (d) n a b 1 b c 1 of xn 3 + ...) in 2 is :

the expansion (1 + x + x (a) n + 1 (c) n 1 1 1 1 a b c b c , then a

c a 1 equals :

107.

If D =

(a) D (c) 0 108. Given 2 y = 1 + b + b fractions. equals : 2 x (a) (c) x y xy + y 2

(b) D (d) none of these x = 1 + a + a2 + + , where a, b are proper 2 2 Then 1 + ab + a b + and

x + y (b) x y (d) none of these

x + y 1 109. The set of all points for 2 x f (x) = x e strictly increases is : (a) (0, 2) (b) (2, ) (c) (2, 0) (d) ( , ) If (a) (b) (c) (d) sin = sin , and cos = cos , then : = 2n + , n is any integer = 2n + , n is natural number = = 134 ----------------------- Page 134----------------------Mock 4 (x 111. x) 5 x Test3 1 4 dx is equal to : 5 1 3 x 5 1 3 x 5 1 3 x 4 + c 4 + c 4 + c (Mathematics) which

110.

4 (a) 15 1 + (b) 15 1 (c) (d) 112. 15 4 1 + 4

none of these value is : 2 (b) 1 (d) none of these AC , where k is a scalar, then are coincident have the same magnitude are coplanar are collinear term in the : of f (x) = sin x in

Minimum x 2 (a) 1 (c) 0 If AB = k (a) A, B, C (b) AB , AC (c) A, B, C (d) A, B, C

113.

114.

The middle 1 + 110 is : x 10 1 (a) C5 5

expansion

of

10 (b) C6

1 6

x 10 (c) 115. C5 x The value of cos 12 + cos 84 + cos 156 + cos 132 is : 1 1 (a) (b) (c) 1 8 2 2 2 The circle x + y + 4x 7y + 12 = 0 intercept on y-axis is equal to : (a) 7 (b) 4 (c) 3 In an equilateral triangle, the ratio of the incircle, circumcircle and excircle are : (a) 1 : 3 : 2 (b) 1 : 1 : 1 (c) 2 : 3 : 4 (d) 1 : 2 : 3 cos2 (x + h) cos2 x 118. lim h h 0 (a) 2 sin x (c) sin 2x 1 5 10 (d) C6 x 1 6

1 (d) 2 cuts (d) 1

116.

117.

is equal to : (b) sin x cos x (d) cos2 x 135

----------------------- Page 135----------------------Mock 119. If Test3 (Mathematics) for same

x . a = 0, x . b = 0, x . c = 0, non-zero vectors x , then [a b (a) true (b) false (c) cannot say anything (d) nothing can be said The number of permutations different things, taken r at a which P (r n p) particular things will never occure is : (a) P (n p, r) P (n, n p) (b) P (n, r) P (n, p) (c) P (n, r) P (n, p) (d) P (n p, r) 2 cos 0 3 (a) 4 (b) 2 (c) 2 2 t dt is : The value of

c ] = 0 is :

120.

of time,

n in

121.

4 (d)

122.

The vertices of a triangle A (0, 0), B (0, 2) and C (2, 0). The distance between circumcentre and orthocentre is : 1 (a) 2 (b) 2 (c) 0 (d) none of these Consider y2 + 4ax = 0, the equation where a > 0. of a Which

are

123.

parabola of the

following is false ? (a) tangent at the vertex is x = 0 (b) directrix of the parabola is x = a (c) focus of the parabola is at (a, 0) (d) vertex of the parabola is at the origin 124. The direction cosines of a line inclined with co-ordinate axes are : 1 1 1 (a) < , , > (b) < 0, 1, 0 > 3 3 3 (c) < 1, 0, 0, > (d) < 1, 1, 1 > The of 3, (a) (c) sum of the digits in all numbers formed with 4, 5, 6 taken all at a time 144 (b) 108 (d) the unit the help is : 432 18 place of equally

125.

136 ----------------------- Page 136----------------------Answer Sheet (Mock Test-3) (MATHEMATICS) 81. (b) 86. (a) 91. (d) 96. (b) 101. (c) 106. (a) 111. (b) 116. (d) 121. (d) Hints & 81. ns (Mathematics) 82. (d) 87. (d) 92. (c) 97. (a) 102. (a) 107. (b) 112. (b) 117. (d) 122. (a) Solution y = sin x + ex Mock Test3 Solutio 83. (b) 88. (c) 93. (d) 98. (c) 103. (a) 108. (c) 113. (d) 118. (c) 123. (c) 84. 89. 94. 99. 104. 109. 114. 119. 124. (a) (a) (d) (d) (b) (a) (a) (a) (a) 85. 90. 95. 100. 105. 110. 115. 120. 125. (c) (a) (b) (a) (d) (a) (b) (d) (c)

dy dx dx dy 2 d x 2 dy = (cos x + e ) (ex sin x) = = cos x + ex 1 1 = cos x + e

...(1)

x [ sin x + e ] x 2

dx dy

x 2 (cos x + e ) (ex sin x) = x 3 (cos x + e ) sin x ex = x 3 (cosx + e ) .

1 x cos x + e

137 ----------------------- Page 137----------------------Mock 82. Test3 Solutions (Mathematics)

Equation

is ax2 + bx + c = 0 + 1 and 1 + 1 2 + ( 1) 2 2 1 = 2 a b ...(1) 2 1 = a b + 1 = a b is given as

Sum of roots

Product of roots . 1 2 + 1 = a c

+ =

c a c = a c = a

(1 + ) ( 1) + 1 1 Putting value (1), we get

a + a = c c (a c) = (c + a) a + c = c a of from = c a (2) in equation c + a ...(2)

c + a 2 2 c a

1 b = a

c + a2 c + a c a c a 2 (c + a)2 (c a)2 = 2 2 2 (c + a) (c a ) 2 [a2 + c2 + 2ac] (c2 + a2 2ac) a2 + c2 + 2ac c2 + a2 2 (a 2a2 + 2ac 2 (a + c) + c 2

b a b = a b = a + 4ac = 2a2 + 2ac a b

+ 2ac) + 4ac

2 (a + c) + 4ac = 2b (a + c) (a + c)2 + 2b (a + c) + 4ac = 0 a (a + c)2 + 2b (a + c) + b2 = b2 4ac 2 + c 2 2 2 + 2ac + 2ba + 2bc + b = b 4ac (a + b + c)2 = b2 4ac. 138

----------------------- Page 138----------------------Mock 83. Test3 Solutions 2 (sin + cosec ) = sin (Mathematics) 2 + cosec 2 + 2 sin cosec 2 (sin + cosec ) = sin . . . cosec = 2 2 = sin 2 2 + cosec + 2 . . [ . sin + cosec = 2 2 + cosec f (xn) 84. xn + 1 = xn f (x ) n 1 85. 0 e x + e 1 = 0 1 = 0 Put ex = t 1 + e ex dx =dt e 1 1 + t 0 . . [ . e = 1 = t, when x = 0 x = 1] 1 and e e = tan 1 t 1 = tan e tan 1 e 1 1 1 1 = t when dt 2 x e + 1 e ex dx 2x dx x dx x = 2. 2 + cosec 2 + 2 1 sin

is given]

2 sin

= tan

. 1 + e 1 y = tan 3 0 1 x y 1 + xy

. . 1 . tan x tan 0 86. AB = 0 0 = 0 x2 bx 87. = 0 1 2 2 3 0 0 .

ax c + 1 ( + 1) x2 ( + 1) bx = ( 1) ax ( + 1) x2 x [ + b + a] Since + = 0 So, + b + a = 0 (a + b) = a b a b = . a + b ( 1) c + ( 1) c = 0

139

----------------------- Page 139----------------------Mock 88. Test3 Solutions (Mathematics) of numbers

A matrix is an arrangement in rectangular array. a b c d b c d a d a b c d a b c a + b + c + d a + b + c + d = a + b + c + d a + b + c + d a d b c

89.

c d a b

d a b c

[C 1 C 1 + C 2 + C 3 1 1 = (a + b + c + d) 1 1 [Taking a + b + c + d common One 90. + C ] 4 b c d a from C ] c d a b

d a b c 1

factor = a + b + c + d.

The equation dy + ay = emx is of the form dx dy + Py = Q dx Here P = a, Q = emx I.F. = e Pdx = e adx = eax now solution is y (I.F.) = {Q (I.F.)} dx + c yeax = (emx . eax) dx + c yeax = e(m + a) x dx + c (a + m) x ax ye = a + m (a + m) yeax = eax . emx + c (a + m) y = emx + cex e + c ...(1)

Which is the required solution. 91. A B = {1, 2, 3, 4} {2, 3, 4, 5, 6} = {2, 3, 4}. 140 ----------------------- Page 140----------------------Mock 92. Test3 Solutions (Mathematics)

Let AB be a pole of height h and BC = x be the shadow of pole and be the angle of elevation of the source of light.

Given : Length of shadow of pole = height of pole now, from figure h tan = x 93. We have dx 3 x 1 x 2 tan = 1 = 45. . . [ . x = h] x = h ...(1)

r r multiplying the N and D by x , we get 2 3 x dx 3 x 1 x 3 putting 1 x = t 3 t = 1 x 3x2 dx = 2t dt 2 x dx = 2 tdt and x 3 . . . The above integral is of form dx 2 2 = (x a ) 1 = 3 log t + 1 t 1 + c 2 3 t dt 2 (1 t ) t 2 dt 3 t2 1 3 = 1 t 2 2

2 1 t 1 log + c 3 2 t + 1

1 log 2a

x a + c x + a

1 = 3 log [Substituting the value of t]

3 1 x 1 3 1 x + 1 + c

141 ----------------------- Page 141----------------------Mock 94. Test3 Solutions (Mathematics)

|x + 5 | < 3 x + 5 < 3 and x 5 < 3 x < 3 5 and 5 3 < x x < 2 and 8 < x 8 < x < 2.

95.

Since mean of first 3 terms = 14 sum of first 3 terms = 14 3 = 42 Also mean of next 2 terms = 18 sum of next 2 terms = 18 2 = 36 Mean of above 5 terms sum of first 3 terms + sum of next 2 terms = 5 42 + 36 = 5 = 5 78 = 15.6

96.

It is given that P (A B) = 0.6 and P (A B) = 0.2 Therefore P (A B) = P (A) + P (B) P (A B) 0.6 = P (A) + P (B) 0.2 P (A) + P (B) = 0.8 __ __ (1 P (A)) + (1 P (B)) = 0.8 __ __ P (A) + P (B) = 2 0.8 __ __ P (A) + P (B) = 1.2

97. 98.

Localised vector. The series 1 1 1 + + + + 9 terms is in A.P. 2 3 6 The sum S of n terms of

an

A.P.

with

n first term a and common difference d n [2 a + (n 1) d] 2 1 Here n = 9, a = 2 1 d = 3 9 Sn = 2 1 = 6 1 6 9 = 6 3 = 3 2 142 ----------------------- Page 142----------------------Mock 99. Test3 Solutions (Mathematics) that a 2 1 3 4 1 ,

is given by Sn =

1 2 . + (9 1) . 2 2 1 + 8 2 1

9 = 9 = 2

Let P (A) P (B) be the probability man, woman less live for another 10 yrs. 1 1 i.e. P (A) = and P (B) = 4 3 __ 1 3 P (A) = 1 = 4 4 __ 1 2 P (B) = 1 = 3 3 The probability that both will be alive in 10 yrs is __ __ __ __ P (A, B) = P (A) P (B) 3 = 4 . 3 2 = 2 1

100.

Let the number S are a, b a, A , A , b are in A.P. 1 2 A1 = a + d A2 = a + 2d ...(1)

b = a + 3d i.e. A1 + A2 = 2a + 3d A1 + A2 = 2a + b a A1 + A2 = a + b ...(2)

a, G , G , b are in G.P. 1 2 2 G1 = ar G2 = ar 3 b = ar G G 1 2 3 = a r = a 2 a = ab 1 , a H1 1 = H2 1 = b 1 H1 + H2 = a 2 = a 1 = = ab A1 + A2 = G G 1 1 1 2 A1 + A2 143 [from (2) and (3)] + a b b + a + b 1 1 = a 2 + 3d 1 a 1 1 + d + a a 1 + 2d a 1 + 3d H1 1 , H2 1 , b 1 = a 1 + 2d 1 + d 1 are in H.P. ...(3) 2 b

H1

+ H2

= G G 1 2

----------------------- Page 143----------------------Mock 101. Test3 Solutions (Mathematics)

Let E , E and A be the events defined as 1 2 follows E1 = six occurs E2 = six does not occurs A = the man reports that it is a six. We have, 1 P (E ) = 1 6 , P (E ) = 2 6 . . . 1 Now, P (A E ) = Probability that the man 1 reports that there is a six on the die given then 6 has occured on die = Probability the man speaks truth 3 = 4 A P E2 = Probability that the man reports that there is six is on die given has not occured on die = Probability that the man does not speak truth 3 1 = 1 = 4 4 We have to find P (E A) 1 6 1 = 6 5 5

that

six

i.e.

the die that

probability given that there is six. By E 1 P A =

that there is six on the the man has reported Bayes rule, we have P (E ) P (A E ) 1 1

P (E ) P (A E ) + P (E ) P (A E ) 1 1 2

1 6 = 1 6 102.

3 4 = 5 3 . 8

3 + 4

1 4

The sum of series 1 + 3 3x 1 + 3x2 1 3 = 1 1 x a . . . Since sum S = 1 r is definite 1 < 1 x 1 + 3x3 1 1 . . . a = , r = 3 x 1 +

|x | > 1. 144

----------------------- Page 144----------------------Mock 103. Test3 Solutions (Mathematics)

Since 2 + 3i is a root 2 3i is also a root Hence, required equation is x2 (sum of roots) x + (product of roots) = 0 Sum of roots = 2 + 3i + 2 3i = 4 Product of roots 2 = (2 + 3i) (2 3i) = 4 9i 2 So, equation is x 1 1 + 5 C r (5 r) ! r ! 5 ! + 6 ! n 6 C r = 4 C r (6 r) ! r ! = 4 ! n ! (4 r) ! r ! 4x + 13 = 0. 1 = 13 . . 2 [ . i = 1]

104.

. . . (5 r) ! 5 ! (5 r) (4 r) ! 5 (4 !) + + 6 ! (6 r) ! =

Cr = (n r) ! r !

(4 r) ! 4 ! (6 r) (5 r) (4 r) ! 6 . 5 (4 !) (4 r) ! = 4 !

(5 r) 5 2 r +

(6 r) (5 r) = 1 30 2 = 30 2 = 30

30 6r + 30 6r 5r + r 30 6r + 30 11r + r 2 r 17r + 30 = 0

15r 2r + 30 = 0 r = 2

(r 15) (r 2) = 0 . . [ . r = 15 is not possible] 105. 2 loge (1 + x + x ) = loge (1 + x + x ) 1 x3 = loge 1 x 3 = loge (1 x ) loge (1 x) 3 x6 x9 = x x 2 3 = x 3 2 x6 x9 + x + 3

2 (1 x) (1 x)

x2 2 x2 + + 2

xn n xn

Since n is not a

multiple of 3.

n co-efficient of x in first bracket = 0 as all powers of x are multiple of 3 also co-efficient of xn in second

bracket

1 = n 1 Hence, required co-efficient = n 145 ----------------------- Page 145----------------------Mock 106. Test3 Solutions 2 (1 + x + x = ((1 x) = (1 x) 2 3 = 1 + 2x + 3x + 4x + + (n + 1) x Co-efficient of xn = (n + 1). a b c b c a 1 1 1 [interchanging a b 1 = b c 1 c a 1 rows and columns] a b c 2 1 2 = (1) 1 1 [interchanging C and C ] 1 1 = 1 1 2 108. x = 1 + a + a + a b c a b c 1 1 1 b c a 3 b c a 2 b c = D. a n + + x 1 2 ) 2 3 + ...) [using Binomial] (Mathematics) 2 .

107.

= (1)

[interchanging C and C ]

1 x = 1 a [sum of a = 1, r = a] infinite G.P. 2 y = 1 + b + b 1 y = 1 b x (1 a) = 1 and y (1 b) = 1 x ax = 1 and y by = 1 a = x 1 and b = y 1 x y 1 1 a = 1 and b = 1 x y 2 2 1 + ab + a b 1 = 1 ab 1 1 = 1 1 1 1 1 = 1 1 1 1 + x y y 1 = 1 + x = x + y 1 146 ----------------------- Page 146----------------------Mock 109. Test3 Solutions (Mathematics) 2 x f (x) = x e x 2 x f (x) = 2xe x e = xe x (2 x) increasing f (x) > 0 y xy . 1 xy 1 1 x xy + . . [ . a = 1, r = ab] + = a 1 r, where

Since f(x) is strictly

110.

xe x (2 x) > 0 x (2 x) > 0 x > 0, 2 x > 0 x > 0, 2 > x 0 < x < 2 x (0, 2).

sin = sin , cos = cos sin sin = cos cos tan = tan = 2n + , where n is any integer. 4 (x 1 4 x) 5 x 3 [x (x = 1 4 x = x 3 (x = x = 1 1 4 x 1 4 1 dx 3 x 1 Putting 1 x (3) x4 dx = dt 3 4 x dx 4 x 1 Given integral = 3 t 1 4 dt = 3 1 dt dx = dt 3 = t 1) 19 4 1 4 dx = (x 1 4 1) 4 3 4 dx x . x 3 (x 5 1)] 5 x 3 1) dx dx 1 4 dx 1 4

111.

5 4 1 t = 3 5 4 = 4 1 15 5 4 1 + c 3 x 147 ----------------------- Page 147----------------------Mock 112. Test3 Solutions (Mathematics) + c

f (x) = sin x For maxima f (x) = 0 f (x) = cos x or minima cos x = 0, x = 2 , 2 f (x) = sin x > 0 when x = 2 it has minimum value at x = 2 Now, f = + sin 2 = sin 2 = 1 Hence minimum value of f (x) = sin x in 2 x 2 10 1 Here n = 10 which th So, 10 + 1 2 i.e. 6 is the middle 10 T6 = T5 + 1 = C5 (1) term 5 10 5 1 is an even number x is 1 2

113.

A, B, C are

collinear if AB = k AC .

114.

The

given

expression

is 1 +

x . . [ . Tr + 1 = n Cr (a) n r b r

where r = 5, n = 10, a = 1, b = 1 x] 10 T6 = C5 x 148 ----------------------- Page 148----------------------Mock 115. Test3 Solutions (Mathematics) 1 5

cos 12 + cos 84 + cos 156 + cos 132 = (cos 12 + cos 132) + (cos 156 + cos 84) = 2 cos 12 + 132 cos 12 132 2 2 2 A B cos 2 2

+ 2 cos 156 + 84 cos 156 84 2 . . . cos A + cos B = 2 cos A + B

= [2 cos 72 cos ( 60)] + [2 cos 120 cos 36] = 2 cos 72 cos 60 + 2 cos 120 cos 36 . . [ . cos ( ) = cos ] = 2 5 1 1 + 2 1 5 + 1 4 2 2 4 5 1 = 4 = 4 2 = 4 116. = 2 1 5 + 1 4

5 1 5 1

The length of the intercepts made by the 2 2 circle x + y + 2gx + 2fy + c = 0 with y-axis is 2f 2 c Now circle is

2 2 x + y + 4x 7y + 12 = 0 Comparing it with equation of circle, we get 7 f = , c = 12 2 2 f 2 c 2 7 = 2 12 2 49 = 2 12 4 49 48 = 2 4 2 = = 1 2

...(1)

149 ----------------------- Page 149----------------------Mock 117. The Test3 Solutions (Mathematics) 3 area of equilateral = 4 where a is side Also inradius r = s circum radius abc R = 4 and exradius r1 = s a (3 4) a2 = a 2 = 2 3 a = 3 a2 = s = 2 3a2 2 4 3a 3 a = 3 a ...(2) of a + a + a = 2 a = 2 3 ...(1) 3a 2 a .

Hence

a a r : R : r1 = 2 3 : 3 : = 1 : 2 : 3

3 2 a

118.

We know f (x + h) f (x) f (x) = lim h h 0 2 If we put f (x) = cos f (x) = lim h h 0 2 cos x sin x 2 cos = lim h h 0 2 cos sin 2x = lim h h 0 x . a = 0 x is perpendicular to a x . b = 0 x is perpendicular to b x . c = 0 x is perpendicular to c a , b , c are coplanar. are perpendicular, hence (x + h) cos 2 x . (x + h) cos 2 x x cos2 (x + h) cos2 x

119.

they

[a b c ] = 0 = true we have of such to

120.

Since p particular things out of n different things is never taken. So, determine the number of ways in which r places can be filled with (n p) distinct things. Clearly, the number arrangements is n pPr

= P (n p, r), where P (r n p) 150 ----------------------- Page 150-----------------------

Mock 121. 0

Test3 2 cos 1

Solutions t dt 2

(Mathematics)

= 0 2 =

2 cos 2 t dt + 1 cos 2 t dt 1 t sin 2 0 2 sin 2 t 1 2

= 2 0 0 2 4 = 122. .

We have ABC is a right angled with right angle at A orthocentre = A (0, 0) . . [ . The perpendiculars from opposite sides meet at point 0, 0] Also circumcentre 2 + 0 = Distance orthocentre 2 , 2 between circumcentre 2 = 2. and 0 + 2 i.e. (1, 1)

A, B, C

on

2 = (1 0) + (1 0) 2 123. We have y + 4ax = 0 2 y parabola (y2 + 4ax) = 0 focus of parabola 2 y = 4ax is ( a, 0) Hence (a, 0) is false. = 4ax

151 ----------------------- Page 151-----------------------

Mock 124.

Test3

Solutions

(Mathematics) by inclined line on to

Let , , be angles made co-ordinate axes. Since line is equally co-ordinate axes. = = cos = cos = cos 2 + cos 2 + cos + cos 2 Since cos = cos = cos = 3 1 The direction cosines are 3 cos 2 cos = 2 2

...(1) = 1 ...(2)

2 Also cos + cos from (1) and (2) 2 cos

= 1 = 1 1 3 1 cos = 3 1 1 , . 3 3 3 , is 3 ! unit 3, then place 1

125.

When number at unit place other 3 numbers can be arranged in ways. The sum of the digits in when 3 is their at unit place = 3 ! 3 ...(1) Similarly, the sum of the digits in .... when 4 is .... = 3 ! 4 5 .... = 3 ! 5 6 .... = 3 ! 6 from (1), (2), (3), (4) The sum of the digits in the unit place of all numbers formed with the help 3, 4, 5, 6 taken all at a time is (3 ! 3) + (3 ! 4) + (3 ! 5) + (3 ! 6) = 3 ! (3 + 4 + 5 + 6) = 6 18 = 108.

...(2) ...(3) ...(4)

of

152

----------------------- Page 152----------------------PHYSICS (Mock Test-3) 1. When a certain metallic surface illuminated with monochromatic light of wavelength , the stopping potential for photoelectric current is 3V , when 0 same surface is illuminated with light of wavelength 2 , the stopping potential is V . The threshold wavelength 0 surface for photoelectric effect is : ysics) 4 (a) 6 (c) 4 2. (b) 3 (d) 8 of is the

for

this Mock Test3 (Ph

How are the velocity of light in medium cm, polarising angle ip and velocity light in air are related ? c (a) cm = cot i (b) cm = c tan ip p (c) c = cm sin ip (d) cm = c cot ip

3.

In a microscope, the focal length of objective and eye lenses are 1.6 cm and 2.5 cm respectively. The distance between the two lenses is 21.1 cm. If the final image is formed at infinity, the distance between the object and the objective lens is : (a) 1.65 cm (b) 0.7 cm (c) 1.75 cm (d) 1.8 cm A convex glass lens ( = 1.5) has a focal length of 8 cm when placed in air. The focal length of the lens when placed in water ( = 4 3) will be : (a) 16 cm (b) 32 cm (c) 4 cm (d) 8 cm In LCR circuit, the capacitance is changed from C to 4C. For the same frequency, the inductance should be : L L (a) (b) 2 4 (c) 4L (d) 2L Which of the following radiation the part of electromagnetic spectrum ? (a) ry (b) Cathode rays (c) ry (d) ry resonant

4.

5.

6.

form

7.

Two bulbs have the following ratings (i) 40W, 220V (ii) 40W, 110V. The ratio of their resistances is : (a) 1 : 2 (b) 4 : 1 (c) 1 : 4 (d) 2 : 1 153

----------------------- Page 153----------------------Mock 8. Test3 (Physics) be of

How much should a 40W lamp lowered to make double the illumination on an object which is at a distance 40 cm directly under it ? (a) 11.72 cm (b) 21.72 cm (c) 3.172 cm (d) 31.72 cm A lens of focal length 50 cm forms the image of a distance object subtends an angle 1 milliradian lens. What is the size of the image ? (a) 0.5 mm (b) 0.1 mm (c) 5 mm (d) 1 mm In a resonance tube experiment, the first and second resonance with a tuning fork were observed at 16.7 cm and 51.7 cm respectively. The wavelength as deduced from the data is : (a) 70.0 cm (b) 66.8 cm (c) 68.8 cm (d) none of these The dispersive powers of the materials of the two lenses are in the ratio 4 : 3. If the achromatic combination of these two lenses in contact is a convex lens of focal length 60 cm, then the focal length of the component lenses are : (a) 20 cm and 25 cm (b) 15 cm and + 120 cm (c) + 15 cm and 20 cm (d) 20 cm and + 25 cm An AC generator produced voltage E = 170 sin 377t volt, where in second. The frequency of A.C. voltage is : (a) 110 Hz (b) 60 Hz (c) 230 Hz (d) 50 Hz In Bohrs model hydrogen single electron rotates around the nucleus in a circle of radius of making 6.6 1015 an

9.

which at the

10.

given

11.

12.

output t is

13.

atom, 53 1012

the m each

revolutions

second. What is the magnetic field at the

centre ? (a) 7 testa (c) 12.5 testa 14. A coil 2 0.001 m the coil having is is 100 free to placed

(b) 9 testa (d) 11 testa turns and area of to of is 154

rotate about an axis, perpendicular 2 magnetic field of 1.0 wb m . If the coil is rotated rapidly through an angle 180, how much charge will flow through the coil ? The resistance of the coil 10 : (a) 0.02 coulomb (b) 0.04 coulomb (c) 0.08 coulomb (d) 0.07 coulomb

----------------------- Page 154----------------------Mock 15. Test3 (Physics) are placed on the square. If the force and that between 12

Three equal charges three corners of a between q and q is F 1 2 q and q is F 1 3 F12 F13 is : 13

, the ratio of magnitudes

1 (a) 2 (c) 2 16. (b) 2 1 (d) 2 Iron and silicon wires are heated from 30C to 50C. The correct statement that : (a) resistance of iron wire decreases (b) resistance of iron wire increases and that of silicon wire decreases (c) resistance of iron wire decreases and that of silicon wire increases (d) resistance of both wires increases Two spheres each of mass M and radius R 2 are conected with a massless rod of length 2R as shown in figure, what will be the moment of inertia of the system about an axis passing through the centre of one of the sphere and perpendicular to the rod ? 5 (a) 4 21 MR 2 2 (b) 2 2 5 MR 2 2 is

17.

(c) 5 18.

MR

(d) 5

MR

A beam of monochromatic blue light of wavelength 420 nm in air travels in water ( = 4 3). Its wavelength in water be : (a) 315 nm (b) 400 nm (c) 280 nm (d) 560 nm The waves produced in a vibrating spring with fixed ends are of the type : (a) progressive longitudinal (b) progressive transverse (c) stationary transverse (d) stationary longitudinal

will

19.

155 ----------------------- Page 155----------------------Mock 20. The amount 2 of Test3 energy (Physics) flowing through

a cm at a point per second is called : (a) intensity (b) pitch (c) loudness (d) quality 21. The surface temperature estimated by measuring the wavelength for which the intensity of radiation is maximum, using : (a) intensity data (b) Plancks law (c) Stefans law (d) Weins displacement law of its sun emitted is

22.

When diameter of the aperture of the objective of an astronomical telescope is increased, its : (a) magnifying power and resolving power both are increased (b) magnifying power remains the same but resolving power is increased (c) magnifying power and resolving power both are decreased (d) magnifying power is increased and resolving power is decreased If the spinning speed of the increased, then weight of the body at the equator : (a) does not change (b) doubles (c) decreases (d) increases A closed bottle containing opened on the moon. What will happen ? (a) The water will remain as before (b) The water will decomposses into H2 earth is

23.

24.

water

is

and O2 (c) The water will freeze (d) The water will boil 25. To raise the temperature of 2 mol of an ideal gas at constant pressure from 30C to 35C, if 70 cal of heat is required. What is the amount of heat required (in calories) to raise the temperature of same gas through the same range (30C to 35C) at constant volume (R = 2 cal mol k) : (a) 45 cal (b) 75 cal (c) 50 cal (d) 25 cal A freshly prepared radioactive sample of half life 2 hour emits radiations intensity that is 64 times the permissible safe limit. The minimum time after which it would be possible to work safely with this surface is : (a) 24 hour (b) 10 hour (c) 12 hour (d) 36 hour of

26.

156 ----------------------- Page 156----------------------Mock 27. Test3 (Physics)

Two rods having thermal conductivity in the ratio of 5 : 3 and having equal length and equal cross-section are joined face to face. If temperature of free-end of first rod is 100C and the free end of second rod is 20C, then calculate temperature of junction : (a) 50C (c) 85C (b) 70C (d) 100C enters a 3 T applied electron. 31 kg 19 C)

the

28.

An electron is accelerated by a potential difference of 12000 volt. It then uniform magnetic field of 10 perpendicularly to the path Find the radius of path : (Given of

mass of electron = 9 10

and charge of electron = 1.6 10 (a) 3.67 m (b) 36.7 m (c) 36.7 cm (d) 3.67 cm 29. Two satellites having masses 100 kg and 400 kg respectively, go around the earth in circular orbit of the same radius. The ratio of the orbital speed of 100 kg to that of 400 kg satellite is : (a) 4 : 1 (b) 1 : 4 (c) 1 : 2 (d) 1 : 1

30.

A man weighing twice as much as a boy and the man takes half time as much time as the boy takes to climb a height. What is the ratio of power spent by the man to that spent by the boy ? (a) 4 : 1 (b) 1 : 2 (c) 5 : 7 (d) 2 : 3

particular

31.

A conducting square loop of side l and resistance R moves in its own plane with uniform velocity v perpendicular to one of its sides. A magnetic field H, constant in time and and space pointing perpendicular and into the plane of the loop every where. The current induced in the loop is : H v (a) anti-clockwise R H v (b) clockwise R 2H v (c) anti-clockwise R (d) zero 157

----------------------- Page 157----------------------Mock 32. Test3 (Physics) wave nature of light

Evidence for the cannot be obtained from : (a) diffraction (b) interference (c) reflection (d) dopplers effect

33.

At what speed the particle of m should move, so that the mass becomes double ? (a) 3c (c) 2c (b)

rest 3 c

mass

2 (d) c 2

34.

In a thermodynamic process, pressure of a fixed mass of a gas is changed in such a manner that the gas molecules give out 30 J of heat and 10 J of work is done on the gas. If the internal energy of the gas was 40 J, then the final internal energy will be : (a) 20 J (b) 20 J (c) 80 J (d) zero In an experiment, the saturation in the

35.

plate current in a diode is observed 240 V. But a student still wants to increase the plate current. It can be done if the : (a) filament current is increased (b) filament current is decreased (c) plate voltage is decreased (d) plate voltage is increaed further 36.

at

The order of magnitude of the density of nuclear matter is : 34 3 4 3 (a) 10 kg m (b) 10 kg m (c) 1017 kg m3 (d) 10 15 kg m3 The ionisation energy of hydrogen atom is 13.6 eV. Following Bohrs theory, the energy corresponding to a transition between the 3rd and 4th orbit is : (a) 1.51 eV (b) 0.85 eV (c) 3.4 eV (d) 0.66 eV Heavy water is used as moderator in a nuclear reactor. The function of the moderator is : (a) to cool the reactor (b) to slow down the neutrons (c) to absorb neutrons and stop chain reaction (d) to control the energy released in the reactor 158

37.

38.

----------------------- Page 158----------------------Mock 39. Test3 (Physics) in

If Y is the Youngs modulus of the wire, the work done per unit volume stretching it is : 1 2 1 2 (a) Y (strain) (b) (strain) 2 2 1 2 1 2 (c) Y (strain) (d) Y (stress) 2 2 A missile is fired with a velocity less than escape velocity. The sum of its kinetic energy and potential energy is : (a) negative (b) positive (c) zero (d) positive or negative will depend upon its initial velocity

40.

159 ----------------------- Page 159-----------------------

Answer Sheet (Mock Test-3) (PHYSICS) 1. 6. 11. 16. 21. 26. 31. 36. (c) (a) (c) (b) (d) (c) (d) (c) Hints & 1. 2. 7. 12. 17. 22. 27. 32. 37. (b) (b) (b) (c) (a) (b) (c) (d) 3. 8. 13. 18. 23. 28. 33. 38. (c) (a) (c) (c) (c) (c) (b) (b) 4. 9. 14. 19. 24. 29. 34. 39. (b) (a) (a) (c) (d) (d) (a) (c) 5. 10. 15. 20. 25. 30. 35. 40. (b) (a) (c) (a) (c) (a) (a) (a)

Solution photoelectric Mock Test3 Soluti

Einsteins equation for effect is given by ons (Physics) h = h0 + eVs where Vs = stopping potential In the first case, hc = h + e (3V ) 0 0 In second case hc = h0 + eV0 2 Subtracting eq. (ii) from eq. (i), we get hc = e(2V ) 2 0 hc V0 = 4 e Suppose then from is the threshold wavelength, 0

(i)

(ii)

(iii)

Hence, 2.

eq. (i) and (iii), we have hc = hc + 3e hc 0 4e hc hc 3 hc = 0 + 4 1 1 3 = + 0 4 1 1 3 = 0 4 1 = 4 0 = 4

From Brewsters Law

tan ip = c = cm c m = c tan i p where ip = polarising angle c = velocity fo light in vacuum and cm = velocity of light in medium 160 ----------------------- Page 160----------------------Mock 3. Test3 Solutions (Physics)

Given : Focal length of objective lens f = 1.6 cm o Focal length of eye-piece fe = 2.5 cm Distance between two lenses L = v + u = 21.1 cm o e and final image distance ve = Now for eye-piece, from lens formula 1 = f e or or Again 1 = fo 1.6 u o uo v o u e 1 + v e 1 = (L v ) o 1 + 1

fe = L vo = L f e = 21.1 2.5 = 18.6

for objective lens, 1 + vo 1 = u o = 1.75 cm 1 + 18.6 1 1 fo 1 = uo 1 + L fe 1

4.

From lens maker formula 1 1 f = ( 1) R1 R2 1 f 1

( 1) fw a g 1 Therefore, = fa w g 1 fw 1.5 1 0.5 = = 8 1.5 1.125 1 1 4 3 fw 8 = 0.125 length of lens when 0.5 = 4

Hence, fw = 4 8 = 32 cm Hence, the focal immersed in water is = 32 cm 5.

In an LCR circuit, resonance frequency is given by 1 f = 2LC 1 or f LC f Hence, f Here : f = f, C = 4C from eq (1) or 1 = 4LC L L = 4 161 = LC LC LC ...(1)

----------------------- Page 161----------------------Mock 6. Test3 Solutions (Physics)

From the given options, (gamma) rays form the part of electromagnetic spectrum. 2 V

7.

Resistance of the bulb

is R = P

Where P = power of the bulb V = voltage across the bulb R = resistance of the bulb 2 V1

R1 = P1 = 220 220 40 2 V2 = 1210

and

R2 = P2 = R1 110 110 40 1210 = R2 R : R 1 Inverse square 302.5 = 4 : 1 2 law E 2 r 2 E1 r2 = E2 r2 r2 1 E1 = r1 E2 ...(1) formula, 1 = 4 = 302.5

Hence, So, 8. Using we have

Hence,

or

Here : r1 = 40 cm, E2 = 2E1 From eq (1) we get r2 40 = 2E1 40 2 2 which lamp be angle lens = 40 r2 = 2 = E1

= 28.28 cm Hence, the distance by lowered = 40 28.28 = 11.72 cm 9.

If length of image is I, then subtended by the object at the angle subtended by the image at the lens. i.e., 1 10 3 = f I = 50 1 10 3 I

or

or or

= 0.05 cm = 0.5 mm 162

----------------------- Page 162----------------------Mock 10. Test3 Solutions (Physics)

By using end correction v n1 = 4(l1 + e) 3v and as 3v So, 4(l2 + e) Therefore, n n1 = 4 (l1 + ) or = 4 (16.7 + 0.8) = 70 cm of 1 2 1 Here : 2 From So, f2 or Now combined is given by f1 = 3 focal length of the lens eq (1) and (2) f1 = 3 4 f2 4 = 3 4 lenses ...(1) = 4(l1 + e) l2 + e = 3 (l1 + e) 51.7 + e = 3 (16.7 + e) 51.7 + e = 50.1 + 3e) 2e = 1.6 1.6 e = 2 n2 = 4(l2 + e) n1 = n2 = n v

= 0.8

11.

For achromatic combination f1 = f2

1 = F 1 or 60 1 or 60 or and 4f2 = 60 = =

1 + f1 3

1 f2 1 + 4f2 f2 . . [ . F = 60 cm]

3 + 4 4f2

f2 = 15 cm 4 f1 = 3 4 f2

15 3 Hence, f1 = 20 cm Hence, the focal length lenses are + 15 cm and 20 cm.


f1 =

of

component 163

----------------------- Page 163----------------------Mock 12. Test3 Solutions is E = 170 sin 377t E = E sin t 0 Comparing (2) we get or or or Hence, 13. The frequency of (1) with standard (1) (2) equation (Physics)

Given equation

= 377 2 = 377 377 f = 2 377 f = 2 3.14 = 60.03

f 60 Hz the electron = 6.6 1015 /sec charge on electron (q)

current i =

time (t) = charge (q) frequency of electron () = 1.6 10 19

6.6 10
3

15

Now

= 1.06 10 magnetic field at the B =

amp centre is

0 i 2R 4 10 7

1.06 10
12

2 53 10 = 12.5 tesla 14.

The flux linked with the coil when the plane of the coil is perpendicular to the magnetic field is given by = nAB cos = nAB cos 0 = nAB Now the flux change in rotating the coil by 180 is d = nAB ( nAB) = 2nAB d Again the induced charge = is R = R 2 100 0.001 1 10 = 0.02 coulomb 164 2nAB [ = 0]

----------------------- Page 164----------------------Mock 15. Test3 Solutions side of a 1 F12 = 40 1 (Physics) square q q 1 2 a2 is a. Then

Suppose the AC = 2a

and

q q 1 3 F13 = 40 (2a)2 F 12 1 q q 1 2 (2a)2

so,

F13 = 40 = 2

a2

q1 q3 [as q 1 = q

40
2 = q ] 3

16.

Iron is semiconductor. directly temperature semiconductor to its resistivity increases decreases temperature. The moment of

a metal and silicon is a The resistivity of metal is proportional to its absolute but the resistivity of is inversely proportional absolute temperature. Hence, of metallic wire like iron and resistivity of silicon wire with the increase of inertia 2 = 5 = 4MR 21 = 5 MR M R + M (2R) 2 2 1 2 + MR 5 2 of 2 2 + 5 2 M 2 R the system is 2

17.

18.

We know So,

that c m = cm = light c wavelength 1 ...(1) in

where m = of medium cm = velocity of light in medium = wavelength of light in air = 420 nm = refractive index of medium = 4 3 c = velocity of light in air = 3 10 3 = 420 or m = 4 4 3 8 m s

From eq. (1) m

420 = 315 nm
165

----------------------- Page 165----------------------Mock 19. Test3 Solutions (Physics)

Since, the two ends of a spring are fixed, the wave produced in the string will be

reflected at two ends with a phase change and also interfere. As produce stationary transverse waves. 20. The amount of energy flowing a unit area per sec at a intensity. From Weins displacement law m

result

this

through point is

called 1

21.

, T sun can be

surface temperature of calculated by measuring the wavelength m for which the intensity of its emitted radiation is maximum. 22.

2 Since, intensity (diameter of the lens) , therefore, diameter of aperture of the objective lens should be large, so, that more and more rays may enter in the telescope and hence, the intensity of image gets increased. Also resolving power is also increased. Using the relation for acceleration due to gravity at the equator g = g R 2 e = spinning speed of earth Now observing from the given expression when spinning speed or angular speed of earth is increased, the value of g will decrease and hence, the weight of the body will decrease.

23.

24.

The boiling point of water is lowered on moon because at the moon, the pressure is very low and so, the water will boil. Let the specific heats of a gas at constant pressure and volume be C and C . p At constant pressure Q = mCp t or or C p We know that C C v . . [ . Therefore, volume heat Q = mCv t required R = 2 cal mol K] at constant = C R v p = 7 2 = 5 cal mol K 70 = 2 Cp (35 30) = 7 cal mol K v

25.

= 2 5 (35 30) = 50 cal 166 ----------------------- Page 166----------------------Mock 26. Given : Using N0 decay law Test3 N = 64 N 1t T1 2 N0 = 2 t 2 1 1 = 64 2 6 t 2 1 1 = 2 2 t 27. In steady state Q = t Q = t = temperature of interface K 1 ( 1 ) = K 2 l ( ) 2 l K A ( ) 2 2 K A ( 1 ) 1 = 6 2 t = 12 hour Solutions 1 , T1 2 = 2 hour (Physics)

5 (100 ) = 3 ( 20) 500 5 = 3 60 8 = 560 560 Hence, = 8 28. Kinetic energy 1 or 2 of electron is K.E. = eV 2 mv = eV = 70C

2eV and m or r = eB 1 = B 1 = e m 2mV v = m mv r = eB 2eV

2 9 1031 12000 3 19 10 1.6 10 = 0.367 m = 36.7 cm

167

----------------------- Page 167----------------------Mock 29. Test3 Solutions (Physics)

The velocity

of satellite is given by GMe ves = R e (i)

From equation (i), it is clear that escape velocity is independent of mass of satellite. Hence, the two satellites have same velocity. v1 1 i.e. = v2 1 v 1 30. : v 2 = : 1 : 1

The power is given by the relation P = W = mgh t t P 1 So, P 2 Here m1 = mass of the m2 = mass t1 = time t2 = time = t m gh 1 t 2 ...(i) m gh 2

1 man = 2m of the boy = m taken by man = t 2 taken by boy = t

P1 From equarion (i) P2 P1 = P2 Therefore, 31. P : P 1 2 =

2m t 2 4 1 = 4 : 1

t m

Induced e.m.f. in the loop will be zero due to the reason that the magnetic flux linked with the coil remains same and hence, induced current in the loop is zero. The wave nature obtained from reflection of Using the relation m0 m = 2 1 v 2 c m0 = rest mass of particle c = velocity of light in Given : m = 2m0 m0 2m0 = 2 1 v 2 c 2 v or 1 2 c 2 v or 2 c 2 v or 2 c 3 or v = 2 c = 4 168 3 = 1 4 1 = 4 1 vacuum of light light. can not be

32. 33.

----------------------- Page 168----------------------Mock Test3 Solutions (Physics)

34.

Here : Q = 30 J, W = 10 J Q = W + U or 30 = 10 + U or U = 30 + 10 or U = 20 or U2 U1 = 20 U2 = U1 20 U2 = 40 20 = 20 J The plate current in a diode saturates only when all the electrons are attracted by the plate. To increase the plate current further the filament current should be increaed, so that more electrons are produced. The magnitude of the density of nuclear matter is of the order of 1017 kg m3 The ionisation energy of hydrogen atom in nth orbit is given by Rhc 13.6 En = 2 = 2 n n In the 3rd orbit, E3 = In the 4th E4 = Hence, energy transition from orbit, 13.6 42 = 13.6 16 = 0.85 in 13.6 2 3 = 1.51

35.

36. 37.

radiated by electrons 3rd orbit to 4th orbit is E = E4 E3 = 0.85 ( 1.51) = 0.85 + 1.51 = 0.66 eV

38. 39.

Heavy water is used as nuclear reactor to slow down the neutrons. The work stretching is 1 W = 2 1 = 2 1 = 2 Y (strain) of conservation of energy, done per unit

moderator volume

in in

stress strain (Y strain) strain . 2 . . Y = strain stress

40.

From the law

total energy in the universe constant. As the body moves kinetic energy is converted into potential energy and potential energy will be zero at infinity. Therefore, total energy will be negative before reaching to infinity y at any point. ----------------------- Page 169----------------------REASONING (Mock Test-3) Directions (for Q. 126 to Q. 128) : Choose the correct relation. soning) 126. HARD : SOFT :: LIGHT : ? (a) DARK (c) WEIGHT FIRE : HOT :: ICE : ? (a) COLD (c) WATER PANKOJ : OBMLNK :: SAROD : ? (a) TBOPC (c) RSBPC Directions (for the odd man out. 129. 130. 131. 132. (a) Pen (c) Pair (a) Kid (c) Kitten (a) Bundle (c) Pack (a) And (c) For Directions (for the correct statements. 133. Q. 133 to relation Q. 129 to (b) LITTLE (d) HEAVY (b) WOOD (d) ROAD (b) RBQPC (d) RBPQC Q. 132)

remains up, its

169

Mock Test3 (Rea

127.

128.

: Find

(b) Bird (d) Chair (b) Lamb (d) Horse (b) Packet (d) Glass (b) Now (d) If Q. 135) : Pick from the following

A is the son of B. C is the uncle of A and D is the wife of B. What is D to A ? (a) daughter (b) niece (c) son (d) mother Kavita is the sister of Daya and Daya is the brother of Sangeeta. Sandeep is the uncle of Dayas son. What is the relation between kavita to Sangeeta ? (a) sister (b) brother (c) mother (d) son B is the father of C, but C is not the son

134.

135.

of B. What is C to (a) daughter (c) father

B ? (b) son (d) uncle 170

----------------------- Page 170----------------------Answer Sheet (Mock Test-3) (REASONING) 126. 131. (d) (d) 127. 132. (a) (b) Solution 128. 133. (b) (d) 129. 134. (c) (a) 130 135 (d) (a)

Hints & 126.

Hard is antonym of soft while heavy is the antonym of light. Therefore, option (d) is correct. Mock Test3 Solutio ns (Reasoning) 127. 128. We know that fire is hot and ice is cold. Therefore, option (a) is correct. The coded previous and respective alphabet letter of P and Therefore, option (b) alphabets are placed in successive order of their as O is the previous B is the successive of A. is correct. same

129.

We know that Pen, Bird and Chair stand for one thing while Pair for two things. Therefore, pair is the odd man, i.e., option (c) is correct. It is known represents young a young ones, it horse is the odd that Kid, Lamb and ones while horse is not is adult ones. Therefore, man. Kitten

130.

131.

The Bundle, Packet and Pack represent the collection of things, while glass is not representing the same. Therefore, glass is odd man. And, For and If are the conjuctions while Now is an adverb. Therefore, odd man is Now, i.e., option (b) is correct. According to question, B and D husband and wife and A is the son of B. Therefore, D is the mother of A. are the

132.

133.

134.

We know that Kavita, Daya and Sangeeta are the brother and sisters. Kavita is the sister of Sangeeta. As per question C is not the son of B but they have the relation of

Therefore,

135.

father

and

children. of B.

Therefore,

is

the

daughter 171

----------------------- Page 171----------------------CHEMISTRY (Mock Test-4) 41. Which relative molecule, ion : sequence bond super oxide (a) O2 > O2 > O2 (c) O2 < O2 > O2 42. 1 2 2 (b) correctly describes a strength of oxygen ion and peroxide Mock Test4 (Che O2 > O2 < O2 2 2

mistry)

(d) O2 < O2 < O2

mole of CH contains : 4 of carbon CH4

(a) 3.0 g

(b) 1.81 1023 molecules of (c) 4 g atoms of hydrogen (d) 6.02 1023 atoms of H 43.

In two H-atoms A and B the electrons move around the nucleus in circular orbit of radius r and 4r respectively. The ratio of the times taken by them to complete one revolution is : (a) 1 : 2 (b) 2 : 1 (c) 1 : 4 (d) 1 : 8 The table given below lists the dissociation energy (Ediss) for co-valent bonds formed between C and atoms A, B, D, E Bond CA CB CD CE Which of the atoms has smallest size ? (a) E (b) B (c) D (d) A E(diss) (kcal mol1) 240 382 276 486 bond single

44.

45.

The atom

ionisation is 495.8

energy of kJ mol1.

gaseous Na The lowest

possible frequency of a Na atom is : 12 (a) 1.24 10 14 (c) 4.76 10 46.

light that can ionise 1 s 1 s (b) 3.15 10 14 1 s 15 1 s

(d) 1.24 10

A sudden large jump between the values of second and third ionization energy of an element would be associated with the electronic configuration : 2 2 6 2 3 (a) 1s , 2s 2p , 3s , 3p 2 2 6 2 (b) 1s , 2s 2p , 3s 2 2 6 1 (c) 1s , 2s 2p , 3s 2 2 6 2 1 (d) 1s , 2s 2p , 3s 3p 172

----------------------- Page 172----------------------Mock 47. Test4 (Chemistry) pair of

A bonded molecule MX3 is T-shaped. The number of non-bonding electrons is : (a) 1 (b) 3 (c) 2 (d) can be predicted only number of M is known The value of HOH is 109 kcal mol1.

if

atomic

48.

Then formation of one mole of water in gaseous state from H (g) and accompained by : (a) absorption of 218 kcal of energy (b) release of 218 kcal of energy (c) release of 109 kcal of energy (d) unpredictable 49. One of the following is Bronsted acid but not a Bronsted base : (a) H O (b) NH 2 3 (c) H S (d) HCO 2 3 Calculate the heat of following reaction Na(s) + 0.5 Cl (g) NaCl(s) ; H ? 2 Given H (g) + Cl (g) 2HCl, 2 2 H = 44 kcal 2Na(s) + 2HCl(g) 2NaCl(s) + H (g),

O (g)

is

50.

2 H = 152 kcal (a) 54 kcal (c) 196 kcal 51. (b) 98 (d) 108 kcal kcal

The equilibrium constant for the reaction NH NO (s) N (g) + 2H O(g) 4 2 2 2 is given by : [N ][H O]2 2 2 (a) [NH NO ] 4 [NH NO ] 4 (c) 2 [N ][H O] 2 2 [N ][2H O]2 2 2 (b) 2 2 [NH NO ] 4 2 (d) [N ][H O] 2 2 2 2

52.

In a closed vessel of nitrogen and b made to react according to reaction N2 at equilibrium, 2x obtained then : 2 4x (a) Kc =

of volume V, a moles moles of oxygen are to give nitric oxide, + O2 2NO. If of moles of NO are 1 . V . V

(a x)(b x) 2 4x (b) Kc = (a x)(b x) 2 x (c) Kc = (a x)(b x) 2 4x (d) Kc = (a x)(b x)

. V

173 ----------------------- Page 173----------------------Mock 53. Test4 (Chemistry)

The molality of 1 litre solution of 93% H SO (w/v) having density 1.84 g/ml 2 4 is : (a) 1.043 m (b) 0.143 m

(c) 10.43 54.

(d) none of these

Reaction that take place at graphite anode in dry cell is : 2 (a) Mn + 2e Mn(s) (b) Mn(s) Mn+ + e + 1.5 V 2+ (c) Zn (d) (s) + 2e Zn(s) Zn radius ion of Na+ is Zn(s) +2 + 2e is 181 95 pm pm. and that Predict + co-ordination (a) 8 (c) 4 number of Na : (b) 6 (d) unpredictable of of of the

55.

The Cl

56.

A reaction A B follows a second order kinetics Doubling the concentration A will increase the rate of formation B by a factor of : (a) 1/4 (b) 4 (c) 1/2 (d) 2

57. The diagram given above is pressure composition diagram binary solution of A and B. In the solution A ...... B interactions are : (a) smaller than A .......... B .......... B interactions (b) similar to A .......... A and interactions (c) greater than A ......... A and interactions (d) unpredictable 58. a vapour for and a

B .......... B B ......... B

Calculate the volume occupied by 7.0 g of nitrogen gas at 27C and 750 mm Hg pressure : (a) 62.32 litre (b) 6.232 litre (c) 623.2 litre (d) 0.623 litre 174

----------------------- Page 174----------------------Mock 59. Test4 (Chemistry) only by

235 If 92U is assumed to decay emitting two and one particles. The possible product of decay is : 227 235

(a) 89Ac (c) 89Ac211 60.

(b) 89Ac (d) 89Ac237

The colour of K Cr O changes from red 2 2 7 orange to lemon yellow on with aqueous KOH because of : (a) oxidation of potassium hydroxide to potassium peroxide (b) conversion of dichromate chromate (c) reduction of Cr (VI) to Cr (III) (d) formation of chromium hydroxide treatment

ion

to

61.

Which one of the following metals not be extracted by using Al as a reducing agent ? (a) W from WO3 (b) Mn from Mn O 3 4 (c) Cr from Cr O 2 (d) Na from Na O 2 3

can

62.

The numbers of and particles emitted 238 in the transformation 92U (a) 2, 1 (c) 1, 1 92U (b) 1, 2 (d) 1, 0 angle (b) in H Se 2 hydrides 234 :

63.

The maximum bond of group 16-elements is in : (a) H Te 2 (c) H S 2

(d) H O 2 heating : (b) AgNO3 (d) KNO3

64.

NO2 is obtained by (a) NaNO3 (c) CsNO3 An ingredient of baking (a) sodium bicarbonate (b) sodium carbonate (c) borax (d) sodium chloride Stable compounds are formed by : (a) B (C) Ga

65.

powder is :

66.

in + 1

oxidation (b) Al (d) Tl

state

67.

Which one of the following configurations is correct for alkaline earth elements ? 2 2 3 2

(a) 1s 2s 2p 2 2 6 2 (c) 1s 2s 2p 3s 3p

1s 2s 2p 3s 2 2 6 (d) 1s 2s 2p 175

(b)

----------------------- Page 175----------------------Mock 68. Test4 (Chemistry)

Perchts process is used to prepare : (a) BaCO (b) Li CO 3 2 (c) K CO 2 3 (d) Na CO 2

3 3 albumin and 2.5

69.

The gold numbers of gelatin, and strach are 0.005, 0.15 respectively. Which is the best protective colloid ? (a) Gelatin (b) Albumin (c) Strach (d) None of KO2 + CO2 (gas), Which (a) H2 (b) N2 gas is this ? (c) O2 inter

these (d) CO halogen (d) ClF

70. 71.

Which of the following does not exist ? (a) ClF2 (b) ClF5 (c) ClF3

72.

If a person is injured by the shot of gun and all the bullets could not be removed. It may cause posioning by : (a) Tl (b) Pb (c) As (d) carbon Silicon is the main constituent of : (a) plants (b) rocks (c) alloys (d) animals Which of the Aqua-regia ? (a) 3HCl + HNO3 (c) H PO + H SO 3 4 2 following is known as

73.

74.

(b) 3HNO3 + HCl (d) HCl + CH COOH 4 in 4 3 photography

75.

Silver bromide is used because it is : (a) soluble in NH OH

(b) insoluble in acids (c) photosensitive (d) soluble in hypo-solution 76. The Matte obtained by pyrites with coke mainly : (a) FeS + ZnS smelting and (b) copper sand Cu S + FeS contains

2 (c) CuS + FeS2 77. (d) ZnS + CuS

In the metal carbonyls of general formula M(CO ), where m = metal, x = 4 the metal x is bonded to : (a) oxygen (b) carbon (c) carbon and oxygen (d) C O triple bond 176

----------------------- Page 176----------------------Mock 78. Test4 (Chemistry) following is common

Which one of the example of fibres ? (a) Nylon-66 (c) Buna-S

(b) Nylon-6 (d) Bakelite

79.

Which one of the following species does not exist ? (a) [SnCl ]3 (b) [GeCl ]2 6 6 2 2 (c) [SiCl ] (d) [CCl ] 6 6 (CuSO 4 + NH OH) 4 gives a deep blue

80.

complex of : (a) cupra ammonium sulphate (b) cupra ammonium hydroxide (c) sodium hexametaphosphate (d) ammonal 177 ----------------------- Page 177----------------------Answer Sheet (Mock Test-4) (CHEMISTRY) 41. 46. 51. 56. 61. 66. 71. 76. (a) (b) (a) (b) (d) (d) (a) (b) Hints & 42. 47. 52. 57. 62. 67. 72. 77. (c) (c) (d) (c) (b) (b) (c) (b) Solution 43. 48. 53. 58. 63. 68. 73. 78. (d) (b) (c) (b) (d) (c) (c) (a) 44. 49. 54. 59. 64. 69. 74. 79. (a) (c) (d) (a) (b) (a) (a) (d) 45. 50. 55. 60. 65. 70. 75. 80. (d) (b) (b) (b) (a) (c) (c) (a)

41.

Bond strength depends on bond order. More is bond order, more is the strength. Bond order of O = 2 2 ns (Chemistry) Bond order of O = 1.5 2 Bond order of O2 = 1.0 2 So order of relative bond strength, is O2 > O2 > O2 42. 2

Mock Test4 Solutio

1 mole of CH contains 1 mole of carbon 4 and 4g atoms (mole) of hydrogen. 1

43.

v r v1 i.e., v2 = r1 two r2

The velocity of the electrons in orbitals (r) and (4r) will be in the ratio of 2 : 1. Their respective circumference are 2r and 8r. The ratio of the times for completing one revolution is 1 : 8. 44. Smaller the atom more effective overlap would be, consequently will be the bond dissociation energy. E 45. E = hv, hence, v = h 495.8 10 6.02 10 = 1.24 10 46. After the removal will acquire configuration. of 23 3 J/atom 34 J sec

the larger

6.62 10
15 sec 1

second electron, it stable noble if it has 3

gas

47. 48.

T-shape geometry is possible bond pair and 2 lone pairs.

In water 2(OH) bonds are formed. So the value of H for the formation one mole of water in gaseous state is

of

H = 2 109 = 218 kcal ----------------------- Page 178----------------------Mock 49. Test4 Solutions (Chemistry) as

178

It can not act as proton acceptor while all other can act as an donar as well acceptor of proton. Equation (i) can be obtained eq. (ii) and (iii). H = 152 + (44) = 152 44 = 98 kcal by adding

50.

51.

The equilibrium constant for the reaction NH NO (s) N (g) + 2H O(g) is 4 2 2 2 by 2 [N ][H O] 2 2 Kc = [NH NO ] 4 2

given

52.

For this reaction N2 a mol (a x) Thus, + O2 b mol (b x) [NO]2 Kc = [N ][O ] 2 2 2 (2x) 2 V = (a x) V 4x = (a x)(b x) . 2 (b x) V 2NO 0 (2x)

53.

93%

H SO 2

(w/v) 4

means

93

g.

of

H SO is present in 100 c.c. of solution. 2 4 Since vol. of solution = 1000 ml Amount of H SO = 930 gper 1000 ml

2 4 Density = 1.84 g/ml M D = V M = D V = 1.84 1000 = 1840 g weight of solvent = 1840 930 = 910 g 930 molality = 98 930000 = 89180 = 10.43 m 54. The reaction which occurs at anode is 2+ Zn(s) Zn 55. Radius ratio lies between 0.414 (aq) + 2e to is 6. 179 ----------------------- Page 179----------------------Mock 56. Test4 Solutions (Chemistry) 0.732. + Hence co-ordination number of Na

1000 910

2 Rate = k [A] when concentration is doubled 2 r = k[(2A) ] = k 4[A]2 The new rate become 4 times.

57.

The curve represents the solution with ve deviations, so in the solution A .......... B interactions are greater than A .......... A and B ........... B interactions. 750 Given P = 750 mm = 760 V = ? w = 7.0 g T = 27C = 27 + 273 = 300 K R = 0.0821, Molecular wt (M) = 28 According to ideal gas equation, w atm

58.

PV = V =

R T M w R T
p M 7 760 0.0821 300 130872 28 750

V = =

21,000 = 6.232 litre 2 235 59. 92U 89Ac decreases atomic mass number by particle increases 2K CrO + H O 2 4 2 lemon yellow (potassium chromate) by 227

Loss of rtic number by 2 and where as emission of atomic number by 1 . 60. K Cr O 2 2 Potassium dichromate (red orange) 61. + 2KOH 7

4,

Sodium metal can not be extracted using Al as reducing agent because Na is more electropositive than aluminium. Change in mass number = 4 a.m.u. Change in atomic number = No change 4 Change particle = 2 particle = in atomic hence, 2 = 1 4 number due particles emitted.

62.

to

63.

As we move down the group, the bond angle decrease due to decrease in bond pair-bond pair repulsion. H O H S H Se H Te 2 2 2 2 Bond angle 104.5 92.2 91 90 180

----------------------- Page 180----------------------Mock Test4 Solutions (Chemistry)

64.

2AgNO3 Silver nitrate

red

2NO2 Nitrogen dioxide

+ 2Ag + O2 Silver Oxygen baking soda is

65.

An ingredient of NaHCO (sodium bicarbonate). 3 Thalium (Tl) shows maximum inert pair effect in group III A.

66. 67.

Alkaline earth metals are s-block, normal elements having ns2 general configuration.

electronic

68.

Perchts Process > In this process CO2 gas is passed in the suspension magnesium carbonate trihydrate solution of potassium chloride forming an insoluble double salt. 2KCl + 3MgCO .3H O + CO 3 2 2 2[MgCO .KHCO .4H O] + MgCl 3 3 (double salt) This double salt is then heated with water under pressure. This salt is decomposed to give water soluble K CO 2 insoluble MgCO . 3 2[MgCO .KHCO .4H O] 3 3 2 H O pressure 2 2

in

of the thus

and 3

water

2MgCO 3 + K CO (aq) + 9H O + CO 2 3 2 69. Gelatin (gold number = 0.005) . . . lesser the gold protective colloid. 4KO 2 Carbondi oxide 71. Halogens i.e., the do not number show of + 2CO 2 2

number

better

the

70.

2K CO + 3O 2 3 Potassium Oxygen carbonate even half covalency filled

atomic

orbitals possible 72.

can

1, 3, 5, 7 etc. it

If a person is injured by shot of gun and all the bullets could not be removed may cause poisoning by arsenic (As). Silicon is the main constituent of alloys. Aqua regia of conc. HCl is the and one mixture of three parts part of HNO .

73. 74.

3 75. Silver halides are used in photography because they are photosensitive in nature and on exposure to light are reduced to metallic silver by reducing agents ferrous oxalate etc. like hydroquinone,

easily mild 181

----------------------- Page 181----------------------Mock 76. Test4 Solutions (Chemistry) with escapes This black occurs coke

Copper pyrites on smelting and sand mainly contain a black mixture of Cu S and a little FeS. (which 2 from oxidation) is obtained. mixture is known as Matte.

77. 78.

Bonding in metal through carbon of carbon monoxide.

carbonyls

Nylon-66 is the common example of fibre. Which is a polyamide. polymer chains are held H-bonds between NH group of one chain O and C group of another chain.

The linear together by

79.

Carbon atom cannot expand its valency shell due to non availability of shell, so [CCl ]2 does not exist. 6 CuSO (aq ) + 2NH OH Cu(OH) 4 4

2d

sub

80.

2 Cupiric hydroxide + (NH ) SO 4 2 4 4 2 4 2

Cu(OH) (s) + 2NH OH + (NH ) SO 2 4

[Cu(NH ) ]SO (aq) + 4H O 3 4 4

Instense blue colour cupra ammonium sulphate 182 ----------------------- Page 182----------------------ENGLISH (Mock Test-4) Directions (for Q. 136 to Q. 140) : Read the following passage carefully and answer the questions given below the passage. Certain words/phrases in the passage are given in bold to locate them while answering some of the questions : Mock Test4 (En glish) A man may usually be known by the books he reads as well as by the company he keeps; for there is a companionship books as well as of men and one should always live in the best company, whether it be of books or of men. A good book may be among the best of friends. It is the same today that it always was and it will never change. It is the most patient and cheerful of companions. It does not turn its back upon us in times of adversity or distress. It always receives us with the kindness; amusing and interesting us in youth, comforting and consoling us in age. 136. of

same

A man may usually be known by the books he reads because : (a) books provids him a lot of knowledge (b) the books he reads affect his thinking and character (c) his selection of books generally reveals his temperament and character (d) his reading habit shows that he is a scholar Which one of the following would be the most suitable title for the passage ? (a) books are useful for our youth (b) books as mans abiding friends (c) books show the readers character (d) the importance of books in old age Which of the following statement is not ture ? (a) good books as well as good always provide the finest company (b) we have sometimes to be patient with a book as it may bore us (c) a good book serves as a permanent friend (d) a good book never betrays us

137.

138.

men

183 ----------------------- Page 183----------------------Mock 139. Test4 (English) is opposite in adversity

Which of the following meaning to the word occurring in the passage ? (a) progress (b) happiness (c) prosperity (d) misfortune

140.

The statement A good book may among the best friend in the middle of the passage, means that, (a) a good book can be included among the best friends of mankind (b) our best friends read the same good books (c) there can not be a better friend than a good book (d) books may be good friends, but not better than good man Directions (for Q. 141 to Q. 145) : In each of the following questions, choose the with opposite meaning to the given word out of the given alternatives : word

be

141.

Minor : (a) heighted (c) tall Former : (a) later (c) primer Lend : (a) borrow (c) pawn Violent : (a) gentle (c) shy Unstable : (a) pure (c) steady

(b) big (d) major (b) latter (d) subsequent (b) take off (d) hire (b) harmless (d) humble (b) stagnant (d0 changing sentences

142.

143.

144.

145.

Directions (for Q. 146 to Q. 150) : Choose the correct word to complete the from the given choice : 146. Neither of the ideas : (a) these (c) his boys

could express .......... (b) there (d) him

147.

.......... goes there ? (a) How (c) Whom

(b) What (d) Their 184

----------------------- Page 184----------------------Mock 148. Test4 (English)

The jury were divided in .......... opinion : (a) this (b) it (c) him (d) their Either watch : (a) his (c) her Ram or Mohan has (b) those (d) its .......... away : (b) its (d) that 185 lost ..........

149.

150.

Here is your book; take (a) this (c) it

----------------------- Page 185----------------------Answer Sheet (Mock Test-4) (ENGLISH) 136. 141. 146. (c) (d) (c) 137. 142. 147. (b) (b) (d) Solution their Mock Test4 Soluti is friends. 138. 143. 148. (b) (a) (d) 139. 144. 149. (c) (a) (a) 140. 145. 150. (a) (c) (c)

Hints & 136.

It is true that the selection of books by the readers generally reveal temperament and character. Option (c) is correct. ons (English) 137. The most suitable title books are mans Option (b) is correct. of the passage abiding

138.

According to passage, the statement We have sometimes to be patient with a book at it may bore us is not true because a book having good knowledge and informations may sometimes bore but it may worth something. Option (b) is correct. The opposite prosperity. According to passage, meaning a good of book adversity can is

139. 140.

be included among the best mankind because it gives company and sometimes suggestions. Option (a) is correct. 141. Minor means smaller or less important but major means greater Therefore, option (d) is correct antonym. or

friends of a good good

full

age.

142.

Former means first of the two things but latter means second of the two things. Therefore, option (a) is correct antonym. Lend take. antonym. to grant Therefore, but borrow option (a) annoyed Therefore, firm means to is correct gentle (a) is steady (c) is 186

143.

144.

Violent means means polite. correct antonym.

but options but option

145.

Unstable means not means firm. Therefore, correct antonym.

----------------------- Page 186----------------------Mock 146. 147. 148. Test4 Solutions (English) possessive as an adjectives, interrogative

His is a singular used with neither. Who pronoun. is used

As jury is used in plural number, it takes plural form of possessive adjective like their. Either is a distributive pronoun singular form of verb and adjective like his. It is a personal pronoun, used for lifeless things lying near you. 187 takes possessive

149.

150.

----------------------- Page 187----------------------MATHEMATICS (Mock Test-4) 81. If 2 x px +36 = 0 value of p are hematics) and : , are the roots 2 + of 2 = 9, then the Mock Test4 (Mat the equation

(a) 8 (c) 6 2r 1 82. If D r = n 2 1 n the value of D is equal r r = 1 (a) + + (b) . 2n + . 3n + . 4n (c) 0 (d) 83. If a, b, b (a) b a (c) b c are in a a = c c b c = c b G.P., then n 3

(b) 9 (d) 3 2 . 3r 1 4 . 5r 1 n 1 5 1 to : , then

: a b (b) b c a b (d) b c = b = a a b

84.

Which of the following matrices does not have an inverse : 1 2 1 (a) 2 1 (b) 2 1 0 2 (c) 2 2 (d) 1

1 2 1 2

85.

The area of parallelogram whose adjacent sides are i 2j + 3k and 2i + j 4k is : (a) 106 (b) 5 6 (c) 10 3 (d) 53

86.

If the co-efficient of correlation between x and y is 0.28, covariance between x and y is 7.6 and variance of x is 3, then the S.D. in y series is : (a) 10.05 (b) 9.05 (c) 10.1 (d) 9.8 If p, q, numbers, p = q r r q r p are then r p is : q negative the distinct real determinant

87.

(a) > 0 (c) < 0

(b) (d)

0 0 188

----------------------- Page 188----------------------Mock 88. lim x 0 1 (a) 4 (c) 1 89. Let = 1 sin 1 (b) 2 (d) 0 sin 1 sin sin . Then 1 1 Test4 (Mathematics) is equal 1 to :

2 (1 cos x) 2 x

lies in the interval : (a) [2, 4] (c) [1, 4] 90. If a , b , c are

(b) [3, 4] (d) none of these

unit vectors b + c such that a (b c ) = , b and c 2 are non-parallel, then the angle between a and b is : (a) (b) 4 2 3 (c) (d) none of these 4 In an examination, a student answer 4 questions out of 6. Questions 1 and 2 are however compulsory. number of ways in which the students can make the choice is : (a) C (6, 2) (b) 6 (c) C (6, 4) (d) 4 If sin A = 3 sin (A + 2B), is equal to : (a) 2 cos B (c) cot B The is : range of the then (b) 2 tan B (d) 2 has The

non-coplanar

91.

to

92.

tan (A + B)

93.

function f (x) = tan 1 x

(a) (c) 94.

, 0 0, 2 , 2 2

(b) , 2 2 (d) R equation

The degree of the differential 2 3 2 2 1 + dy = d y is : dx dx2 (a) 4 (c) 2 th

(b) 3 (d) 1 p term of an A.P. is q and th term is : (b) p + q r (d) p q + r the

95.

If th

the

q term is p, then the r (a) p + q + r (c) q p + r

189 ----------------------- Page 189----------------------Mock 96. Test4 (Mathematics)

The number of ways in which four faces of a tetrahedron can be pointed with four different colours is : (a) 2 ! (b) 4 (c) 1 ! (d) 4 ! sin 1 2x = 2 tan1 x for : 1 + x

97.

(a) |x | 1 (b) x 0 (c) |x | 1 (d) all x R 98. The solution set of (2 cos x 1) (3 + 2 cos x) = 0 in the interval 0 x 2 is : 5 1 3 (a) , , cos 3 3 2 (b) 3 (c) (d) 5 3 3 none of these ,

99. 0 1 (a) log a + log b 1 (c) log a log b 100. (d) log a The number of real roots of the equation 2 2 2 (x 1) + (x 2) + (x 3) = 0 (a) 3 (b) 0 (c) 1 (d) 2 4 + x 2 101. Let f (x) = x continuous at x = 0, equal to : (a) 1 (c) 4 102. Let 1 f (a) (b) (c) (d) f : X Y f f f f be we must have f (0) (b) 1/4 (d) 0 a given function, then , x 0. For f (x) to be log b 1 1 (b) log a + log b (ax bx) dx is equal to :

is :

exists (as f is invertible) if : is onto is one-one but not onto is one-one and onto is one-one 2 log (1 + x + x ) + log (1 x + x ) 2 is sec x cos x

103.

lim x 0 equal to (a) 0 (c) 2 : (b) 1 (d) does not exist

190 ----------------------- Page 190----------------------Mock 104. Test4 (Mathematics) to an

P, Q, R, S are to give lecture audience. The organizer can arrange the order of their presentation in : (a) 256 ways (b) 24 ways (c) 4 ways (d) 12 ways Let a, b, c R and a 0. If is the root of 2 2 a x + bx + c = 0, is root

105.

of

2 2 a x bx c = 0 and 0 < < , then 2 2 equation a x + 2bx + 2c = 0 has a root that always satisfies : + (a) = (b) = 2 (c) < < (d) = 106. If S = n = 1 then S equals : (a) 2e 1 (c) 2e + 1 107. If the equations a + ay z 2x y + az = 0, and consistent, then a is equal (a) 2, 3 (c) 2 3, 2 If (b) 2e (d) none of these = 0, ax + y + 2z = 0 to : (b) 2, 2 (d) 1 3, 2 n Pn nC0 + nC1 + nC2 + + nCn

the

are

108.

a, b, c are different real numbers ai + bj + ck , bi + cj + ak and ci + aj + bk are position vectors of three non-collinear points, then : (a) perpendicular from the origin of the plane of the triangle does not meet it at the centroid (b) triangle ABC is a scalene triangle (c) (i + j + k ) is not equally inclined to three vectors (d) centroid of ABC is a + b + c (i + j + k ) 3 If e, e are the eccentricities of hyperbola 2 2 2 2 x y = 1 and x y = 1, then a2 b2 b2 a2 1 (a) 2 e e (c) e = e + 2 (d) e = e of 1 between x and x 1 , where x > 0, is : n geometric means 1 = 1 (b) ee = 1

109.

110.

The

product

(a) n

(b) n

(c) zero

(d)

1 191

----------------------- Page 191----------------------Mock 111. Test4 (Mathematics) other

The mean deviation from the median is : (a) less than measures from any value (b) greater than that measured from other value (c) maximum if all observations positive (d) equal to that measured from any other value If z1 = 8 + 4i, z2 = 6 + 4i and z z1 arg = , then z satisfies : z z2 4 (a) |z 7i | = 18 (b) |z 4i + 7 | = 8 (c) |z 7 5i | = 2 (d) |z 7 4i | = 1 3 + i 69 , then z 2 (a) 1 (c) i 2 cos (x + h) cos h h 0 (a) sin x cos x 2 (c) cos x (b) 2 sin x (b) 1 (d) i 2 x is equal to : is equal to :

are

112.

113.

If z =

114.

lim

(d) sin 2x of

115.

The

co-efficient of x in the expansion 5 2 c x + is equal to : x (a) 10c (c) 10c3 2 3 (b) 20c (d) 20c3 2 4 + cos

116.

The value of cos

is equal to :

5 3 (a) 4 117. (b) 4 5 (c) 2 5

5 4 (d) 5 the relation

P (A B) = P (A B) if between P (A) and P (B) is : (a) P (A) + P (B) = 2P (A B) (b) P (A) + P (B) = 2P (A) P (A B) (c) P (A) + P (B) = 2P (A) P (B A) (d) none of these The foot of the (1, 2, 3) on y-axis is : (a) (0, 0, 3) (c) (0, 2, 0)

118.

perpendicular (b) (0, 0, 0) (d) (3, 0, 0)

from

192 ----------------------- Page 192----------------------Mock 119. Test4 (Mathematics) is

For a continuous series, the mode computed by the formula : fm fm 1 fm f1 (a) l + or l + fm fm 1 fm + 1 fm f1 f2 fm fm 1 (b) l + c 2fm fm 1 fm + 1 fm f1 or l + 2fm f1 f2 2fm fm 1 (c) l + fm fm 1 fm + 1 fm 1 (d) l + fm fm 1 fm + 1 or l + fm f1 f2

i
2fm f1

c
or l + fm f1 f2 f1

120.

_ If z = z, then : (a) z is purely real (b) z is purely imaginary (c) z is any complex number (d) real part of z = complex part of z The focus 2 x 8x + 2y + 7 = 0 9 (a) 4, 2 of is : (b) (4, 4) the parabola

121.

(c) 4, 122. The

9 2 sum of the n

(d) 0, co-efficients

1 2 in the

expansion of (x + y) is 4096. The greatest co-efficients in the expansion is : (a) 924 (b) 1024 (c) 724 (d) 824 123. The total number of (x + y) is : of 100 terms 100 + (x y) (b) 50 (d) none of these after in the

expansion simplification (a) 51 (c) 202 124.

3 In any ABC, a cos (B C) is equal to : (a) 3abc (c) 3 (a + b + c) 2 (b) ab + bc + ca (d) 0 4 + 3 sin x log 4 + 3 cos x dx is : (b) 2 (c) 2 (d) none of these 193 0

125.

The (a)

value of 0 3

----------------------- Page 193----------------------Answer Sheet (Mock Test-4) (MATHEMATICS) 81. 86. 91. 96. 101. 106. 111. 116. 121. (b) (b) (b) (c) (b) (d) (a) (a) (b) 82. 87. 92. 97. 102. 107. 112. 117. 122. (c) (a) (b) (c) (c) (d) (c) (c) (a) Solution is x2 px + 36 = 0 of roots + product of roots = 36 ...(2) = p ...(1) Mock Test4 Solutions 83. 88. 93. 98. 103. 108. 113. 118. 123. (d) (c) (c) (c) (b) (d) (d) (c) (a) 84. 89. 94. 99. 104. 109. 114. 119. 124. (b) (a) (c) (d) (b) (a) (d) (b) (a) 85. 90. 95. 100. 105. 110. 115. 120. 125. (b) (c) (b) (b) (c) (d) (c) (b) (b)

Hints & 81. Equation (Mathematics) sum

Also ( + )2 2 p

2 +

2 = 9 = 9 [expanding 2 +

[given] 2]

p2 2 36 = 9 2 p 2 = 81 p = 9. n 72 = 9

[from (1) and (2)]

n 82. Dr = r = 1

n 2r 1 r = 1 2n 1 n r 1

n 2 . 3r 1

4 . 5r 1

r = 1

r = 1

3n 1

5n 1

2 = 1 + 2 + 2 + + 2 n = 2 1 2

n 1

Now, 2 r = 1

1 . (2n 1) = (2 1) n r 1 2 . 3 r = 1 2 (3n 1) = (3 1) n r 1 4 . 5 r = 1 4 (5n 1) = (5 1) n Dr = r = 1 n 2 n 2 . = 0 [ n 1 3 n 1 . . R 3 = 4 [1 + 5 + 5 = 2 [1 + 3 + 3

n 1 + + 3 n ]

= 3 2

1 n 1 + + 5 n ]

= 5 1

1 n 5 n 1 194

and R

are identical]

----------------------- Page 194----------------------Mock 83. Test4 Solutions G.P. 2 So, 2 b 2 ab b = ab ac b (a b) = a (b c) a a b = b b c 1 84. 2 2 1 = 2 2 = 0 . . . A 1 = |A | 1 adj A = ac b = ac (Mathematics)

a, b, c are in

It does not have an inverse. 85. Area of parallelogram = |(i 2j + 3k ) (2i + j 4k ) = |(2i i + i j i 4k ) + (4j i 2j + (6k i + 3k j = |(0 + k + 4j ) + (4k 0 + 8i ) |

j + 8j k ) 12k k ) | + (6j 3i 0) |

= |5i + 10j + 5k | = 25 + 100 + 25 = 150 = 5 6. 86. Co-efficient of correlation cov (x, y) = . x y Given = .28 cov (x, y) = 7 . 6 x = 3 Putting values in (1), we 7.6 0.28 = 3 y get

...(1)

7.6 y = .28 3 = 9.05. 195 ----------------------- Page 195----------------------Mock Test4 p q r q r p p q q r p 1 = (p + q + r) 1 1 1 = (p + q + r) [R 2 R 0 p q R , R R 2 1 3 q r R ] 3 1 0 r p [C C 1 q q r r p p q q r r q p r Solutions r (Mathematics)

87.

p + q + r = p + q + r p + q + r

+ C 1 2

+ C ] 3

= (p + q + r) [ (r q)2 {(p q) (p r)}] 2 = (p + q + r) [r q 2 + 2rq (p 2 pr pq + qr)]

2 2 2 = (p + q + r) [ q r + qr + rp + pq)] = (p + q + r) [p2 + q2 + r2 qr rp pq] 1 = 2 2 [(p q) . . [ . p, q, r number] are + (q r) negative 2 2 + (r p) ] > 0 distinct real (p + q + r)

2 (1 cos x)

88.

lim 2 x 0 x 2 2 (2 sin = lim 2 x 0 2 4 sin = lim 2 x 0 = x 0 2 . . [ . when x 0 then, x 2 0] 2 sin . . = (1) = 1. . lim 0 1 = sin 1 0 = sin 1 sin 1 sin 0 1 sin = 2 [sin2 + 1] 2 Now, 0 sin So, 1 2 1 1 + sin 2 2 (1 + sin 2 4 [2, 4]. 196 2 2 ) 4 [from (1)] 2 sin 1 1 sin 1 [R R 1 1 + R ] 3 ...(1) lim x sin2 x 2 2 (x 2) x 2 x x 2)

= 1

89.

----------------------- Page 196----------------------Mock Test4 Solutions (Mathematics)

90.

a (b c ) =

b + c 2 1

1 (a . c ) b (a . b ) c = 2 b + 2 c Comparing a . c =

the co-efficients of b and c 1 1 and (a . b ) = 2 2 1 a . b = 2 1 . . cos = [ . a , b are unit vectors] 2 3 = . 4 91. Since question 1 and 2 are compulsory, 4 questions are left from 6 and 2 are be chosen from them. Required number of ways = 4C2 = 92. sin A = 3 sin (A + 2B) sin (A + 2B) sin A Using componendo and sin (A + 2B) + sin (A + 2B) 2 cos (A + B) sin B . . . tan (A + B) tan B 1 93. tan () = tan (A + B) = 2 tan B = 2 x + y sin x + sin y = 2 sin sin x sin y = 2 cos 2 cos to

4 3 2

= 6.

1 = 3 dividendo sin A = sin A 1 3 = 2 x y 2 1 + 3

2 sin (A + B) cos B

x + y x y sin 2 2

2 1 [since domain tan 1 tan () = 2 Range = 94. 2 3 2 2 1 + dy = d y dx dx2 2 3 2 2 = 1 + dy = d y dx dx2 Degree = 2. 197 ----------------------- Page 197----------------------Mock 95. Test4 Solutions (Mathematics) 2 , . 2 x = , ]

The pth term of an A.P., where first term is a and common difference is d Tp = a + (p 1) d = q th also q term is p ...(2) Tq = a + (q 1) d = p ...(1)

from (1) a = q pd + d Putting value of a in (2), we get th Then r term is Tr = a + (r 1) d = a + (p 1) d + (r p) d = q + (r p) d = q + (r p) (1) = p + q r 96. The number of ways in which four faces of a tetrahedron can be pointed = 4 ! [from 1] . . [ . d = 1] q pd + d + (q 1) d = p q pd + d + qd d = p (p q) d = q p d = 1

The number of different colours is 4 !. Total 97. Since 2 2 . . . sin 4 1 |x | 98. 2 tan 1 2x 1 x number of ways =

ways

of 4 !

selecting = 1 !. 4 !

sin

1 x

2 2 1 x

2 = 2 tan 1 + x 1 tan x 4 x 1 1.

(2 cos x 1) (3 + 2 cos x) = 0 2 cos x 1 = 0 1 cos x = 2 Also 3 + 2 cos x = 0 3 cos x = which is not possible 2 1 cos x = 2 5 x = , 3 3 5 Solution = , . 3 3 198

----------------------- Page 198----------------------Mock 89. 0 = log a a0 = 0 + log a log b ax log b0 b0 bx (ax bx) dx Test4 Solutions (Mathematics)

1 = log a 100. We have 2

1 . log b 2 . . [ . a

0 = b

0 = 1]

2 + (x 3) = 0 2 2 2 (x 1) , (x 2) , (x 3) cannot simultaneously for any real value of x. their sum will have be zero. Hence, number of roots = 0. (x 1) + (x 2) 4 + x 2 101. lim f (x) = lim x 0 x 0 = lim x x 0 4 + x 4 lim x 0 x [4 + x + 2] 1 1 = lim = 4 + x + 2 2 + 2 x 0 1 Hence f (0) = . 4 = 102. x 4 + x 2 .

be

zero

4 + x + 2 4 + x + 2

1 = 4

f is invertible or f exists, if f is one-one onto. 199

----------------------- Page 199----------------------Mock 103. Test4 Solutions (Mathematics) 2

We have 2 lim log (1 + x + x ) + log (1 x + x ) x 0 sec x cos x Using L-Hospital rule, we have d 2 [log (1 + x + x ) + log (1 x + x )] dx

lim d x 0 dx 1 2 (1 + 2x) + 1 + x + x 1 x + x 1 2 (2x 1) (sec x cos x)

lim sec x tan x + sin x x 0 2 2 (1 x + x ) (1 + 2x) + (1 + x + x ) (2x 1)

= lim x 0

2 (1 + x + x ) (1 x + x ) 2 1 + 2x x 2x + x 2 + 2x 3

2 sin x 2 cos

+ sin x x 2 3 x + 2x 2 x

+ 2x 1 + 2x

= lim x 0 2 2 2 (1 + x + x ) (1 x + x ) (sin x sec x + sin x) 2 (4x = lim 2 x 0 1 + x = lim 2 x 0 (1 + x 2 (4x = lim 2 x 0 0 + 2 = 1 + 0 + 0 1 1 + 1 . = 2 1 104. 1 = 1. 2 (1 + x 4 + x ) 1 + 2) 4 + x ) sin x (sec x sin x . . lim 0 = 1 2 x + 1) 1 2 sec sin x + 1 x + x x 2 + x 3 2 3 4 2 + x x + x (sin x sec x + sin x) 2 (4x + 2) x + 2) x

Since there are 4 people P, Q, R, S who are to give lecture to an audience. The number of ways in which their presentation can be arranged = 4 ! = 4 3 2 1 = 24. 200

----------------------- Page 200----------------------Mock 105. Test4 Solutions (Mathematics)

Since is root of equation 2 2 a x + bx + c = 0

again,

2 2 a + b + c = 0 2 2 is a root of a x bx c = 0 a2 2 b c = 0

...(1)

...(2)

when a 0 and 0 < < 2 2 let from (1) f (x) = a x + 2bx + 2c 2 2 b + c = a 2 (b + c) = 2 2 2 2 b + c = a 2 2 Putting 2 (b + c) = 2a in (3), we get 2 2 f () = a + 2b + 2c 2 2 2 = a 2a = Also f () = a 2 2 = a = 3a + 2a 2 2 > 0 f (), f () are of opposite sign. Hence it is continuous an [, ] Hence by intermediate value theorem of continuity f (x) = 0 has a root such that < < . 106. S = n = 1 2n S = n ! = (e 2 1). n Pn nC0 + nC1 + nC2 + + nCn 2 2 2 2 + 2b + 2c 2 2 < 0 2 2 ...(3)

from (2)

n = 1 201 ----------------------- Page 201----------------------Mock 107. Test4 Solutions (Mathematics)

The equations are a + ay z = 0 2x y + az = 0 ax + y + 2z = 0 For consistency, we have 1 2 a 1 2 a C 2 2 0 (1 + 2a) 2 1 a aC , C 1 1 1 a = 0 1 2 0 a + 2 = 0 a + 2 C 3 3 + C ] 1 2 [ (a + 2) (1 + 2a) (a + 2) (1 a )] = 0 2 2 4a (a a 2 2 4a a + a 3 (a 6a 4) = 0 2 (a + 2) (a 2a 2) = 0 2 4 + 8 2 2). = 1 3 3 3 2 + 2 2a )] = 0 2 2 + 2a ) = 0 a ...(i) ...(ii) ...(iii)

[C

[ 2a ( 2a

a = 2, a = Hence a = (1 3, 108. =

Centroid of ABC (ai + bj + ck ) + (bi + cj + ak ) + (ci + aj + bk ) 3

(a + b + c) i + (a + b + c) j + (a + b + c) k = 3 a + b + c = 3 202 ----------------------- Page 202----------------------Mock 109. Test4 Solutions (Mathematics) 2 e is eccentricity of x 2 a 2 b e = 1 + 2 a 2 2 e = 2 a 2 2 2 2 e a = a + b ...(1) a + b 2 b y 2 2 = 1 (i + j + k ).

2 2 2 b = a (e 1) Also e is eccentricity of hyperbola 2 x b2 a2 2 a e = b2 2 2 Now e = 2 b e 2 b2 = b2 + a2 2 2 2 a = b (e 1) from (1) and (2) b2 a2 2 2 2 . 2 = (e 1) (e 1) b + a 2 1 + y = 1 2

...(2)

b 2 2 2 2 1 = e e e e + 1

2 e 110. Let e 1 2 + e + e 1

2 2 = e e 2 2 [dividing by e e ] be n n geometric

2 = 1

G , G , G , ..., G 1 2 3

means between two quantities a and b. Then a, G , G , ..., G , b is a G.P. 1 2 n Gn = (ab)n Here a = x, b = 1 x Gn = (x . 1 x )n = (1)n = 1. 111. The mean deviation from the median less than measures from any other value. is 203 ----------------------- Page 203----------------------Mock 112. Test4 Solutions (Mathematics)

z = x + iy z z1 z z2 = z z1 arg z z2 tan x 8 [from above] (y 4) 1 (x 8) tan (y 4)2 1 + (x 8) (x 6) . . . tan 1 A tan 1 B = tan 1 (A B) (y 4) (x 6) = 4 4 1 y 4 tan x 6 4 = = (x 6) + i (y 4) = (x + iy) (6 + 4i) (x 8) + i (y 4) (x + iy) (8 + 4i)

1 y 4

(y 4) (x 8) (y 4)2 1 + (x 8) (x 6) (x 6) (x 8) (y 4) (x 8) (x 6) 2 (x 8) (x 6) + (y 4) (x 8) (x 6) (y 4) 2 2 x 2 2 = 1 1 (x 6) 1

1 + AB

= tan 4 = 1

= 1

6x 8x + 48 + y + 16 8y 2 x + y 14x + 64 8y = 2y 8 2 2 (x 14x + 49) + (y 10y + 25) = 2 (x 7)2 + (y 5)2 = (2)2 113. = 2 = i 3 + i 2 = iz 69 = (iz) 69 = i 69 z 69 . . 2 [ . i = 1] z = 2 1 + 3i |z 7 5i | = 2. 3 + i

3 23 68 69 ( ) = i . iz 23 (1) 4 17 69 = (i ) . iz 17 1 = (1) . iz 69 69 . . [ . 3 = 1, i 4 = 1]

1 = 1iz 69 1 z = i i i

i i

= i2

= i. 204

----------------------- Page 204----------------------Mock 114. Test4 Solutions (Mathematics)

We know f (x + h) f (x) f (x) = lim h h 0 2 If we put f (x) = cos 2 f (cos x) = x cos2 (x + h) cos2 x lim h h 0 cos2 (x + h) cos2 x 2 cos x sin x = lim h h 0 cos2 (x + h) cos2 x lim h h 0 5 2 c = sin 2x

115.

We

have the expression x Tr + 1 = 5 = Cr x

+ x 5 2 5 r c r Cr (x ) x 10 2r r (c) . . [ . Tr + 1 = n Cr (a) n r r (b) ] r

10 3r = 1 3r = 9 r = 3 5 Co-efficient of x = C3 (c) 3 = 10c 3

2 3 116. cos 5 + cos

2 4 5

= cos 2 = cos = sin

2 108 + cos

2 144 2 (180 36)

(90 + 18) + cos 2 18 + cos 2 2 36 2 5 + 1 + 4 + 16 12 = 16 = 4 3 .

5 1 = = 4

5 + 1 25

5 + 1 + 2 5 16

117.

We have P (A B) = P (A) + P (B) P (A B) ...(1) Since P (A B) = P (A B) [given] P (A B) = P (A) + P (B) P (A B) [from (1)] 2P (A B) = P (A) + P (B) P (A) + P (B) = 2P (A B) P (A) + P (B) = 2P (A) P (B A). The co-ordinates of the foot perpendicular from point P or p (a, b, c) on y-axis is (0, 4, 0). Foot of perpendicular from (1, 2, 3) on y-axis is (0, 2, 0). of the

118.

205 ----------------------- Page 205----------------------Mock 119. Test4 Solutions (Mathematics) series, the mode is

For a continuous computed by the formula fm fm 1 l +

2fm fm 1 fm + 1 fm f1 or 120. l + 2fm f1 f2 z = x + iy _ z = x iy _ It is given that z = z

i.

x + iy = (x iy) x + iy = x + iy 2x = 0 x = 0

Putting value of x = 0 in z = x + iy, we get z = iy z is purely imaginary. 121. Given parabola is 2 x 8x + 2y + 7 = 0 2 (x 4) (x 4) + 2y + 7 = 16 2 + 2y = 9 2 (x 4) = 2 y 9 2

2 X = 2Y Which is parabola with vertex and latus rectum 4a = 2 1 a = 2 This is downward parabola. given by X = 0 x 4 = 0 9 Y = a y 2 9 x = 4 and y = 2 Focus = (4, 4). 122. (x + y )n = 4096 n clearly 2 n = 12 Here n = 12 n for r = 2 r = 6. Hence the greatest co-efficient is 12 12 11 10 9 8 7 C6 = 6 5 4 3 2 1 . Therefore = 4096 = 2 is even, then nCr 12 C6 is is 12 2 = 2 1 = 4 1

(4, 9 2)

Focus

is

greatest greatest for

= 11 3 4 7 = 924. 206 ----------------------- Page 206----------------------Mock 123. Test4 Solutions are n = C0 x n + r y + + n Cn 1 x 100 (x + y) = + 100 C0 x 100 C2 x C2 x 1 y n 1 + real n y (Mathematics) numbers, 0 + n C1 x n 2 y 2 + + n Cn x 0 y then for 1 y n n r Cr x n all

If x and y n N n (x + y)

n 1

100 0 100 99 y + C1 x y 98 y 2 + 100 100 C3 x 0 C100 x 100 C x 2 0 C100 x y y 98 y 100 2 97 y 100 3

+ + Similarly, (x y)100 = 100C0 x100 y0 100 100 100 (x + y) 100 = [(x + 100 C x 1 + (x y) 99 y + 100 C x 2 + + 100 + (x 100 C x 1 99 y + 100 C x 2 + 100 = 2 [x + 100 C x 2 98 y 98 y 100 C3 x 97 y 3 + + 100 C 1 100 99 x y +

2 +

100 C x 3 0

97 y 100 ) C x 3 100

C100x y 2 100

97 3 y

100 C100 y 98 2 100 96 4 y + C x y 4

100 + + = 51 terms. 124. a 3 cos (B C) 3 3 = k sin A cos (B C) = k3 sin2 A sin (B + C) cos (B C) 3 k = 2 3 k = 2 2 + sin B (sin 2C + sin 2A) + sin2 C (sin 2A + sin 2B)] 3 = k 3 = k sin A sin B sin (A + B) 3 = k [sin A sin B sin C + sin B sin C sin A + + sin C sin A sin B] = 3 (k sin A) (k sin B) (k sin C) = 3abc. [sin 2 A sin B cos B + sin 2 [sin 2 A (sin 2B + sin 2C) sin 2 A (sin 2B + sin 2C) C100 y

100 ]

. . [ . a = k sin A]

B sin A cos A]

207 ----------------------- Page 207----------------------Mock 125. Let Test4 I = 0 Solutions 2 (Mathematics) x dx ( 2 x) dx ( 2 x) ...(2) f (a x) dx 0 ...(1)

4 + 3 sin log 4 + 3 cos x 2 4 + 3 sin I = log 0 4 + 3 cos 2 = 0

4 + 3 cos x log 4 + 3 sin x dx a a f (x) dx =

Adding

. . . 0 (1) and (2)

4 + 3 sin x log 4 + 3 cosx dx 2 4 + 3 cos x + 0 log 4 + 3 sin x dx 2 4 + 3 cos x 4 + 3 sin x = 0 log 4 + 3 sin x 4 + 3 cos x 2 = log1 = 0 0 2I = 0 208 ----------------------- Page 208----------------------PHYSICS (Mock Test-4) 1. The emf E = 4 cos (1000t) volt is applied to an L-R circuit containing inductance 3 mH and resistance 4. The amplitude of current is : (a) 4 7 A 4 (c) A 7 2. (b) 1.0 A

Mock Test4 (P

hysics) (d) 0.8 A

Two lens in contact form an achromatic combination. Their focal lengths the ratio of 2 : 3. Their dispersive powers will be in the ratio of : (a) 3 : 1 (b) 1 : 3 (c) 2 : 3 (d) 3 : 2 The efficiency of a transformer is high because : (a) there is no moving part transformer (b) it produces very high voltage (c) it produces very low voltage (d) none of these In figure, the value of resistance x, when the potential difference between B and D is zero, will be : (a) 5 (c) 9 (b) 6 (d) 8

are

in

3.

very in the

4.

point

5.

Two waves have the intensities I and 9I respectively, what will be the ratio of their amplitudes ? (a) 1 4 (b) 8 4 (c) 2 2 (d) 1 3 The velocity of the emitted in photoelectric only on : photo-electron effect depends

6.

(a) the frequency and intensity incident light. (b) the frequency of incident light. (c) both (a) and (b) (d) the wavelength of incident light.

of

209 ----------------------- Page 209----------------------Mock 7. Test4 (Physics) will be of

Light of certain colour has 2000 waves to the millimeter in air. What wavelength of this light in medium refractive index 1.25 ? (a) 1000 (b) 2000 (c) 3000 (d) 4000

8.

A 50 Hz A.C. current of rest value of 1 A flows through the primary of transformer. If the mutual inductance between the primary and secondary coils is 1.5 H. The rest value of voltage induced in the secondary is : (a) 75 V (b) 300 V (c) 250 V (d) 225 V A 60 watt bulb carries a current of 0.5 A. Total charge passing through in one hour is : (a) 3600 C (b) 3000 C (c) 2400 C (d) 1800 C In the circuit of given figure, produced in 5 resistance due to a unit flow in it, is 10 cal/sec. produced in 4 resistor is : (a) 1 cal sec (c) 3 cal sec (b) 2 cal sec (d) 4 cal sec the The heat heat

9.

10.

11.

Light of wavelength strikes a photosensitive surface and electrons are ejected with kinetic energy (E). If the kinetic energy is increased two times i.e. 2E. The wavelength must be changed to , where : (a) < < (b) > 2 (c) = (d) < 2 2 Which of the following expression for velocity of light : 1 (a) c = 0 0 is correct

12.

(b) c = 0 0 (c) c = 0 0 (d) c = 0

210 ----------------------- Page 210----------------------Mock 13. Test4 (Physics)

A magnetic needle suspended by a silk thread is vibrating in the earths magnetic field, if the temperature of the needle is increased by 500C, then : (a) the time period decreases (b) the time period remains unchanged (c) the time period increases (d) the needle stops vibrating A glass rod rubbed with piece of silk brought near a rubber rod rubbed with a piece of wool. Then between there will be a force of : (a) attraction (b) repulsion (c) no force (d) repulsion and attraction both is them,

14.

15.

Water in a lake is converted into ice at 0C when the temperature of atmosphere is 10C. If it takes 7 hours to form ice of thickness of 1cm. Then the time required for the thickness of ice to increase from 1 cm to 2 cm, will be : (a) less than 7 hour (b) more than 14 hour (c) 7 hour (d) 14 hour An alternating source of 220V, 50 Hz connected to an electric lamp. The peak voltage of source is : (a) 211 V (b) 210 V (c) 320 V (d) 311 V At what temperature will thermometer show the same reading as on a Fahrenheit thermometer ? (a) 0F (b) 40F (c) 459.4F (d) 25.6F The dimensions of permittivity 1 3 2 4 are : 0 is

the

16.

17.

Reaumur

18.

(a) [M

A T ] 3 2 4 (b) [M L A T ] 1 1 2 2 (c) [M L A T ] 1 3 2 4 (d) [M L A T ] 2 19. A bomb is dropped from an aeroplane flying horizontally with a velocity 720 km/hour at an altitude 980 m. When will bomb hit the ground ? (a) 7.2 sec (b) 1 sec (c) 0.15 sec (d) 14.14 sec of

211 ----------------------- Page 211----------------------Mock 20. Test4 (Physics) is moving time 2T 3 up with

A simple pendulum has a period T. It taken inside a lift uniform acceleration of g/3. Now its period will be : 2 3 (a) 3 T (b) 2 T (c)

3T (d) 2 1013 move be metre in

21.

The distance of saturn and neptune from the sun is nearly 1012 and respectively. circular orbits, in the ratio : (a) 1000 : 1 (c) 10 : 1 Assuming their time period they will

(b) 1 : 100 (d) 1 : 1010 two

22.

The difference in angular momentum associated with an electron in the successive orbits of hydrogen atom, is : h h (a) (b) 2 2 4 (n 1)h (c) (d) 2 Hubbles law is expressed as : (a) v = 1 + HR (b) v = HR 2 R R (c) v = (d) v = N N 540g of ice at 0C is mixed with 540g of water at 80C. The temperature mixture is : (a) 40C (b) 80C (c) 0C (d) less than 0C

23.

24.

of

25.

A block of table. Its walls springs k (k > 2 1

mass m is placed on a smooth two sides are attached to fixed by means of collinear of spring constant k ) as shown in the fig. The block 2

horizontal k1 and

is made to oscillate horizontally the line of two springs. The frequency of its oscillation is : 1 (a) 2 1 (c) 26. k 1 m (d) 2 m 2 k1 + k2 + k 2 (b) 2 1 1 m m k1 + m k k 1 2

along

Two organ pipes 50.5 cm produce velocity of sound is : (a) 30 m/s (c) 303 m/s

of length 50 cm three beats. Then (b) 300 m/s (d) 606 m/s

and the

212 ----------------------- Page 212----------------------Mock 27. Test4 (Physics) number of

A gas is taken through a thermodynamic states. When happen to its specific heat ? (a) It is always constant (b) It increases (c) It decreases (d) It can have any value depending upon process of heat absorbed or evolved Four corners done centre to (a) zero 2 Q2 (c) a 0 (d) 2 a 0 equal charges of a cube of in carrying infinity is :

28.

are placed at side a each. Work a charge  from 2 Q2 (b) a 0 Q2

four its

29.

An incompressible fluid flows steadily through a cylindrical pipe which has radius 2R at a point A and radius R at a point B. Further along the flow of direction if the velocity at point A is v, its velocity at point B will be :

(a) v 4 (c) 4v 30. Penetrating power increased by : (a) increasing the between anode and cathode (b) decreasing the between anode and cathode (c) decreasing the filament (d) increasing the filament

(b) (d)

2v v 2

of

X-rays

can difference difference of of cathode cathode

be

potential potential current current

31.

On the basis of kinetic theory of gases, the mean kinetic energy of 1 mol degree of freedom is : 1 1 (a) kT (b) RT 2 2 3 3 (c) kT (d) RT 2 2 Compton effect is associated with : (a) positive rays (b) ry (c) ry (d) Xry If we dip capillary tubes of different radii r in water and the water rises to different heights h in them, then we constant : 2 (a) h r (b) h r 2 (c) hr (d) hr

per

32.

33.

shall

have

213 ----------------------- Page 213----------------------Mock 34. Test4 (Physics) to

Which of the quantities : (I) magnetic declination and (II) dip is/are used determine the strength of earths magnetic field at a point on the earth surface ? (a) both I and II (b) neither I nor II (c) I only (d) II only A p-type semiconductor has acceptor levels 57 meV above the valence band. The wavelength of light required to create a hole is : (a) 57 (b) .57 103 (c) 11.61 1033 (d) 217100 with a velocity of

35.

36.

A motorcycle moving

72 on of

km/hour on a flat road takes a turn the road at a point where the radius curvature of the road is 20 m. 2

The

acceleration due to gravity is 10 m s . In order to avoid skidding, he must not bent with respect to the vertical plane by an angle greater than : (a) = tan 1 (2) (b) = tan1 (6) (c) = tan1 (4) 37. (d) = tan1 (25.92)

Two carts of masses 200 kg and 300 kg respectively standing on horizontal straight rails are pushed apart by an explosions of the device kept in connecting mechanism of carts. The coefficient of friction between carts and rails are identical. If the 200 kg cart travels a distance 36 cm and stops, what is the distance covered by cart weighing 300 kg : (a) 4 m (b) 8 m (c) 12 m (d) 16 m A train accelerated uniformly from rest attains a maximum speed of 40 m/s in 20 sec. It travels at this speed for 20 sec and is brought to rest with retardation in 40 sec. The average velocity during this period is : (a) (80 3) m s (b) 30 m s (c) 25 m s (d) 40 m s Pressure at the bottom of tank of water is 3P, where P is atmospheric pressure. If the water is drawn out till the level of water is lowered by one fifth, then the pressure at the bottom of the tank is : (a) 2P (b) 13P 5 (c) 8 P 5 (d) 4P 5 ^ ^ ^ ^ The vector A = i + j where i and j are unit vectors along x and y axis respectively, makes an angle with x axis of : (a) 30 (b) 45 (c) 60 (d) 90

38.

uniform

39.

40.

214

----------------------- Page 214----------------------Answer Sheet (Mock Test-4) (PHYSICS) 1. 6. 11. 16. 21. 26. 31. (d) (d) (a) (d) (d) (c) (b) 2. 7. 12. 17. 22. 27. 32. (c) (d) (a) (d) (a) (d) (d) 3. 8. 13. 18. 23. 28. 33. (d) (b) (d) (a) (b) (c) (d) 4. 9. 14. 19. 24. 29. 34. (c) (d) (a) (d) (c) (c) (a) 5. 10. 15. 20. 25. 30. 35. (d) (b) (b) (b) (b) (a) (d)

36.

(a) Hints &

37.

(d) Solution of

38.

(c)

39.

(b)

40.

(b)

1. From the formula ons (Physics)

impedance

Mock Test4 Soluti

Z =

2 2 2 R + L

2 3 2 = (4) + (1000 3 10 ) = 5 Now, emf E = 4 cos (1000t), = 100 and v = 4 V 4 Hence, current I = = = 0.8 A Z 5 2. The condition given by 1 = 2 Hence, 3. 4. : 1 = 2 : 3 2 f2 for the f1 = 3 achromatism 2 is

None of explanation is correct. The equivalent resistance of arm AB RAB = 15 + 6 = 21 The equivalent resistance of arm AD 6 6 RAD = 6 + 6

+ 15

= 3 + 15 = 18 The equivalent resistance between C and D RCD = 4 4 4 + 4 = 2 + 4 = 6 The equivalent resistance between B and C R BC Since, Hence, Wheatstone RBC = RAB or RAD RCD RCD = 8x 8 + x + 3 bridge is balanced + 4

RBC = RAB

RAD 8x So, 8 + x or + 3 = 21 x = 8 6 18 215

----------------------- Page 215----------------------Mock 5. Test4 Solutions (Physics) intensity is

We know that the proportional to square of amplitude, 2 i.e., I a 2 I1 a1 I2 = a2 2 2 a1 I = 2 9I a2 a1 or a2 I = = 9I the the h As p = h or mv = 1 v : no. 3 = 10 m So, wavelength of light in air is given by 3 = 10 2000 = 5000 1010 = 5000 = 5 10 7 m of waves wave

Hence,

(here I1 = I I2 = 9I)

or

1 3 velocity of

6.

We know that photo-electron depends upon length of incident light.

7.

Given

= 2000,

distance

Using

formula for refractive index m =

m or m = m 5000 10 1.25 = 4000 10 = 4000 1 8. Here : T = = f 1 10 10

, 50 In this time interval, the current swings from times between zero and 1 maximum value. So, the time for changing current from zero to one ampere is given by 1 dt = = From the 1

50 4 200 formula, induced emf is di e = M dt = 1.5 1 1 200 216

= 200 1.5 = 300 V ----------------------- Page 216----------------------Mock 9. Test4 Solutions (Physics)

From the formula of current and charge. charge = current time or q = it (given i = 0.5 A, t = 1 hour = 60 60 sec) or q = 0.5 60 60 = 1800 coulomb current flows i2 current resistors, so,

time

10.

Suppose i1 resistor and 4 and 6

through 5 flows through

i1 5 = i2 (4 + 6) So, Now heat i1 = 2i2 produced in 5 resistor (1) is

given by or From 2 i1 5 = 10 cal i1 = 2 cal ohm 1 2 eq. (1), i2 = i1 = 2 2 1 cal 2 Hence, heat produced in 4 resistor 2 = i2 4 = 1 4 2 2 11. Since, hc E = hc and From 2E = equations (1) and (2), we get 1 W0 (2) W0 (1) = 2 cal sec =

1 E = hc For E to be positive >

and hc 2 1 = W 0 2 1 > or > Hence, < < 2 12. We In

1 know that the velocity c = 0 0 free space E c = B = H 0 1 = 0 1 = 0 0 . . E . H 0 = 0 E

0 0 217 ----------------------- Page 217----------------------Mock 13. Test4 Solutions (Physics) the

When the magnetic needle is heated to such a high temperature, then magnetic needle losses its magnetism, so, the restoring torque no longer acts and hence needle stops vibrating. When the glass rod is rubbed with silk, the rod acquires positive charge when the rubber rod is rubbed with wool, the rod acquires negative charge. Again when the glass rod and rubber rod are brought close, they attract. Here, t1 = 7 hour, x2 = 2 cm and x1 When the ice is grown in lake, then time for ice formation 1 L t = (x 1 2 K 1 L = (1) = 1 cm. initial 2 ) 1 2 (1)

14.

and

15.

2 K The final time for ice formation is 1 L t 2 = (x 2 x ) 2 1 [from (1)] 2

2 K = 1 L (2)2 (1)2 2 K = 3 t1 = 3 7 = 21 16. From the formula E0 E rms or = 2 E0 = 2 Erms = 2 220 = 311 volt 17. From the formula R = 4 9 F 32

(Let R = x and

F = x)

x = or or

x 32

4 9 9x = 4x 128 x = 25.6F q q 1 2

18.

2 4Fr The dimensions of permittivity dimensions of q1 dimensions of q2 = dimensions of force dimensions of r [AT] [AT] 2 [MLT 1 = [M L 2 ] [L ] 3 2 A T 4 ] 218 2

0 =

----------------------- Page 218----------------------Mock 19. Here Using Test4 Solutions (Physics) 2 : u = 0, h = 980 m, g = 9.8 m s 1 the relation s = ut + 2 980 = 0 t + or 20. 1 2 gt 2 2

9.8 t

t = 102 = 14.14 s upwards, g g = (g + a) = g + 3 = 3 by 4g then

When the lift is moving the acceleration becomes

Hence, the

time period is given T = 2 l g L T = 2

Now

= 2

4g 3 3 2

L g

3 2 2 3 = T 2 of 2 time 2 R 3 2 T R

L g

21.

Using the relation Keplers law T T Hence, T2

period

from

R 3 2 1 = R2 3 2 1012 = 13 10 1 = 1010

T 1 22. Angular momentum

: T 2

= 1 : 1010 nh

in nth

orbit = 2

Angular momentum in (n + 1)th orbit (n + 1)h = 2 Hence, difference in angular momentum between two successive orbits by (n + 1)h nh h = 2 2 2 23. Hubbles law is v = HR 219 ----------------------- Page 219----------------------Mock 24. Test4 Solutions (Physics) is given

The amount of heat required to melt 540g of ice is given by = 540 80 = 43200 cal (latent heat of ice = 80 kcal g) Now heat lost by 540 g of 80C to 0C.

water

form

= 540 1 (80 0) = 43200 cal Therefore, all the temperature of mixture 25. ice must is zero. be melt and

Two springs are connected in parallel, so, the equivalent force constant keq = k1 + k2 So, frequency of f = 2 oscillation is 1 m open v n = 2l v n1 = 2 0.5 v and 2 0.505 (Given l1 = 50 cm= 0.5 m,l2 = 50.5 = 0.505) From equations (1) and (2), we get n1 n2 = 3 3 = v v 1.0 1.01 0.01v = 1.01 0.01v = 3.03 3.03 or v = 0.01 = 303 m s n2 = (2) (1) organ pipe is an k1 + k2

26.

The frequency given by

of

27.

The specific heat of a gas depends upon the process and can have any value as specific heat at constant volume is (C ) v and at constant pressure is (C ). p

28.

The potential due to four charges, at the centre is given by

1 = 4 0

4Q = a

4Q 2 4 a 0

2 The work done will be = QV = Q 4Q 2 4 a 0 220 a 0 ----------------------- Page 220----------------------Mock 29. Test4 Solutions of (Physics) liquid of 2 flowing per flowing 2 per

2Q2 =

Since, the volume second at A = the volume second at B

liquid

v (2R)

= v R v = 4v

30.

We know that by increasing the potential difference between anode and cathode, X-rays having short wavelength will be produced which have more penetrating power. According to kinetic theory of gases, the mean kinetic energy of molecules degree of freedom is given by 1 = and for a gram 2 mole, the kT K.E. since k = R N effect of liquid is in per

31.

= NkT = RT 2 32. 33. We know that related with X-rays. the 2 compton

If the height h is the rise capillary tube of radius r, then expression of height is given by 2T cos h = g

(1)

Since, we dip capillary tubes of different radii r in water and water

rises

to

different height h in them, then equation becomes 2T cos hr = g 34. When we measure horizontal component of earths magnetic field and angle of dip, then the strength of earths magnetic field can be found out. The energy of a photon is given by hc h = = 57 meV hc 3 Hence, = 57 10 1.6 10 6.6 10 57 10 = 0.2171 10 = 217100 3 34 = constant

35.

19 J 8

or

3 10

1.6 10
4

19

221 ----------------------- Page 221----------------------Mock 36. Test4 Solutions (Physics)

Using the formula for motor cycle not ot to skid 1 v2 = tan rg where r = 20 m v = 72 km hour = 20 m s 18 = tan1 20 20 20 10 1 or = tan (2) = 72 5

37.

Apply conservation of momentum 200v1 = 300v2 v1 300 3

So, If E and 1

= = v2 200 2 E are the kinetic energies after 2 work

explosion and W 1 and W2 are the done in stopping the carts by friction of force (= mg). 1 2 2 mv1 W 1 We have 1 mv2 2 1 2 or 2 = 2 2 v1 or 2 v2 9 or or = 4 x x = 16 m 36 = x 36 2 = W 2 =

F d 1 F d 2

200 v1

200 g 36 300 g x

1 300 v2 2

222 ----------------------- Page 222----------------------Mock 38. Test4 Solutions (Physics)

The acceleration of train in 20 sec is given by 40 0 a = = 2 m s = 0)] given by 2 u + 2as a 20 [from the formula v = u + at (here u Now the distance travelled is 2 equation of motion v = v2 u2 So, s1 = / 2 = Now distance 40 40 0 2 2 travelled 2

= 400 m with constant

speed of 40 m/s in t2 = 20 sec is s2 = 40 20 = 800 m Again given by the acceleration 0 40 a = Distance given by 40 travelled = 1 m s in this direction is in (3rd 2 case) is

s3 =

40 40 0 2 1 of the train is

= 800 m Therefore, given by average vavg = = 80 39.

speed

400 + 800 + 800 20 + 20 + 40 2000 = 25 m s due to

Since the pressure at the bottom height h of water and atmosphere is 3P. Hence, pressure due to the height h will be 3P P = 2P = gh 2P So, h = g When height h is reduced by one fifth, it 4 will become . 5 Total pressure at the bottom will be 4 = P + 5 4 = P + 5 8 = P + 5 P = 5 hg 2P g

g
13P

223 ----------------------- Page 223----------------------Mock Test4 Solutions (Physics)

40.

Angle

between by cos

vectors

^ ^ A = i + j AB |A | |B|

and

^ B = i is given

or

cos =

1 1 + 1 0 + 0 1 = 2 2 1

So,

= 45 224

----------------------- Page 224----------------------REASONING (Mock Test-4) Directions (for Q. 126 to Q. 128) : Answer the following questions : asoning) 126. A is the sister of B. B is the brother of C. C is the son of D. How is D related B ? (a) mother (b) uncle (c) daughter (d) son B is Cs husband. A is the sister of B. D is the sister of C. How is D related to B ? (a) son (b) sister-in-law (c) uncle (d) brother Sumitra, Jyoti and Kavita are the sisters. Anand is the son of Sumitra, Divya and Archana are the daughters of Jyoti and kavita. Mona is the daughter of Anand. What is the relation between Mona and Kavita ? (a) sister (b) mother (c) daughter (d) grand-mother Directions (for Q. 129 to Q. 132) : Select the pair of words, which are related same way as the capitalised words are related to each other. 129. ARGUMENT : DISAGREEMENT : : ? (a) ignore : judgment (b) mitigate : repent (c) punish : criminal (d) thought : thinking KICK : FOOTBALL : : ? (a) smoke : cigar (b) boat : fisherman (c) wine : bottle in the to Mock Test4 (Re

127.

128.

130.

(d) 131.

table : chair

MUSIC : MELODY :: ? (a) sound : noise (b) heat : warmth (c) tough : delicate (d) love : hate FRICTION : RESISTANCE :: ? (a) moisture : humidity (b) paper : page (c) fast : quick (d) speak : sing 225

132.

----------------------- Page 225----------------------Mock Test4 (Reasoning) Q. 133 to Q. 135) :

Directions (for Complete the series. 133.

1, 2, 3, 5, 7, 11, 13, 17, : (a) 19 (b) 23 (c) 29 (d) 33 7, 11, 17, 19, 23 : (a) 15 (c) 13 4, 64, 5, 125, 216 : (a) 8 (c) 146 (b) 19 (d) 4 (b) 6 (d) 256 226

134.

135.

----------------------- Page 226----------------------Answer Sheet (Mock Test-4) (REASONING) 126. 131. (a) (b) 127. 132. (b) (a) Solution B Mock Test4 Solutio C is is the are 128. 133. (d) (a) 129. 134. (d) (c) 130. 135. (a) (b)

Hints & 126.

As per the question, A is the sister of and C whereas D is the father or mother of A, B and C. Therefore, D is the mother of B. ns (Reasoning) 127. According to question, B husband and wife. A and D are the sisters of B and C respectively. Therefore, the sister-in-law of B. As per the question, Mona and D

128.

grand-daughter of Sumitra and Kavita is the sister of Sumitra. Therefore, Kavita is grand-mother of Mona. 129. As argument and disagreement are synonym. In the same way thought and thinking are also synonyms. option (d) is correct. The football is kicked by player same as cigar is smoked by smoker. option (a) is correct. We know melodious, warmth. that by Similarly, music heat is the Therefore,

130.

Therefore, we used felt for

131.

132.

As per question, friction and resistance are synonyms. Similarly, moisture humidity are also synonyms. Therefore, option (a) is correct.

and

133. Therefore, the 134. next number will be 19. number number in in

This series shows increasing order. blank space will be 13.

that all Therefore,

135.

As per question, IInd and 4th numbers are the cube of first and third. Therefore, last number will be cube of six. Therefore, correct answer is (b), i.e., 6. 227

----------------------- Page 227----------------------CHEMISTRY (Mock Test-5) 41. emistry) (a) (b) (c) (d) 42. nucleus of inner orbital outer most orbit number one the atom of the atom orbit of the atom having principal In a electron radio active comes from : decay an emitted Mock Test5 (Ch

quantum

pK values of four acids are given below a at 25C. Indicate the strongest acid : (a) 2.0 (b) 2.5 (c) 3.0 (d) 4.0

43.

Carbon with metal (a) carbide (c) hydroxide

forms : (b) carbonate (d) oxide

44.

In

O , H O 2 2

and O 2 3

the

correct order of

OO bond length will be : (a) O > O > H O 2 3 2 2 (b) O 3 (c) H O 2 (d) 45. O 2 2 > H O 2 2 > H O 2 > O 3 > O 3 2 2 (a) +2 (c) +5 46. (b) +3 (d) 1 7 2 > O 2 > O 2

Oxidation number of P in Mg P O is :

In which of the following molecules, octet rule is not obeyed ? (a) PCl3 (b) ClF3 (c) ICl (d) CO2 Nesslers reagent is represented by (a) K [HgI ] + KOH 2 4 (b) K [HgI ] 2 (c) HgI (d) K [HgI ] 2 2 :

47.

3 2 H O, H = 68.39 kcal 2 2 H = 48 kcal

48.

H 2

+ 1/2O

K + H O + water KOH (aq) + 1/2 H 2

KOH + water KOH (aq) H = 14 kcal The (a) (b) (c) (d) heat of formation of KOH is ( in kcal) 68.39 + 48 14 68.39 48 + 14 68.39 + 48 + 14 68.39 + 48 + 14 228 ----------------------- Page 228-----------------------

Mock 49.

Test5

(Chemistry) one of the is correct for the

At constant T and P, which following statements reaction ? CO(g) + 1/2O (g) CO (g) 2 (a) (b) (c) (d) H H H H = E < E > E is independent of reactant

of

physical

states

50.

The pH of solution is zero. The solution is : M M (a) H SO (b) HCl 2 4 2 2 M M (c) HCl (d) H SO 2 10 10 What weight of K Cr O 2 2 7 is

4 required to

51.

prepare 500 ml of 0.1 N solution in acidic medium ? (a) 2.45 g (b) 4.5 g (c) 14.9 g (d) 98 g 52. A saturated The solution value of its (b) Ag SO 2 solubility 6.25 104 of is 4

2.5 102 M. product is : (a) 6.25 105

(c) 15.625 106 53.

(d) 3.125 106 pairs at of

Which one of the following solution can we expect to be isotonic the same temperature ? (a) 0.1 M urea and 0.1 M NaCl (c) 0.1 M urea and 0.2 M MgCl2 (c) 0.1 M NaCl and 0.1 M Na SO 2 (d) 0.1 M Ca(NO ) and 0.1 M Na SO 3 2 of two are u gases 1 masses are m and m respectively. Which 1 2

4 2 4 Their

54.

Molecular velocities same temperature

at the and u . 2

of the following expression is correct ? m1 m2 (a) = (b) m u = m u 2 2 1 1 2 2 u1 u2 m1 m2 2 2 (c) = (d) m u = m u 1 1 2 2 u1 u2 55. + A solid A B has a body centered cubic structure. The distance of closest approach between the two ions is 0.707 . The edge length of unit cell is : (a) 3/2 pm (b) 142.2 pm (c) 2 pm (d) 81.63 pm 229 ----------------------- Page 229----------------------Mock 56. The ice (a) (b) (c) (d) Test5 (Chemistry)

high density of water compared to is due to : hydrogen bonding interactions dipole-dipole interactions dipole induced dipole interactions induced dipole induced interactions data for the

dipole reaction Rate

57.

The experimental 2A + B2 2AB is : [A]

Experiment

[B ] 2 (mole s 0.50 1.00 1.00 1.6 104 3.2 104 3.2 104 2

1 )

1 2 3

0.50 0.50 1.00

The rate equation for the above data is : (a) rate = K[B ] 2 2 2 (c) rate = K[A] [B] (d) rate = K[A] [B] 58. The ion that is isoelectronic with CO is : (a) O (b) N+ 2 2 (c) O+ (d) CN 2 (b) rate = K[B ] 2 2

59.

Cell reaction is spontaneous when (a) Ered is negative (b) Ered is positive (c) G is negative (d) G is positive Which on NaCl (a) Na

60.

one of the following is prepared a large scale by the electrolysis ? CO (b) NaOH 2 3 (d) NaOCl

of

(c) NaHCO3 61.

In a first order reaction, half of reaction is completed in 100 seconds. The time for 99% reaction to occur will be : (a) 664.6 seconds (b) 600 seconds (c) 630.2 seconds (d) 676.6 seconds Oxidation is : (a) +2 (c) + 1 number of Fe in K [Fe(CN) ] 3 (b) +3 (d) +4

the

62.

63.

The separation of colloidal particles from those of molecular dimensions is known as : (a) photolysis (b) dialysis (c) pyrolysis (d) peptisation 230

----------------------- Page 230----------------------Mock 64. Test5 (Chemistry)

Which one of the following burns with an explosion in contact with water and air ? (a) Na (b) Ca (c) Mg (d) Zn Product AlCl is : 3 (a) AlH3 (c) Li of reaction between LiH and

65.

(b) LiCl3 (d) LiAlH4

66.

Vitamin A is also known as : (a) axerophytol (b) aneurin (c) riboflavin (d) folic acid If 8.0 g of radioactive isotope has a half life of 10 hour, then the half life of 2.0 g the same substance is : (a) 2.5 hr (b) 5 hr

67.

(c) 10 hr 68. Westrosol is : (a) acetylene tetrachloride (b) trichloro ethylene (c) vinyl chloride (d) acetylene di chloride

(d) 40 hr

69.

Which one of the following reaction will not take place ? (a) Fe + H SO FeSO + H 2 4 4 (b) Cu + 2AgNO 3 (c) 2KBr + I2 KI + Br2 (d) CuO + H Cu + H O 2 Cu(NO )

2 + 2Ag 3 2

2 inter-halogen

70.

Which of the compound is not formed ? (a) IF5 (c) BrCl7 Glass is soluble in (a) HF (c) HClO4 :

following (b) BrF5 (d) ICl (b) H SO 2

71.

(d) aqua-regia volume strength of 1.5 N H O 2 2

72.

The

solution is : (a) 4.8 Volume (c) 3.0 Volume 73. The second earths crust is : (a) iron (c) aluminium Water gas is mixture of : (a) H O + air 2 (c) CO + CO2 an most

(b) 8.4 Volume (d) 8.0 Volume abundent (b) silicon (d) carbon important fuel. (b) It is 2 (d) H2 + CO2 231 the element in

74.

CO + H

----------------------- Page 231----------------------Mock 75. Calomel is : (a) Hg Cl 2 Test5 (Chemistry) (b) HgCl 2

and Hg 2

(c) Hg + HgCl 2 76. Na S O 2 2 (a) Na S 2 (c) Na S O 2 4 77. + I 3 2

(d) Hg Cl 2 product is : (b) (d) S Na SO 2 2 is : (b) steel (d) graphite

The hardest substance (a) iron (c) diamond

78.

The velocity of electron in the fourth orbital of H atom is v. The velocity of electron in first orbit would be : (a) 4v (b) 16v (c) v/4 (d) v/16 Most acidic is : (a) CH COOH 3 (c) HCOOH (b) C H CH COOH 6 5 2

79.

(d) CH CH COOH 3 2

80.

The Bohr orbit radius for the hydrogen atom (n = 1) is approximately 0.53 . The radius for the first excited state orbit (in ) is : (a) 0.13 (b) 1.06 (c) 4.77 (d) 2.12

(n = 2)

232 ----------------------- Page 232----------------------Answer Sheet (Mock Test-5) (CHEMISTRY) 41. 46. 51. 56. 61. 66. 71. 76. (a) (b) (a) (a) (a) (a) (a) (c) 42. 47. 52. 57. 62. 67. 72. 77. (a) (a) (a) (a) (b) (c) (b) (c) Solution decay, neutron charges Mock Test5 Solutio 0 1e + 1H 1 43. 48. 53. 58. 63. 68. 73. 78. (a) (b) (d) (d) (b) (b) (b) (a) 44. 49. 54. 59. 64. 69. 74. 79. (c) (b) (d) (c) (a) (c) (b) (c) 45. 50. 55. 60. 65. 70. 75. 80. (c) (a) (d) (b) (d) (c) (d) (d)

Hints & 41.

In a radioactive to a proton as : ns (Chemistry) 1 0n

Hence, electron comes from the nucleus. 42. pKa = 2.0 represents the because lesser pK , stronger the acid : a Carbon with metal Bond length is order. In H O is 1.5 45. while in forms carbide. inversly related to bond bond order is 1. In O 2 2 O the bond order is 2. 2 oxidation number of P in it 3 strongest acid

43. 44.

Let x be the Mg P O 2 2 7

2x(+2) + 2x + 7(2) = 0 4 + 2x 14 = 0 2x = 10 x = 5 46. Geometry of ClF molecule is T-shaped. 3 the central

There are ten electrons (three bond pairs and two lone pairs) about chlorine atom, so ClF does not obey octet 3 rule. 47. Nesslers K [HgI ] + KOH. 2 4 reagent is

represented

by

233 ----------------------- Page 233----------------------Mock 48. Test5 Solutions (Chemistry) equation is : 1 K + 1/2 O (g) + 2 2 H = ? H 2 + 1/2O 2 H O 2 (i) H (g) KOH 2

The desired therochemical

H = 68.39 kcal K + H O + water KOH(aq) + 1/2H

2 H = 48 kcal KOH + water KOH (aq) On reversing H = 14 kcal equation (iii)

2 (ii) (iii)

KOH(aq) KOH + water H = + 14 kcal Now, on adding (i), (ii) and (iv), we will get the desired equation. i.e., K + 1/2H2 + 1/2O2 KOH 49. H = 68.39 48 + 14 Kcal

...(iv)

For the reaction CO(g) + 1/2O (g) CO (g) 2 H = (1 mol of CO )

2 2

(1mole of CO + 0.5 = 0.5, i.e., it is negative Hence, H < E 50. H SO 2 4 1 mole 0.5 M M 1.0 M 2 i.e.,

mole

of

O ) 2

2H+ 2 mole

+ SO 4

+ [H ] = 1.0 M pH = o 1 = 0.00

51.

Equivalent medium

weight

of

K Cr O 2 2 7

in

acidic

molecular weight = 6 294 = 49 6 Now, by using the relationship NEV Normality = N/ 10 w = 1000 V = 500 ml, E = 49 W = 1 49 500 =

10 1000 245 = = 2.45 g 100 234 ----------------------- Page 234----------------------Mock 52. Ag SO 2 Test5 4 + ions and one SO4 in solution. 2 ion on Hence, its Solutions is a 2 (Chemistry) : 1 type electrolyte i.e., it

gives two Ag ionisation Ksp = 4s Given

3 where s = molar solubility 2 s = 2.5 10 M Ksp = 4 (2.5 10 2 3 ) 6 5 Na SO 2 4 are 3 2 also the is 3 each.

or

= 62.5 10 = 6.25 10

53.

for

Ca(NO ) 3 2

and

Their colligative molarity same. Hence, 0.1 M Ca(NO ) and 0.1 M Na SO 2 54. solutions are isotonic. 4 1 We know PV = RT =

2 mnu for two gases

3 at the same temperature. P and V are also constant so, 1 2 1 m nu = m nu 1 1 2 3 3 55. 2 m u = m u 1 1 2 2 2

2 2

For a bcc arrangement of + + ) A B , 2(r - r = length of body diagonal of the cube. r+ + r = distance of closest approach the ions = 70.7 pm (given)

of

i.e., 2 70.7 = a3 length of cube or a = 2 70.7 3 + H

where

is

the

edge

= 81.63 pm + | H | + +

56.

+ + HOHOHO | H

Hbonding The volume of ice is higher in camparison with water, so density of ice is lower in camparison with water. 235 ----------------------- Page 235----------------------Mock 57. Test5 Solutions (Chemistry)

Consider a following rate law equation dx = k [A] dt 4 m 1.6 10 = k[0.50] [0.50) 3.2 104 = k [0.50]m [1.0]n 3.2 10 From 4 = k[1.00] m n (i) (ii) n (iii) m [B] n (i)

[1.00]

equation number (ii) and (iii) 3.2 104 k [1.00]m [1.00]m = 4 m n 3.2 10 k[0.5] [1.0] m 0 or 2 = 2 m

1 = 2 From

m = 0 equation number (i) and (ii) 3.2 104 [0.50]m[1.0]n = 3 m n 1.6 10 [0.50] [0.50] 2 = 2n 1 2 n = 2 n = 1

Rate law equation

dx (Rate equation)

0 = k[A] [B ]

dt = k[B ] 2 58. Number of electrons in

CO = 6 + 8 = 14 O = 16 + 1 = 17 2 N+ = 14 1 = 13 2 O+ = 16 1 = 15 2 CN = 6 + 7 + 1 = 14 CN .

Hence, CO is isoelectronic with 59. G = H TS For spontaneity, G = ve 60. NaOH is prepared electrolysis of NaCl. on a

large

scale

by

electrolysis NaCl(aq) at mercury cathode : Na+ + e Na Na + Hg Na.Hg (sodium amalgam) at graphite anode 2Cl (aq) Cl (g) + 2e 2 2Na.Hg + 2H O 2NaOH + H (g) + 2Hg 2 2 236 ----------------------- Page 236----------------------Mock 61. Test5 Solutions (Chemistry) Na+ + Cl

Given t1/2 = 100 second 0.693 for a k = 100 first order reaction sec 1

2.303 k = t Now . log

a (a x)

a = 100 x = 99, t99% = ? t99% = 2.303 100 0.693 = 460.6 = 0.693 = 664.64 seconds. Fe in 2.303 100 2 0.693

log

100 (100 99)

62.

Let x

be the oxidation number of K [Fe(CN) ] 3 6 3(+1) + x + 6 (1) = 0

3 + x 6 = 0 x = +3 63. The separation of colloidal particles from those of molecular dimensions is known as dialysis. Sodium is highly reactive metal and burns with an explosion in contact with water and air so it is kept stored kerosene oil. Product of AlCl is LiAlH . 3 reaction 4 between LiH it in and

64.

65.

66. 67.

Vitamin A is also known as Retinol and Axerophytol. It is a fat soluble vitamin. Half life period of the substance depend upon the nature of the substance, not on the amount, so the half life period g is also 10 hrs. Westrosol is the CCl ==CH.Cl. It is an anaesthetic. 2 2KBr + I2 2KI + Br2 does not take place electronegative than Br .

of

68.

trichloroethylene

69.

this because 2

reaction I2 is less

70.

Chlorine is slightly stronger agent than bromine. Therefore, chlorine cannot oxidise bromine to +7 oxidation

oxidising

state. So, BrCl 71.

is not formed. 7 of H SiF . 2 6

Glass is super-cooled liquid. It is soluble in HF due to formation Therefore, HF solution is used for etching of glass.

237 ----------------------- Page 237----------------------Mock 72. Test5 Solutions (Chemistry)

Normal H O solution is 5.6 Volume. So, 2 2 the volume strength of 1.5 H O solution 2 will be : 1.5 5.6 = 8.4 Volume abundent element in 2

73. 74.

The second most the earths crust is silicon. Water gas is a mixture carbon monoxide [H2 + CO]. of

hydrogen and (CO) i.e.,

it

is

75. 76.

Calomel is Hg Cl . (Mercurous chloride) 2 2 Sodium thiosulphate is oxidised iodine to give sodium tetrathionate. 2Na S O + I Na S O + 2NaI 2 2 3 2 2 4 6 Sodium tetrathionate The hardest substance in diamond due to complex structure. Velocity of electron in nth orbit of H atom Velocity of electron in first orbit v n n Given, velocity of electron in = v velocity of e v = 4 in first orbit = 4v or velocity of electron in first orbit 4th orbit = the nature tetrahedral by

77.

is

78.

79.

Due to +I effect methyl group, COOH group has lesser acidic formic acid is more acidic in comparision to other given acids. Radius 2 of nth orbit

property,

so

80.

of

H-atom

rn = a0 n where a0 = Bohr radius, radius of 1st orbit = 0.53 Now, for the radius of first excited state, n = 2, r2 = a0 2 = 0.53 4 = 2.12 238 ----------------------- Page 238----------------------ENGLISH (Mock Test-5) Directions (for Q. 136 to Q. 140) : Read the following passage carefully and answer the questions given below in the passage. Certain words/phrases in the passage are given in bold to locate them while answering some of the questions. Mock Test5 (E nglish) The world of today has achieved much, but for all its declared love for hymanity it has based itself for more on hatred and violence than on the virtues that make man human. War is the negation of truth and humanity. Sometimes, war may be unavoidable but its progeny are terrible to contemplate. Not mere killing, for man must die, but the deliberate persistent propagation of harted falsehood, which gradually become the normal habits of the people. dangerous and harmful to be guided in our lifes course by hatreds aversions, for they are wasteful of energy and limit and twist the mind, and prevent if from perceiving the truth. 136. War (a) (b) (c) (d) The (a) (b) (c) (d) is the negation of truth means : wars are evil wars do not exist war kills human beings wars spread and advertisae falsehood worlds declared love of humanity is : not to be taken seriously non existent false true 2

and and It and is

137.

138.

According to the author achievements of the world are impressive because : (a) there is nothing much to boast of (b) the world hasnt made achievement (c) they are mostly in the field of violence (d) its love of humanity is a pertence Hatred and aversions are as they are : (a) dangerous (b) harmful (c) narrow (d) barriers in seeing truth unwholesome

the not any

139.

239 ----------------------- Page 239----------------------Mock 140. Man (a) (b) (c) (d) Test5 (English)

should be guided by : materialism pratical wisdom generous human feelings scientific discoveries word

Directions (for Q. 141 to Q. 145) : In each of the following questions, choose the with similar meaning of the given word out of the given alternatives. 141. Hoard : (a) Destroy (c) Divide Moot : (a) Obsequious (c) Unexpected Graphic : (a) Pictorial (c) Mathematical Eradicate : (a) Eliminate (c) Indicate Mere : (a) Empty (c) Some (b) Hide (d) Store (b) Invisible (d) Controversial (b) Squarish (d) Geometrical (b) Dedicate (d) Complicate (b) Only (d) Complete

142.

143.

144.

145.

Directions (for Q. 146 to Q. 150) : Choose the correct synonyms. 146. Stand by : (a) support (c) interrupted (b) attacked (d) started

147.

Sets in : (a) attacked by (c) reseved Set free : (a) disparages (c) released The (a) (b) (c) (d) backdone of the team : before doing something on whom one can rely to have some selfish to divulge a secret

(b) removed (d) begins (b) rubbish (d) enfeebled

148.

149.

150.

Run across : (a) incited him to (b) meet by chance (c) pretended to be (d) strange person 240

----------------------- Page 240----------------------Answer Sheet (Mock Test-5) (ENGLISH) 136. 141. 146. (d) (d) (a) 137. 142 147. (a) (d) (d) Solution 138. 143. 148. (d) (a) (c) 139. 144. 149. (d) (a) (b) 140. 145. 150. (c) (a) (b)

Hints & 136.

According to the passage the meaning of the sentencewar is the negation of truth is that wars always spread and give rise to falsehood. Therefore, (d) is correct. ons (English) 137. According to the author declared love of humanity is considered to be a serious matter because of its false nature. Therefore, (a) is correct. the

Mock Test5 Soluti worlds

138.

According to the author achievements of the world are impressive because these achievements are not based on true grounds humanity. Therefore, (d) is correct. According to the author hatred aversions are unwholesome as they are create problems in the visibility truth. Therefore, (d) is correct. According to the passage a man should be guided by generous human feelings

the not of and of the

139.

140.

because one should be a generous and casring to others like as God. Therefore, (c) is correct. 141. Hoard means ro store up or reserve and the Store means Therefore, option (d) is correct. Moot means debate and Controversial means disputed. Therefore, option (d) is correct. Graphic means lines expressing mathematical or chermical truth or law and Pictorial means a picture having lines and curves etc. Therefore, option (a) is correct. Eradicate means Indicate means option (a) is correct. to eliminate and pointout. Therefore, 241 ----------------------- Page 241----------------------Mock 145. 146. 147. 148. 149. 150. Test5 Solutions (English) accumulation.

142.

143.

144.

Mere means only and Empty means blank. Therefore, option (a) is correct. Standby support. is an idiom that means

Sets in is an idiom that means beings. Set free is an idiom that means released. The back done of whom one can rely. the team chance. 242 means on

Run across means meet by

----------------------- Page 242----------------------MATHEMATICS (Mock Test-5) 81. A survey shows that Americans like cheese where as 76% like apples. If x% of the Americans like both cheese and apples, then : (a) x = 39 (c) 39 x 63 82. 63% of the Mock Test5 (Mat

hematics) (b) x = 63 (d) none of these

The total number of injective mappings from a set with m elements to a set with n elements m n is equal to :

(a) mn n ! (c) (n m) ! 83.

(b) nn (d) n !

Let z1 z2 be two complex numbers such that z + z and z z both are real, then : 1 2 1 2 __ (a) z1 = z2 (b) z1 = z2 __ (c) z1 = z2 (d) z1 = z2 If a, b, c are then : (a) a = b c (c) a b c in A.P. as well as in G.P.,

84.

(b) a b = c (d) a = b = c equation b

85.

If the roots of the ax2 + bx + c = 0 are real and distinct, then : (a) both roots are greater than

2a b (b) both roots are less than 2a b (c) one of the roots exceeds 2a (d) 86. If none of these the equations x2 ax + b = 0 and

x2 + bx a = 0 have a common root, then : (a) a = b (c) a + b = 1 87. In how many hung from 5 picture (a) 2520 (c) 2522 (b) a + b = 0 (d) a b = 1 ways 7 pictures nails on a wall ? (b) 2500 (d) none of these can be

88.

The number of permutations of all the letters of the word MISSISSIPPI is : (a) 46504 (b) 34650 (c) 77880 (d) none of these th th If the co-efficients of r and (r + 1) terms in the expansion of (3 + 7x)29 are equal, then r is equal to : (a) 15 (c) 14 (b) 21 (d) none of these 243

89.

----------------------- Page 243----------------------Mock 90. Test5 (Mathematics)

4 The co-efficeint of x in the expansion of 10 x 3 2 2 is equal to : x 405 (a) 256 450 (c) 263 x 2 1 + y , then (b) 2 1 (d) 2 matrix (e the 1 (e x + e and value x e x ) n N, of y x ) is (b) 259 (d) none of these 504

91.

If e equal

= y + to : x x (a) e e x (c) e + e x

92.

If A then An

is a symmetric is :

(a) symmetric (b) skew-symmetric matrix (c) a diagonal matrix (d) none of these sin 93. 2 x cos 2 x 1 1 is equal 2 to :

cos2 x sin2 x 10 12 (a) 0 (b) 12 cos2 x 10 sin2 x 2 (c) 12 sin x 10 cos (d) 10 sin 2x

2 x 2

94.

Let P and Q be points on the line joining A (2, 5) and B (3, 1) such AP = PQ = QB. Then the mid PQ is : 1 1 (a) 2, 3 (b) 2, 4 (c) (2, 3) (d) (1, 4) 2

that point of

95.

If the parabola intersect at (16, 8) at is equal to : (a) tan 1 3 5

y an

= 4x and x = 32y angle , then 1 4 5

(b)

tan

(c) 1 96. If f (x) = x x (a) f (x) + f (x) (c) f(x) f (x)

(d) 2 2 , x 0, then f (x ) equals : (b) f (x) f (x) (d) none of these 244

----------------------- Page 244----------------------Mock 97. lim x 0 (a) 4 (c) 10 98. The function f (x) to : (a) discontinuous at (b) continuous at x = (c) differentiable (d) none of these y 99. If x = 0 (a) 2y (c) 8y 100. The 2 1 = sin x = 0 0 at x = 0 2 d y Test5 (Mathematics)

sin 4x 1 1 x is equal to : (b) 8 (d) none of these 1 (cos x) is equal

2 dt, then 2 is equal to : 1 + 4t dx (b) 4y (d) 6y to

equations of those tangents 2 4x 9y = 36. Which are perpendicular to the straight line 5x + 2y 10 = 0, are : (a) 5 (y 3) = 2 x (b) 2x 5y + 10 218 = 0 (c) 2x 5y 10 218 = 0 (d) none of these 1x 117 2

101.

The (a) e

maximum

value of x (b) ee

is equal to :

(c) e1 e 102.

(d) (1 e)1 e the : (b) 2 (d) value of the integral integral

The value of 2 log tan x dx is equal to 0 (a) 4 (c) 0

103.

The 1 1

log (x + x2 + 1) dx is equal to :

(a) 0 (c) log 1 2 104. The area sides a and b (a) |a b | of is equal the

(b) log 2 (d) none of these triangle to 1 (b) 2 : |a b | with adjacent

(c) |a | |b |

(d) none of these 245

----------------------- Page 245----------------------Mock 105. If Test5 (Mathematics) vectors, then

a , b are any two 2 |a b | is equal to : 2 2 2 (a) |a | |b | + (a . b ) 2 2 2 (b) |a | |b | (a . b ) 2 2 (c) |a | |b | + (a . b ) (d) none of these

106.

The

x 2 = y + 1 = z 1 intersects 3 2 1 2 the curve xy = c , z = 0, if c is equal to : (a) 1 (b) 1 3 (c) 5 (d) none of these

lines

107.

If there are 6 girls and 5 boys who sit in a row, then the probability that no two boys sit together is : 6 ! 6 ! 7 ! 5 ! (a) (b) 2 ! 11 ! 2 ! 11 ! 6 ! 7 ! (c) (d) none of these 2 ! 11 ! Probability that a student will succeed in I.I.T. entrance test is 0.2 and that he will succeed in Roorkee entrance test is 0.5. If the probability that he will successful at both the places is 0.3, then the probability that he does not succeed at both the places is : (a) 0.4 (c) 0.2 (b) 0.3 (d) 0.6

108.

be

109.

In a triangle ABC, sin A cos B = cos C, then the value of angle B is : (a) 2 (c) 4 If (d) 6 then x is sin sin1 1 + cos 1 x = 1, 5 (b) 0 (d) 1/5 246 (b) 3

110.

equal to : (a) 1 (c) 4/5

----------------------- Page 246----------------------Mock 111. Test5 (Mathematics)

The angle of elevation of the top of an incomplete vertical pillar at a horizontal distance of 100 mt. from its base is 45. If the angle of elevation of the top of the complete pillar at the same point is to be 60, then the height of the incomplete pillar is to be increased by : (a) 502 mt (b) 100 mt (c) 100 (3 1) mt (d) 100 (3 + 1) mt In a moderately skewed distribution, the values of mean and median are 5 and 6 respectively. The value of mode in a situation is approximately equal to : (a) 8 (b) 11

112.

such

(c) 16 113. 2 If 1, , be the cube roots of 2 5 (1 + ) + (1 + (a) 30 (c) 34

(d) none of these unity, then 2 5 ) is equal (b) 32 (d) none of these between

to

114.

The arithmetic mean positive numbers is : (a) greater than geometric mean (b) equal to geometric mean (c) less than geometric mean (d) greater than or equal mean 1 1 + 2 + 2 ! (a) e (c) e 4 3 ! + 4 ! 1 + 2 + 3

two

to

geometric

115.

+ is equal to : (b) e 2 (d) none of these

116.

The foci of a hyperbola coincide with the 2 2 foci of the ellipse x + y = 1. Find 25 9 equation of the hyperbola, if eccentricity is 2 : (a) 3x2 y2 = 12 (b) 3x2 + y2 = 12 2 (c) y 3x 2 = 12 (d) none of these balls 2 ! 4 ! of these

the the

117.

The number of ways in which 12 can be divided between two friends, one receiving 8 and the other 4, is : 12 ! 12 ! (a) (b) 8 ! 4 ! 8 ! 12 ! (c) (d) none 8 ! 4 ! 2 !

247 ----------------------- Page 247----------------------Mock 118. Test5 (Mathematics) of

50 In the expansion of (1 + x) , the sum the co-efficients of odd powers of x is : 49 (a) 0 (b) 2 50 51 (c) 2 (d) 2

119. If

the (a) (c) 120.

b + c a + b c + a value of k 1 3

c + a b + c a + b

a + b c + a b + c (b) 2 (d) 4 = k

a c b

b a c b

c , then a

is equal to :

2 2 The circles x + y + 2x 2y + 1 = 0 x2 + y2 2x 2y + 1 = 0 touch each other : (a) externally at (0, 1) (b) internally at (0, 1) (c) externally at (1, 0) (d) internally at (1, 0) 4 sin 4 x x + cos 2 x + x tan x

and

121.

The

function f (x) =

is :

(a) even (b) odd (c) periodic with period (d) periodic with period 2 122. The value of lim x (a) b log a (c) b x a x dx is equal x a (a) + c 2 log a x (c) 2a . log a + c 4 124. If In = 0 In + 2 + In 1 (a) n 1 (c) n + 1 equals : 1 (b) n 1 1 (d) n + 2 tann x dx, n N, then (b) log a (d) none of these 2a + c x a sin b is (a > 1) : x a (b) a log b (d) none of these

123.

to

: x

125.

Solving the dy + (sec x) y = tan x, we dx (a) y (sec x + tan x) = sec x (b) y (sec x + tan x) = sec x (c) y (sec x + tan x) = sec x (d) none of these

differential get : + tan x + x + c tan x + x + c + tan x x + c

equation

248 ----------------------- Page 248----------------------Answer Sheet (Mock Test-5) (MATHEMATICS) 81. 86. 91. 96. 101. 106. 111. 116. 121. (c) (d) (b) (d) (c) (c) (c) (a) (b) 82. 87. 92. 97. 102. 107. 112. 117. 122. (c) (a) (a) (b) (c) (c) (a) (b) (c) Solution 83. 88. 93. 98. 103. 108. 113. 118. 123. (b) (b) (a) (b) (a) (d) (b) (b) (a) 84. 89. 94. 99. 104. 109. 114. 119. 124. (d) (b) (a) (b) (b) (a) (d) (b) (c) 85. 80. 95. 100. 105. 110. 115. 120. 125. (c) (a) (a) (d) (b) (d) (b) (a) (c)

Hints & 81.

Let A denote the set of Americans who like cheese and let B denote those who like apples. Let the population of America is 100. Then, Mock Test5 Solutio ns (Mathematics) n (A) = 63 and n (B) = 76 Now, n (A B) = n (A) + n (B) n (A B) n (A B) = 63 + 76 n (A B) n (A B) = 139 n (A B) But n (A B) 100, n (A B) 39 Further, A B A and A B B imply that n (A B) n (A) and n (A B) n (B), that is n (A B) 63 and n (A B) = 76 Hence, 39 n (A B) 63 39 x 63 82. Let A = {a , a , a 1 2 m

so

that

},B = {b , b , , b }, 1 2 n

and let f : A B. The possible choices for f (a ) are n. 1 Having fixed f (a ), the 1 possible choices

for f (a ) are n 1, and so on. Thus, there 2

are n ! n (n 1) (n 2) (n (m 1)) = (n m) ! injective mappings from A to B. 249 ----------------------- Page 249----------------------Mock 83. Test5 Solutions (Mathematics)

Let z1 = a + ib, z2 = c + id. Then z + z is real 1 2 (a + c) + i (b + d) is real b + d = 0 z z is real 1 2 d = b ...(1)

(a + ib) . (c + id) is real 2 ac + iad + ibc + i bd is real 2 ac + i (ad + bc) bd is real [i (ac bd) + i (ad + bc) is real ad + bc = 0 a () + bc = 0 [from (1)] a + c = 0 a = c z1 = a + ib = c id [from (1) and (2)] __ = z2 . . [ . z2 = c + id] 250 ----------------------- Page 250----------------------Mock 84. Test5 Solutions (Mathematics) ...(2) = 1]

It is given that : a, b, c are in G.P.

b2 = ac

...(1) ...(2)

and a, b, c are in A.P. 2b = a + c 2 Now, b = ac and 2b = a + c

a + c = b 2 b = ac a + c = ac 2 2 a + c = ac 2 2 (a + c)

and

4 2 2 Putting a = c in Hence, 85. The roots of the given a + c a (a + c) 2 + 2ac 4ac = 0 2 + c 2

= ac

4ac = 0

2ac = 0 2 (a c) = 0 a = c

(2), we

get 2b = a + a 2b = 2a b = a a = b = c

equation

2 ax + bx + c = 0 are b2 4ac + b2 4ac = and = 2a 2a Since , are real and distinct, therefore 2 b b 4ac > 0. Now if, a > 0, then > 2a b if a < 0 then > 2a b Thus, one of the roots exceeds . 2a 86. Let be the common roots of the given equations. Then

2 a + b = 0 and 2 a + b = 2 + b a (a + b) = (a + b) (a + b) (a + b) = 0 a + b = 0 or = 1 2 For = 1,

2 + b a = 0

. . . = a + b = 0 1 a + b = 0 a b = 1.

(a + b) (a + b)

251 ----------------------- Page 251----------------------Mock 87. Test5 Solutions (Mathematics) on a number at a

The number of ways in which 7 pictures can be hung from 5 picture nails wall is same as the arrangements of 7 things, taking 5 time. Hence, the required number = 7P5 7 ! = (7 5) ! 7 ! = = 2 ! 7 6 5 4 3 2 2 = 2520.

of

88.

In the word MISSISSIPPI there are letters of which 4 are Ss, 4 are Is and 2 are Ps. So, the number of permutations 11 ! = =

11

4 ! 4 ! 2 ! 11 10 9 8 7 6 5 4 3 2 4 3 2 4 3 2 2 29 r C 3 r Binomial) 29 (7x) r

= 34650. 29 89. T r + 1 (expanding by =

. . [ . in (3 + 7x) 29 = ( Cr 3

, n = 29, a = 3, b = 7x] 29 r r 7 ) x r th

ar = co-efficient of (r + 1) 29 = C r

term

29 r

r th

and ar 1 = co-efficient of r 29 = Now, 29 Cr 3 Cr 1 3 30 r

term

r 1 (given)

ar = ar 1 29 r 29 Cr 3 = 29Cr 1 30 r 3 = r 7 210 7r = 3r 210 = 10r r = 21. 7

r =

29

Cr 1 3

30 r

r 1

252 ----------------------- Page 252----------------------Mock 90. Test5 Solutions (Mathematics) th term. 3 r 2 x r 10

Suppose x We have,

4 occurs in (r + 1) Tr + 1 = 10Cr x 10 r 2 10 = Cr x 10 3r

r (3) 2

4 So, x

10 3r = 4 3r = 6 r = 2 rd occurs in 3 term and its

co-efficient is 10 C2 (3) 10 = C2 2 10 9 3 2 2 5 9 9 8 2 405 = 256 91. We have x 2 e = y + 1 + y x (e y) = 1 + y 2 2 = 1 + y x 2ye = 1 2x e y = 2ex 1 = 2 92. Since A is symmetric, therefore A = A Now n (A ) = (A ) So, A n = A n . . [ . A = A] (e x e x ) 1 2 8 2 2 2 3 8 2 10 2 . . [ . r = 2]

2x e 2ye

x 2 (e y) = 1 + y x + y 2x e 2

n is also a symmetric matrix.

sin 93.

2 x cos

2 x 1 sin2 x 12 2 2 x + cos 2 2 x + sin 2 x x

1 2 2 cos 2 sin 12 x 1 2 [C 1 C 1 2 x + cos x = 1] 253 [C and C 1 are identical] 3 + C ] 2 x 1

cos2 x 10 sin = cos

1 = 1 2 = 0.

cos2 x 2 sin 12 x

1 1 2 . . [ . sin

----------------------- Page 253----------------------Mock 94. Test5 Solutions (Mathematics)

Let R be the mid point of PQ. Then, PR = RQ AP + PR = AP + RQ AP + PR = BQ + RQ AR = BR R is the mid point of PQ So, co-ordinates of R are i.e. 1 , 3 2 2 = 4x at 2 + 3 2 , 5 + 1 2 . . [ . AP = BQ]

95.

The slope of the tangent to y (16, 8) is given by dy 4 m1 = dx = 2y 16, 8 (16, 8) 2 1 = = 8 4 2

The slope of the tangent to x (16, 8) is given by m = dy = 2x = 1 2 dx 32 (16, 8) (16, 8) 1 1 4 3 tan = = 1 5 1 + 4 = tan1 3 5 96. We 2 have f (x ) = x 2 = x + = x + Clearly, (a), (b) 97. sin 4x lim x 0 1 1 x sin 4x = lim x 0 1 1 x = = lim x 0 lim 1 2 x 1 x x 1 f (x) x true. 1 x

= 32y

at

and (c) cannot be

(1 + 1 x) (1 + 1 x)

sin 4x (1 + 1 x) 1 (1 x)

sin 4x (1 + 1 x) x x 0 lim 4 sin 4x (1 + 1 x) 4x x 0 . . . lim x 0 sin x x = 1 254

= 4 1 2 = 8.

----------------------- Page 254-----------------------

Mock 98.

Test5

Solutions

(Mathematics)

We have, (LHL at x = 0) = lim x 0 1 = lim sin (cos (h)) h 0 = lim sin1 (cos h) h 0 . . [ . f (x) = sin 1 (cos x) and cos () = cos ] 1 = sin 1 = sin (RHL at x = 0) = lim x 0 1 = lim sin h 0 1 = sin = and 2 f (0) = sin 1 (cos 0) 1 = sin = 2 (LHL at x = 0) = (RHL at x = 0) = f (0) So, f (x) is continuous at x = 0 sin x Now, f (x) = 2 = 1 cos x sin x sin x = 1, = sin x = 1, sin x (LHD at x = 0) = 1 and (RHD at x = 0) x > 0 sin x |sin x | x < 0 (1) (1) (cos h)= sin 1 (cos 0) 1 = 2 f (x) = lim + h 0 f (0 + h) (cos 0) f (x) = lim h 0 f (0 h)

= 1 Hence, f (x) is not differentiable at x = 0. y 99. x = 0 1 2 dt 1 + 4t dx 1 = dy 2 1 + 4y dy 2 = 1 + 4y dx 4y = 2 dx 2 d y dx 2 = 1 + 4y 255 ----------------------- Page 255----------------------Mock 100. Test5 Solutions 2 4x 8x 18y dx dy dx Slope of the tangent = 9y For this tangent to be perpendicular the straight line 5x + 2y 10 = 0 we must have 4x 9y Putting we get 2 this value to = 9y 4x 4x 9y dy (Mathematics) 2 = 36 = 0 2 dx 1 + 4y 4y 2 . 1 + 4y 2 = 4y. dy

2 d y

5 = 1 2 10x y = 9

of y

in

4x2 9y2 = 36,

4x2 9 10x = 36 9 2 2 4x 2 36x 100x 2 64x = 324, which does not have real roots. Hence, at no point on the given curve can the tangent be perpendicular to the line. 1x 101. Let f (x) = x 1x Then, f (x) = x = xx (log x + 1) f (x) = 0 xx (log x + 1) = 0 (log x + 1) = x x = e x log x . 9 2 = 324 100x = 36

given

log x + 1 = 0 log x = 1 1 x = e 1 Clearly, f (x) < 0 at x = e Hence, f (x) = xx is maximum x = e1. Then 102. Let I = 0 2 Then, I = 0 2 I = 0 Adding (1) and (2) 2I = 0 I = 0 log cot x dx log tan 2 x dx maximum value is e1 e. 2 log tan x dx

for

...(1)

...(2)

256 ----------------------- Page 256-----------------------

Mock

Test5

Solutions

(Mathematics)

103.

2 Let f (x) = log (x + x + 1 ). Then f (x) + f (x) = log (x + x2 + 1) + log (x + x2 + 1) 2 2 = log (x + 1 x ) = log 1 = 0 f (x) =  (x) for all x (1, 1) f (x) is an odd function (1, 1) 1 f (x) dx = 0. 1

defined

on

104.

The

105.

area of the triangle with 1 sides a , and b is |a b |. 2 We have |a b | = |a | |b | sin 2 2 2 |a b | = |a | |b | sin 2 2 2 = |a | |b | (1 cos 2 2 2 = |a | |b | |a | |b 2 2 = |a | |b | ( |a | |b 2 2 2 = |a | |b | (a . b ) Hence, |a b | 2 = |a | 2 |b | 2

adjacent

2 ) 2 | cos | cos ) 2 2

2 (a . b )

106.

At the point on the line where it intersects the curve, we have z = 0, so that x 2 = y + 1 = 0 1

3 2 1 x 2 = 1 and y + 1 = 1 3 2 x, y in curve

x = 3 + 2 = 5, y = 2 1 = 1 Putting the values of 2 2 xy = c , we get 5 . 1 = c 107. c = 5

Six girls and five boys can sit in 11 ! ways. exhaustive number of cases = 11 ! Six girls can sit in a row in 6 ! ways and

in

row

in each such arrangement there places between them in which 5 boys can 7 be seatd in P ways. Therefore, the total 5 number which no 7 sit together = 6 ! P . 5 Hence, required probability 7 6 ! = 11 ! P5 = 2 ! 11 ! 6 ! 7 ! of ways in two boys

are

257 ----------------------- Page 257----------------------Mock 108. Test5 Solutions (Mathematics) that the student I.I.T. entrance test that he is selected Then __ and B in

Let A denotes the event is selected in denotes the event Roorkee entrance test.

P (A) = 0.2, P (B) = 0.5 and P (A B) = 0.3 __ Required probability = P (A B) = 1 P (A B) = 1 (P (A) + P (B) P (A B)) = 1 (0.2 + 0.5 0.3) = 0.6. 109. We have, sin A cos B = cos C sin A = cos B + cos C

2 sin A cos A = 2 cos B + C cos B C 2 2 2 . . . sin 2 = 2 sin cos and cos A + cos B 2 2 = 2 cos A + B cos A B 2 2 sin A cos A = 2 sin A cos B C 2 A cos 2 = cos 2 2 B C . . 2 . sin 2 2 A 2

+ 0 0

A 2 But =

B C 2 A = B C A + B + C = ...(1) ...(2)

because ABC is a triangle Therefore B C + B + C = [from (1) and (2)] 2B = 110. B = 2 sin sin 1 1 + cos1 x = 1 5 1 1 sin 5 1 1 sin 5 1 1 sin 5 . . . sin 1 x = 5 258 ----------------------- Page 258----------------------Mock 111. Test5 Solutions (Mathematics) we 1 x + cos 1 x = 2 = sin = 2 1 x cos 1 x + cos 1 x = 2 . . . sin 1 1 = 2

Let BC be the incomplete and BD be the complete pillar. In s ABC and ABD, have BC tan 45 = and tan 60 =

BD AB

AB BC = AB tan 45 BC = 100 mt and BD = AB tan 60 = 100 3 BC + CD = 100 3 100 + x = 100 3 x = 100 (3 1) mt The length of the incomplete pillar is

to be increased by 100 (3 1) mt. 112. For a moderately skewed distribution, we have, Mode = 3 Median 2 Mean Mode = 3 6 2 5 . . [ . Median = 6 ; Mean = 5] = 18 10 = 8 2 5 113. (1 + ) 2 = (1 + = ( ) ) 5 + ( 2 . . [ . 1 + 5 = (2) 5 = 32 32 9 + (2 ) 10 = and 1 + = ] 2 5 + (1 + ) 5 + (1 + ) 2 2 5 ) 2 2 5 2 5

3 2 = 32 ( . ) 32 ( . ) 2 = 32 32 3 . . [ . 2 = 32 ( = 32 1 = 32 ) = 1 and

9 = ( )

3 3 = 1]

. . [ .

2 + = 1] 259

----------------------- Page 259----------------------Mock 114. Test5 Solutions (Mathematics)

Let a and b be Then A.M. = Now

two positive numbers. 1 (a + b) and G.M. = ab 2 a + b = ab 2

1 = [a + b 2 a b] 2 = 1 (a b)2 0 2 a + b i.e., 115. ab 2 A.M. G.M. is + 2 3 ! 1 + 2 + 3 4 ! + the given series, (1 + 2 + 3 + + n) (n + 1) ! 1 . n (n + 1) 2 = 1 = 2 n ! 1 = . 1 2 1 and so on. 3 ! 1 . 2 + + 3 ! (n 1) ! (n 1) ! 1 + + 1 = 1 ! 2 ! 1 1 (n + 1) ! n

The given series 1 1 2 ! + The nth term of Tn =

2 (n 1) ! Putting n = 1, 2, 3, 4 etc, we get 1 1 1 T1 = = ; T2 = 2 0 ! 2 1 1 1 T3 = ; T4 = 2 2 ! 2 Given series 1 = 2 1 = 2 e = 2 . 1 + 1 + 2 ! + 2 1 + 2 1 . 2 ! 1 + 1 + 2 1 . 3 ! 1 1 + + 1

260 ----------------------- Page 260----------------------Mock Test5 Solutions (Mathematics)

116.

The equation of the ellipse is of the form 2 2 x + y = 1, where a2 = 25 and b2 = 9. 2 2 a b Let e be the eccentricity of the ellipse. 2 2 2 Then, b = a (1 e ) 2 b2 e = 1 2 a = 1 4 e = 5 9 = 16 25 25

and so

ae = 5 4 = 4 5 are the

Thus, foci of the ellipse ( ae, 0), i.e. ( 4, 0). Consequently, foci of the hyperbola are (4, 0). Now, c = 4 ae = 4 4 a = 2 Also, The hyperbola is 2 x 4 or 117. 2 2 2 c = a + b 2 2 2 b = (c a ) = 16 4 = 12 required equation of 2 y = 1 12 2 3x y 2 = 12 = 2

. . [ . e = 2]

the

12 balls can be distributed between two friends A and B in two ways (i) friend A receives 8 and B receives 4. (ii) friend B receives 8 and A receives 4. So, the required number of ways 12 ! 12 ! = + 8 ! 4 ! 4 ! 8 ! = 2 12 ! 8 ! 4 !

50 50 118. We have (1 + x) = r = 0 sum of the co-efficient of odd powers of x 50 = 1 = 2 1 = 2 261 ----------------------- Page 261----------------------Mock 119. Test5 Solutions (Mathematics) (2 50 ) = 2 49 . [ C1 + 50 C0 + 50 C3 + + 50 C1 + + 50 C49 50 C50] 50 r Cr x . Therefore,

We have, b + c c a + b b c + a a 2 (a + b + 2 (a + b + 2 (a + b + + a + c + b c) c) c) a + b c + a b + c c + b + a + a b c = k c a b , b c a a a + b a = k c c + a c b b + c b [C 1 a + b + c 2 a + b + c a + b + c [Applying C 2 c + a b + c a + b C 2 a + b c + a b + c C , C 1 3 C 3 on L.H.S.] we get a + b + c b c a b c = k b c C 1 C 1 a c + C 2

b a c + C ] 3 b a

c b a

c b a

2 a + b + c a b

= k c b c a a b

a + b + c c a Now, applying C C + C + C on L.H.S., we get 1 1 2 3 a 2 c b a 2 c b [Takingcommon 120. from c a b c a b c a = k b c a b c a = k b c a b C and C ] k = 2 b a c b a c 2

c b a c b a 3 form centre circle circle and

The equation of circle of the 2 2 x + y + 2gx + 2fy + c = 0 has C ( g, f) Therefore, the centre of the 2 2 x + y + 2x 2y + 1 = 0 is C1 (1, 1) and centre of the 2 2 x + y 2x 2y + 1 = 0 is C2 (1, 1) both have radii equal to 1. We have 2 2 C1 C1 = (1 + 1) + (1 1) = 2 Sum of radii = 1 + 1 = 2 So, the two circles touch each externally. The equation of the common tangent is obtained by subtracting two equations. The equation of the common tangent is 2 (x + y 2 + 2x 2y + 1) (x 4x = 0 x = 0 Putting x = 0 in the equation of the either circle, we get 2 2 + y 2

other the

2x 2y + 1) = 0

y 2 Hence, touch the (y 1) points

2y + 1 = 0 = 0 where the two y = 1 circles 262

is (0, 1).

----------------------- Page 262----------------------Mock 121. Test5 Solutions (Mathematics)

We have, 4 sin f (x) = x + cos 2 x + x tan x 4 f ( x) = x + cos 2 x + x tan (x) sin 4 x 4 x

4 4 sin x + cos x 2 = f (x) x + x tan x

So, f (x) is an odd function. f (x) is not a periodic function due to the presence of x in the denominator. 122. lim a x x sin b x a sin b x a = lim x . b b x a = 1 . b = b. x 123. We a have x dx get . . . lim x sin = 1

1 Putting x = t and 2x dx = dt, we x a 1 t x dx = 2 a dt

t a = 2 log a x a = 2 log a 124. We have In + 2 + In 4 = 0 4 n = 0 4 n = 0 1 = 0 where t = tan x 2 dt = sec = n + 10 = n + 1 263 ----------------------- Page 263----------------------Mock 125. Test5 Solutions (Mathematics) 1 tn x dx 1 tn dt tan x sec 2 x dx tan x (1 + tan 2 x) dx tann + 2 x dx + 0 4 tann x dx + c + c

The differential equation dy + (sec x) y = tan x is the dx dy + Py = Q dx where P = sec x, Q = tan x

form

Pdx I.F. = e sec x dx = e log (sec + tan x) = e = (sec x + tan x) So, the required solution is given by y I.F. = {(Q I.F.)} dx + c or y (sec x + tan x) = tan x (sec x + tan x) dx + c or y (sec x + tan x) 2 or = sec x tan x dx + tan y (sec x + tan x) = sec x tan x dx 2 + sec x dx dx + c 2 x 1] . . 2 [ . tan x = sec or y (sec x + tan x) = sec x + tan x x + c 264 ----------------------- Page 264----------------------PHYSICS (Mock Test-5) 1. If AB = | A B |, and B is : (a) zero (c) 2. the angle between A (b) 4 (d) 2 a prism Mock Test5 (Physi x dx + c

cs) The angle of minimum deviation for prism of refractive index 3/2 is equal to the angle of prism. The angle of is : (a) 42 (b) 82 (c) 41 (d) 31

3.

When 100 volt D.C is applied across a solenoid, a current of 1 A flows When 100 volt A.C is applied across the same coil, the current drops to 0.5A. If

in

it.

the frequency of the A.C source is 50Hz. The impedance and inductance solenoid are : (a) 100 and 0.55 H (b) 200 and 0.8 H (c) 200 and 1.0 H (d) 200 and 089 H 4. Three equal resistances, each value R, are joined as shown in the figure. The equivalent resistance between A and B is : (a) R R (c) 2 5. The and (d) 3 (b) R 2R of

of

the

same

velocity of sound in air is 330 m/s the velocity of light in air 8 3 10 m s. What frequency in Hz, BBC station transmits at 1500 broadcast ? 6 (a) 0.22 Hz (b) 5 10 Hz 5 3 (c) 2 10 Hz (d) 595 10 Hz A spherical solid ball of 1 radius 3 cm is rotating passing through its angular velocity of kinetic energy of rotation is : 9 (a) J (b) 20 5 (c) J (d) 27 mass and about an centre with 50 rad/sec. 9 J 25 3 J 8 kg

is a m

6.

axis an The

265 ----------------------- Page 265----------------------Mock 7. Test5 (Physics) portion of paint as

How will the image formed by a convex lens be affected if the central the lens is painted with black shown in figure ?

(a) The central position of the image will be absent (b) No image is formed by the remaining portion of lens (c) The full image will be formed, it will be less bright (d) There will be two images, one due to upper half and other due to lower half

but

8.

A rectangular coil of of cross-section 25 cm2

20

turns has

end a

area resistance is coil

of 100. If a magnetic field which perpendicular to the plane of the changes at a rate of 1000 tesla/sec. The current in the coil is : (a) 10 A (b) 50 A (c) 0.5 A (d) 5.0 A 9.

Two charges + 16 and 9 units respectively are placed at A and at B 7 cm apart in air. The point on the line AB where the potential is zero, will be from A at the distance of : (a) 4.48 cm (c) 7.5 cm (b) 7 cm (d) 4 cm mole of a monoatomic gas

10.

If =

one 5 3

mixed with one mole of a diatomic

4 gas = , 5 mixture is : (a) 1.53 (c) 1.50 11.

then

the

value

of

for

the

(b) 1.40 (d) 3.07 characteristic irrespective (b) velocity (d) frequency 266 of a

The distinguishable monochromatic light waves of medium is : (a) wavelength (c) intensity

----------------------- Page 266----------------------Mock 12. Test5 3 (Physics) cm, eye-piece the of length effective of the

In a Ramsdens focal length eye-piece is : (a) 1.5 cm (c) 3 cm The unit of (a) C.s (c) A

(b) 8/3 cm (d) 4 cm

13.

displacement current is : (b) C (d) V/m

14.

A small piece of wire is passed through the gap between the poles of a magnet 8 in 0.1 sec. An emf of 4 10 V is induced in the wire, the magnetic flux between

the poles is (a) 10 Wb (c) 4 10 15. 9 Wb

(b)

0.1 Wb 2 Wb

(d) 4 10

What is immaterial for an electric fuse ? (a) Its length (b) Current flowing (c) Specific resistance (d) None of these Thermal radiations waves belonging to : (a) ultraviolet region (b) visible region (c) gamma region (d) infra-red region are electromagnetic

16.

17.

Musical interval between two frequencies 320 and 240 is : (a) 1.33 (b) 80 (c) 7 (d) 1.78 The velocity dimensional is : 0 2 1 (a) [M L T ] 0 2 1 (c) [M L T ] formula (b)

notes

of

18.

for

areal

0 2 1 [M L T ] 0 2 2 (d) [M L T ] of

19.

The escape velocity on the surface earth is 11.2 km/s. If mass and radius of a planet is 4 and 2 times respectively, than that of earth, the escape velocity on the planet : (a) 11.2 km s (b) 1.12 km s (c) 22.4 km s (d) 15.8 km s Two balls of different mass have the same kinetic energy. The ball have the greater momentum, will be : (a) both having equal masses (b) lighter one (c) heavier one (d) none of these

20.

267 ----------------------- Page 267----------------------Mock 21. Test5 (Physics)

A body is moved along a straight line by a machine delivering constant power. The distance moved by the body in time t is proportional to : (a) t2 (b) t1 2 3 2 (c) t (d) t 3 4

22.

At NTP, the ratio of molecules in 1cc oxygen to that in 1cc of hydrogen is : (a) 1 : 1 (b) 1 : 4 (c) 1 : 16 (d) 1 : 2

of

23.

Out of the following spectral series hydrogen spectrum, the one which lies wholly in the ultraviolet region, is : (a) Paschen series (b) Lyman series (c) Balmer series (d) Brackett series A can filled with water is revolved vertical circle of radius 4 m and the water does not fall down. The revolution will be : (a) 8 s (b) 1 s (c) 10 s (d) 4 s in a time

of

24.

of

25.

The mass number of He (Helium) is 4 and that of sulphur is 32. The radius of sulphur nucleus is larger than that helium, by ........ times. (a) 2 (b) 4 (c) 8 (d) 8 Two sources give interference which is observed on a screen, D distance apart from the sources. The fringe width is 2. If the distance D is now doubled, then fringe width will : (a) become (b) become 2 (c) 4 (d) remain the same pattern

of

26.

27.

The resistance of wire of uniform diameter d and length l is R . The resistance of another wire of the same material, but diameter 2d and length 4l will be : (a) 2R (b)R (c) R 4 (d) R 2 A 1 metre stick is moving at the speed equal to half the velocity of What will be the length of the stick at this speed ? (a) 1 m (b) 0.866 m (c) 1.134 m (d) 2.134 m light.

28.

268 ----------------------- Page 268----------------------Mock 29. Test5 (Physics)

A bubble rises from bottom of a lake 90 m deep. On reaching the surface, its volume becomes (take atmospheric

pressure

correspond up (a) 18 times (c) 8 times 30.

to

10

m of water) : (b) 4 times (d) 10 times

The values of the horizontal component of earths magnetic field and angle of dip 5 2 are 1.8 10 Wb m respectively at some place. intensity of earths magnetic field at that place will be : 3 2 (a) 3.18 10 Wb m 5 2 (b) 3.67 10 Wb m (c) 5 103 Wb m2 (d) 2.08 105 Wb m2 Two triodes A and B amplification factor of 40. The plate and load resistances are 4k respectively. If an amplifier made using only one of them with a load resistance of 8 k. The ratio of gain obtained from them, will be : (a) 5 (b) 4/9 (c) 4/3 (d) 2/3

and The

30 total

31.

have

the

and 8k circuit is voltage

32.

Hydrogen gas is expanded adiabatically at an initial temperature of 300 K so that its volume is doubled. The temperature of the hydrogen gas 0.4 [ = 1.4, and (2) = 1.32] (a) 47C (b) 454.76 K (c) 500.30 K (d) 227.27 K During an adiabatic process, the pressure of a gas is found to be proportional to cube of its absolute temperature. Then C ratio of = p for the gas is : C v (a) 4/3 (b) 5/3 (c) 3/2 (d) 2 The root mean square velocity hydrogen molecules at 300K is 1930 m/s. The rms velocity of oxygen molecules at 1200K will be : (a) 965 m/s (b) 765 m/s (c) 1065 m/s (d) 865 m/s

final is :

33.

34.

of

269 ----------------------- Page 269----------------------Mock Test5 (Physics)

35.

Two rodes of different materials having coefficients of linear expansion 1 and 2 and Youngs modulii Y1 and Y2 respectively are fixed between two rigid massive walls. The rods are heated such that they undergo the same increase in temperature. There is no bending of rods. If : = 2 : 3, the thermal stress 1 2 developed in the two rods are equal provided Y : Y equal to : 1 2 (a) 2 : 3 (c) 1 : 1 (b) 4 : 9 (d) 3 : 2 trolley is

36.

A man weighing 80 kg is standing on a trolley weighing 320 kg. The resting on frictionless horizontal rails. If the man start walking on the trolley along the rails at speed 1 m/s, then after 4 sec his displacement, relative to ground will be : (a) 4.5 m (b) 5 m (c) 8.0 m (d) 3.2 m

37.

Water rises to a height of 10 cm in a capillary tube and mercury falls to a depth of 3.42 cm in the same capillary tube. If the density of mercury is 3 13.6 kg/m and angle of contact is 135. The ratio of surface tensions for water and mercury is : (angle of contact for water and glass is 8) (a) 1 : 0.5 (b) 1 : 65 (c) 1.5 : 1 (d) 1 : 3 A ball falls freely from a height and after striking the ground, rebounds to starting point. Which of the following graphs (fig) shows the variation in vertical acceleration a with time t : its its 270

38.

----------------------- Page 270----------------------Mock 39. Test5 (Physics) of

A person is suffering from the defect astigmatism. Its main reason is : (a) distance of the eye lens from ratina is increased (b) power of accommodation of the eye is decreased (c) the cornea is not spherical (d) distance of the eye lens from ratina is decreased

40.

A liquid with coefficient of volume expansion is filled in a container of a material having the coefficient of linear expansion . If the liquid over flows on heating : (a) = 3 (b) > 3 (c) < 3 (d) > 33 271

----------------------- Page 271----------------------Answer Sheet (Mock Test-5) (PHYSICS) 1. 6. 11. 16. 21. 26. 31. 36. (b) (d) (d) (d) (c) (c) (c) (d) Hints & 2. 7. 12. 17. 22. 27. 32. 37. (b) (d) (d) (a) (a) (b) (d) (b) 3. 8. 13. 18. 23. 28. 33. 38. (a) (c) (c) (a) (b) (b) (c) (d) 4. 9. 14. 19. 24. 29. 34. 39. (d) (a) (c) (d) (d) (d) (a) (c) 5. 10. 15. 20. 25. 30. 35. 40. (c) (c) (a) (c) (a) (d) (d) (b)

Solution Mock Test5 Solutio the must be 45 = 4 A + m sin 2 . angle

1. We know that ns (Physics) AB = AB cos and A B = AB sin For making between A B both

equal,

2.

Using the relation = sin A 2 . . ( . m = A) A + A sin 2 So, = A sin 2 A 2 sin 2 = A sin 2 cos 2 A sin 2 = A sin A

= 2 cos A 2 3 or 2 A or cos 2 A or 2 or 3. For D.C circuit, V R = I = 100 For A.C circuit 2 2 2 V = I R + L 2 2 2 = I R + (2) L 100 = 0.5 (100)2 + 4 (3.14)2 (50)2 L2 200 = (100)2 + 4 (3.14)2 (50)2 L2 2 or or and 10000 + 98596L = 40000 272 L = 0.55 H R = 100 = 1 100 A = 82 41 = 4 3 = 0.75 = 2 cos 2 A

or or

----------------------- Page 272----------------------Mock 4. Test5 Solutions (Physics) parallel

The given circuit corresponds to combination of three equal resistances of value R. Hence the equivalent resistance between A and B is given by 1 = RAB So, RAB = 3 R R 1 + R 1 + R 1 = R 3

5.

We know Broadcasting through travel with

broadcasting from station is transmitted electromagnetic waves which the velocity of light, so, 8

that

v frequency n = = 6. K.E. of rotation =

3 10 1500 1 I 2 2

= 2 10

5 Hz

= 1 2 Mr2 2 2 5 1 = 2

2 2

2 5

(0.03)
9 J 20

(50)

= 0.45 = 7. 8. Two images are formed, one due to upper half and other due to lower half. Using the formula e or Given I = R : R = 100 , 2 Area A = = 25cm Magnetic field = 100 T s or I = = 25 10 4 100 = 0.5 A. 9. According to question, potential due + 16 unit charge and 9 should be same in magnitude to give zero potential i.e. 1 16 9 = 40 x 40 (7 x) or x = 4.48 cm = R for emf e = IR NA(dB/dt)

4 m

20 25 10

1000

to unit charge

273 ----------------------- Page 273----------------------Mock 10. Test5 Solutions (Physics) 3 Hence, for monoatmic gas Cv = 2 5 for diatomic gas, Cv = 2 R and average R and

For Meyers formula Cp Cv = R

3R 2 + 5R 2 Cv = 2 C p So, mean value of Cp = Cv 11. We know that remains same medium is what so ever. Here, F = 3 cm Effective focal 1 = F 1 So, F 1 or 3 Hence, 13. = 3f f = 4 cm unit of displacement = 3f 4 f 4 = 2 the when frequency the of change light of 3 = 1.5 = 2R + R = 3R = 2R

12.

length of eye piece 1 + f 1 3f 2 = 3f 4

is

We know that the current is ampere. d e = dt

14.

or d = edt

(1)

8 Here : e = 4 10 volt, dt = 0.1 sec Putting the values of e and dt in eq. (1), we get d = 4 108 0.1 = 4 109 Wb 15. We know that length is immaterial fuse wire because melting of a fuse wire is independent of length of fuse wire. We know that thermal radiations consists of larger wavelength as compared gamma rays and wavelength in visible regions and so, thermal radiations belong to infra-red region. Musical notes 320 240 274 interval of = 1.33. produced between frequencies two is given for

16.

to

17.

by

----------------------- Page 274----------------------Mock 18. Test5 Solutions (Physics)

From the formula, areal velocity is the area swept in unit time, the areal velocity area = time Dimensions of area velocity dimensions of area = dimension of time = [L] [L] [T] 0 2 = [M L T 1 ] 2 [L ] = [T]

19.

Escape velocity from the surface of earth is given by 2Gme ves(e) = R e Here : v es (e) R p 2GM v es(e) = R p p = = 2R e 2G 4M 2R e get e (2) = 11.2 km s, M p = 4M , e (1)

Dividing eq. (2) by eq. (1), we So, ves(p) = 2 ves(e)

= 1.414 11.2 = 15.8 m s 20. Suppose m , v and m , v 1 1 are the masses 2 2 Since, kinetic

and velocities of the balls. energy is same i.e., K.E1 = K.E2 1 So, 2 2 (m v 1 1) or = m v 1 1 2 2 = 1

2 m v 2 2 2

(m v ) 2 2

m1 2 p1 or m1 i.e. Therefore, greater momentum. 21. the = m2 p m heavier p2 2

m2

ball

will

have

From the formula P = Fv = mav = constant dv or Integrating = constant dt eq. (1), we get 2 v = kt 2 1 2 or dx or dt Integrating = k t 1 v = k t 1 1 2 v

(given) (1)

(2)

eq. (2), We get x = k2 t3 2 275

Hence, distance moved is proportional to 3 2 t . ----------------------- Page 275----------------------Mock 22. Test5 Solutions (Physics) number of

From the formula, the molecules in V litres of gas at NTP is NV = 22.4 It represents that at NTP, all have same number of molecules in same volume. Hence, ratio of molecules in both cases is 1 : 1.

the

gases

23.

We know that Lyman series ultraviolet region which wavelength between 1215 to Balmer series lies in visible region. Other

lies have 912.

series lie in 24.

infrared-region. point on the (1) (2)

The velocity at the highest vertical circle is v = rg 2r and From eqs. (1) T = v and (2), we get 2r T = rg = 2 r g

= 2 3.14 4 sec 25. From the formula, 1 3 R = R0 A 1 3 RS Hence, RHe = AHe 1 3 32 = 4 1 3 = (8) 26. In the given by interference pattern, D 1 = In second doubled, then 2 = case 2D 2d when the = 2 the AS

4 9.8

fringe

is

(1) distance is

= 2 2d 2 = 2 1 = 2 2 = 4

(2)

276 ----------------------- Page 276----------------------Mock 27. Test5 Solutions (Physics)

From formula

for resistance

l R = A =

l 2 r

In second case, the resistance of another wire in which d = 2d or r = 2r and l = 4l l R = r 4l = (2r) l = r Now 28. Using from equations (1) and (2), R = R the relation L = L 0 c 2 c = 1 2 4c = 1 1 4 3 = 4 29. Pressure on the bottom given by P1 = 90 + 10 = 100 m of water Let its volume be V1 At the surface, P2 = 10 m of water the of the lake is = 0.866 m 1 1 v2 2 Here v = c 2 2 (2) 2

pressure

will

be

Now, on reaching the surface its volume be V . 2 Now according to Boyles law P V = P V 1 1 2 2 100 V = 10 V2 V2 = 100 V 10

= 10V Thus, it becomes 10 times. 277 ----------------------- Page 277----------------------Mock 30. Test5 Solutions (Physics) 5 Wb m 2

Given : H = 1.8 10 Angle of dip = 30 Using the formula H = V cos H V =

cos 1.8 10 cos 30 1.8 10 = 3 2 1.8 10 3 2 = 2.08 Wb m 31. From the formula RL AV1 = RP + RL 40 8 10 4 10 3 + 8 10 3 3 3 3 5 5 5

or

V =

40 8 10 12 10 40

= 1.5

(1)

AV2 =

40 8 10 3 8 10 + 8 10 40 8 10 16 103 3

3 3

40 = 2 (2)

= Now

from equations (1) and (2) AV1 40 2 = AV2 1.5 40 4 = 3

32.

Using the expression for adiabatic change TV 1 = constant Hence, T V 1 1 1 = T V 2 1 2

(Here : V1 = V, V2 = 2V, T 1 = T) T 2 T1 V 1 1 = V2 = V 1 2V = 1 1 2

10.4 = 2 T 1 T2 = 1.32 = 1.32 300 = 227.27 K 278

----------------------- Page 278----------------------Mock 33. Test5 Solutions (Physics) T From the relation P 1 P T 1 But From P T3 equations (1) and (2), we get T3 = T ( 1) (1) (given) = constant

or 1

= 3 3

or

= 2 3RT

34.

We

have v rms = M T vrms M vO2 TO2 = TH2 1200 = 300 1 = 2 vH2 2 1930 = 2 = 965 m s stress (1) (2) MH2 MO2

Hence, vH2

2 32

or

vO2 =

35.

From the formula

for thermal 1

(F A) (F A) 2 (Since

= Y T 1 1 = Y T 2 2

temperature T is same) (F A) 1 = (F A) 2 Y 2 2 Y 1 1

For thermal

stress to be equal) Y = Y 1 1 2 2 Y1 2 = Y2 1 = 2 3 as 1 = 2 : 3 2

or 36.

Since, trolley is standing on frictionless surface, then it will move with velocity v. Applying law of conservation

of

momentum

80 1 = (320 + 80)v

So, v = 0.2 m s Then the relative velocity of the man with respect to earth = 1 0.2 = 0.8 m s After 4 sec, the ground is given by displacement relative to 279

= 0.8 4 = 3.2 m ----------------------- Page 279----------------------Mock 37. Test5 Solutions (Physics) rise of liquid in

From the formula, capillary tube is 2T cos h = g T cos h

Angle of contact for water and glass 1 = 8 Angle of contact for mercury 2 = 135 h 1 Hence, h 2 T 1 or T 2 = 0.342 1 = 65 38. When the ball falls freely on the ground and rebounds its starting acceleration is instantaneously changed but the magnitude remains the same. point, the = h cos 2 2 0.1 1 1 13.6 = 1 T 1 cos 1 and glass (given)

2 2

T cos 2
2

h cos 1 1

cos 135 cos 8

39.

We know that when cornea of eye is not spherical, then human with astigmatism.

eye

is

suffered

40.

We know that the level of liquid increases, remains same or becomes down. On account of the coefficient of cubical expansion of liquid is more, equal and less than the cubical expansion of the metal of container. Hence, when liquid is heated, it will overflow so that cubical expansion will be greater than 3. 280

----------------------- Page 280----------------------REASONING (Mock Test-5) 126. asoning) (a) NDISB (c) NDGSB 127. ADHM : ZWSN : : CFJO : ? (a) YURM (c) XUQL (b) NDHSI (d) NDIUB (b) WTPK (d) ZXVT If ZUBIN is coded MEHTA will be coded as : as ATCHO, then Mock Test5 (Re

128.

Among five boys, Vasant is taller than Manohar, but not as tall as Raju. Jayant is taller than Dutta, but shorter than Manohar. Who is the tallest in the group : (a) Raju (b) Manohar (c) Vasant (d) Jayant Foot is related to shoe, in the same way Neck is related to : (a) necklace (b) cut (c) beauty (d) ornament Tea : Leaves :: Coffee : (a) plant (c) beverage (b) seeds (d) stimulant

129.

130.

131.

Engineer : Machine :: Doctor : (a) disease (b) medicine (c) hospital (d) none of these Qs mother is the sister of R and daughter of S, N is the daughter of R and sister of M. How M is related to S ? (a) son (b) brother (c) daughters son (d) data inadequate Introducing a man, a woman said, he

132.

133.

is the only son of my mothers mother. How is the woman related to man ? (a) mother (b) cousin (c) niece (d) none of these 134. In a certain written as PERFECTION is written (a) NOICTEFREP (b) NOITCEFRPE (c) NOITCEFPER (d) NOITCEFREP code INSTITUTION NOITUTITNI. : is How

Directions (for Q. 135) : Find the missing number in the series. 135. 1, 2, 3, 5, 7 (a) 9 (c) 10 .......... : (b) 8 (d) 13 281 ----------------------- Page 281----------------------Answer Sheet (Mock Test-5) (REASONING) 126. 131. (a) (a) 127. 132. (c) (d) Solution one in the Mock Test5 Solutio 128. 133. (a) (c) 129. 134. (b) (d) 130. 135. (b) (c)

Hints & 126.

Letters at the odd places are coded letter ahead, and letter at the even places are coded one letter behind alphabetical order. ns (Reasoning) 127.

ADH and M are the Ist, 4th, 8th and 13th placed alphabet from the left to right in the series of alphabets. Similarly, ZW, S and N are Ist, 4th, 8th and 13th placed alphabet from the right to left. Therefore, CFJO has same relation with XUQL. Final ranking of height in descending order is as Raju-Vasant-ManoharJatant-Dutta. Hence, Raju is tallest one. As shoe is used in neck. used in foot and necklace is

128.

129. 130. 131.

Coffee is obtained from seeds. As the machines diseases. From the above engineer similarly is directly doctor is related related ti to

132.

question, it is clear that

S is the mother of R (not confirmed R is either male or female). On the other hand N is the daughter of R and sister of (whose sex is also not known). If malk then M will be grand daughter/son of S in case of R is female than M be maternal daughter/son of S. Hence, as per the options data is inadiquate. 133. Son of womans mothers mother means the brother of mother. mother is maternal uncle. Hence, woman is related as niece to the man. Letters of the word INSTITUTION have been just reversed in Therefore, PERFECTION will be as NOITCEFREP. Ans

M R will

is

brother

of

134.

the

coded

word. coded

135.

Series moves with a difference of 1, 1, 2, 2, 3, 3 and so on. Hence, number is 10.

the

missing 282

Das könnte Ihnen auch gefallen